You are on page 1of 158

P-156 GSM-54

Sp-5.4756 mm

Price : 150.00
2105H0306P7740
An ISO 9001:2015 Certified Company
HOLY FAITH

10

ANSWER KEY

HOLY FAITH INTERNATIONAL (P) LTD.

HF_Acme_ENG_TM_G10.indd 1 5/22/2021 5:58:37 PM


Section-A: Reading

Chapter 1 Factual Passages


ANSWER KEY

Passage 1
1. (b) Horses
2. (b) They come as processed foods which are consumed with milk like porridge.
3. (b) Because of its gluten content
4. (c) Because the higher concentration of beta-glucan in oats lowers bad cholesterol
5. (a) Celiac 6. (d) Beta-glucan fibre
7. (c) One cup of oatmeal serving contains approximately 2mg phosphorus.
8. (b) beta-glucan 9. (iv) F – (b), (c), (d) and O – (a)
10. (a) Because in her opinion the human body is healthy when the food is varied
11. (ii) Option (b) 12. (a) Gut

Passage 2
1. (iii) Option (c) 2. (b) Veggie Van
3. (a) Joshna and Kaia 4. (d) Germany
5. (d) It has been recognised as an official alternative fuel in the US.
6. (c) without any modification 7. (d) 1000 per cent
8. (c) It can work on its own as well as by blending with diesel.
9. (iv) F – (a) (b), (c) and O – (d)
10. (b) Glycerin 11. (a) an adverb 12. (a) Altered

Passage 3
1. (ii) Option (b) 2. (d) It has led to the rise of sea level.
3. (b) It has resulted in untimely migration of animals.
4. (a) Bill Fraser 5. (a) between 7 and 20 inches
6. (a) Floods and droughts 7. (b) Rainfall
8. (b) it will totally evaporate by 2100
9. (b) Wildlife research scientist
10. (a) Biologist 11. (iii) Option (c) 12. (b) Skinnier

Passage 4
1. (i) Option (a)
2. (d) Diabetes mellitus is affecting children and teenagers who are obese.
3. (a) Link between obesity and television viewing
4. (b) Because reading engages the minds of the children more emphatically

2 ACME English Practice Book GR–10 (Teacher Manual)

HF_Acme_ENG_TM_G10.indd 2 5/22/2021 5:58:37 PM


5. (d) Both (a) and (b)
6. (b) Television viewing induces a person to eat more food.
7. (a) It has tempted children to eat more.
8. (a) Because there is less regulation on advertisement on television
9. (c) they have put restrictions on children’s advertising
10. (c) Both (a) and (b) 11. (i) Option (a) 12. (a) Trigger

Passage 5
1. (c) hydrogen and helium 2. (d) Neptune has an inner heat source
3. (d) frozen ice, dust and rocks 4. (b) Uranus has no internal heat source
5. (b) a region of the solar system that exists beyond the eight major planets
6. (b) Helium is found in abundance on Jupiter. 7. (c) Neptune
8. (b) Uranus 9. (ii) (c), (d), (f) 10. (b) Pluto
11. (a) Orbit 12. (b) Distinctive

Passage 6
1. (iii) Option (c) 2. (d) All of the above
3. (b) Clots in the blood 4. (c) Mango
5. (c) Both (a) and (b)
6. (c) They prevent blood platelets from sticking together.
7. (a) Flax seeds 8. (d) All of these
9. (c) improve immunity and enhance energy
10. (d) They prevent the platelets from clumping.
11. (b) Ailment 12. (iv) Option (d)

Passage 7
1. (c) both (a) and (b) 2. (a) flexibility
3. (a) what contributes to the overall well-being of a person
4. (d) All of the above 5. (a) anxiety, insomnia, pain, depression and fatigue
6. (b) confidence 7. (c) chronic lower back pain
8. (ii) F – (b), (c) and O – (a), (d)
9. (b) Ryan Giggs 10. (d) proper alignment and execution of the asanas
11. (b) execution 12. (i) Option (a)

Passage 8
1. (c) the problem of finding a charger and charging spot everywhere
2. (d) it helps charge your smartphone, iPod or any other handheld device by setting it
down on a high-tech charge pad.
3. (b) Israeli 4. (c) the duration of the charge
5. (a) `2700 6. (a) the vice president of global marketing
7. (iii) (d) and (e) 8. (d) to build a truly wireless world
9. (c) Their cars can be charged on Powermat as big as a car’s area.

Learnwell
ACME Math–III
ENGLISH Practice Book GR–10 (Teacher Manual) 3

HF_Acme_ENG_TM_G10.indd 3 5/22/2021 5:58:37 PM


10. (iii) F – (a), (c), (d) and O – (b)
11. (c) Eradicate 12. (ii) Options (b) and (c)

Passage 9
1. (a) the contribution of agriculture to the GDP is only 14-15% whereas more than
60% of the population depends on agriculture for livelihood
2. (a) deprivation of farmers from attaining new knowledge to reduce workload and
increase productivity of their fields
3. (b) 14-15 per cent 4. (a) to describe the state of agriculture
5. (a) modernising small landholdings
6. (iv) (f) 7. (d) Less than 1 percent
8. (c) 6 percent 9. (d) Both (a) and (b)
10. (iii) F – (a), (b), (d) and O – (c)
11. (a) Allocation 12. (i) Option (a)

Passage 10
1. (d) there is a change in the national currency
2. (b) November 2016 3. (b) 86% 4. (c) gold market
5. (b) PAN card 6. (a) This statement is true.
7. (c) Time till the end of the year
8. (a) To remove black money accumulated from money laundering
9. (b) one failed to provide proof of tax payments
10. (i) F – (a), (d) and O – (b), (c)
11. (c) Eradicate 12. (i) Option (a)

Chapter 2 Discursive Passages

ANSWER KEY

Passage 1
1. Humans cannot satisfy most of their psychological needs like the need for prestige,
recognition, companionship, security, etc. through their own individual efforts. They
frequently require the cooperation of the fellow members in this regard. Living in
groups enhances the prospect of survival. This leads to formation of civilisation.
2. The positive effect of conflicts is that they change the structure of the group, interactions
and relationships of the fellow members. They help the group to adapt better to
the changing environment and maintain harmony within the group and increase
their understanding which helps to face the impinging challenges of the dynamic
environment strongly.
3. A diverse society contains many smaller groups divided according to different criteria
like religion, economy, gender, caste, political and ideological inclination, geographical
location, etc.

4 ACME English Practice Book GR–10 (Teacher Manual)

HF_Acme_ENG_TM_G10.indd 4 5/22/2021 5:58:37 PM


4. The development of local group identities can threaten the integration of the society
as an entity. This is seen in many communal conflicts. Considering one’s religion
superior and the desire to propagate its ideology, have caused clashes within different
religious sects of a society.
5. (iii) (b), (d), (e) 6. (d) Option (b) and (c)
7. (b) Because of living in groups
8. (b) Pertinent
9. (c) Alone we can do so little, together we do so much more. - Helen Keller
10. (a) Integrate 11. (c) in the distant past
12. (b) Diversity and Social Integration

Passage 2
1. Extra-curricular activities teach time management, leadership, responsibility,
teamwork, etc. to students and provide them a platform to understand, test and
execute academic theory in real life.
2. Learning conflict-resolution skill helps students learn to listen to each other, to maintain
peaceful and supportive body language and to determine what is best for the collective
whole rather than the individual.
3. Time management teaches the students how to complete any activity on time. They
learn to manage time to participate in various activities, attend school, do their
homework, maintain their grades, spend quality time with friends and family and get
adequate sleep.
4. ‘This’ refers to the skill of time management. They learn to manage their time efficiently
between various tasks which help them to attain success in life.
5. (b) Suppress 6. (c) Concealing
7. (a) Sufficient 8. (iv) F - (b), (c) and O - (a), (d)
9. (a) to be in a leading position in an important activity
10. (a) Communication skills 11. (a) Take feedback constructively
12. (a) Conflict-resolution

Passage 3
1. If we suffer from decidophobia, we will not be able to take decisions because of the
fear of failure, and this is very risky. If we avoid taking a decision, we leave everything
to chance and float along life with a ‘what will be, will be’ attitude.
2. We can save ourselves from taking wrong decisions by gathering as much information
as possible about the issue under consideration, by setting some guiding principles in
our life, and sometimes by postponing the decision when situations seem at odds.
3. It is appropriate to defer decision-making when situations seem at odds because
circumstances don’t remain the same and then it may be easier to take the decision
when it is the right time.
4. We can get the feedback of our decisions only when we follow-up and act upon them.
5. (a) Fear of taking wrong decision 6. (iv) (a), (d), (e)
7. (b) Decisions should be made at the moment so as not to lose any opportunity.
8. (iii) F – (a), (d); O – (b), (c)

Learnwell
ACME Math–III
ENGLISH Practice Book GR–10 (Teacher Manual) 5

HF_Acme_ENG_TM_G10.indd 5 5/22/2021 5:58:37 PM


9. (a) In the arena of journalism, the responsibility of delivering the truth becomes the
most difficult.
10. (b) adverb 11. (a) make something easier 12. (iv) Option (d)

Passage 4
1. Media serves the country by conveying authentic news to the citizens and helping
in shaping the image of country. It exposes various scams, scandals, embezzlements
and many other cases of corruption. Mass scale coverage by the media helps in the
initiation of inquiries and other processes of prosecution against the accused.
2. Growing consumerism and materialism can hamper the honest and unbiased
functioning of a media organisation. News is often manipulated to influence the young
minds. Unnecessary sensationalism of an issue may project wrong perception to the
public.
3. Sometimes, a news piece or an event or a celebrity is given undue importance due
to media bias or personal preference by a journalist or an editor. This sets a wrong
ideal before the youth. News is often manipulated to influence the young minds.
Unnecessary sensationalism of an issue may project wrong perception to the public.
4. Educational institutions can play a role in negating the negative influence of the media
by educating and training the youth to be more discerning viewers rather than passive
spectators or participants.
5. (b) Media is the third pillar of democracy.
6. (d) Misinterpretation 7. (b) Training by educational institutes
8. (a) It is alleged that Shreya manipulated the other teams to win the task.
9. (iv) F—(c), (d) and O—(a), (b)
10. (c) Adversely 11. (a) Prejudice 12. (i) Option (a)

Passage 5
1. The printed sheets of the newspaper are considered the gospel of truth by many people.
2. The duty of the news editors is to check the authenticity of the news because a false
report may excite the public and raise a ruckus.
3. The public should be aware and serve as a watchdog by keeping a strict vigil on the
newspapers and help them to maintain the ethics of the news. It should not patronise
inflammatory or indecent news.
4. The youth is often misled by inaccuracy, superficiality and one-sidedness that have
crept into modern journalism.
5. (c) An aware public 6. (c) For monetary benefits
7. (a) Uncontrolled volume of water
8. (b) The Responsibility of the Press
9. (a) Sacred truth 10. (a) Watch 11. (a) Submerges
12. (iv) Option (d)

Passage 6
1. When people are influenced by new ideas, changes occur in the society. Writing helps
in propagating ideas and thoughts. Men through their writing have achieved much
more than through weapon power.

6 ACME English Practice Book GR–10 (Teacher Manual)

HF_Acme_ENG_TM_G10.indd 6 5/22/2021 5:58:38 PM


2. (a) The pen signifies writing, and the expression of ideas.
(b) The sword signifies violence and destruction.
3. The major national newspapers, The Hindu, The Tribune, The Bengalee, The Kesari
and the Amrita Bazar Patrika waged a campaign against the British rule.
4. Written words have the power to change the world with noble thoughts. They make
a permanent impact on several generations. They have moulded life through human
history.
5. (b) synonymous 6. (a) Voltaire 7. (iv) Option (d)
8. (c) The Civil Disobedience 9. (d) well-behaved 10. (b) dictatorship
11. (b) Noun 12. (a) dignified

Passage 7
1. Endorphins released by laughter initiate an overall sense of well-being and can remove
pain for some time. They relieve physical tension and stress, and relax muscles up to
45 minutes.
2. Endorphins initiate an overall sense of well-being and can remove pain for some time.
They relieve physical tension and stress, and relax muscles. They increase blood flow
and improve the function of blood vessels. It increases immune cells and produces
infection-fighting antibodies.
3. Humour helps one to maintain a positive outlook during difficult situations and
disappointments in life. Laughter gives us the strength and courage to find new sources
of meaning and hope. In difficult times of life, a laugh or even a smile can make you
feel better.
4. When we laugh with one another, a positive vibration is generated. This bond acts
as a strong support against stress, disagreements and losses. Shared laughter is the
most powerful tool for keeping relationships fresh and charming.
5. (iv) F –(a), (d) and O –(b), (c) 6. (c) natural
7. (c) He gave his injury some time to heal.
8. (b) nurtures 9. (e) Improves functioning of the brain
10. (iv) Option (d) 11. (a) Innate 12. (d) Dull

Passage 8
1. We are always under the influence of our mind as we desire and perform according
to our likes and dislikes.
2. We should ignore our likes and dislikes and concentrate on viveka, the ability to
differentiate between what is just and what is unjust, whenever we are confronted
with darkness.
3. Two paths are:
• Path of ignorance: We are not fully aware of our thoughts, beliefs, experiences
and our understanding of different experiences.
• Path of knowledge: Knowledge boosts awareness and consciousness.
4. Through the growth of awareness, development of consciousness and awakening of
our inner potentials, we can reconstruct our life and personality.
5. (a) By practising yoga 6. (d) All of these
7. (b) viveka, confidence

Learnwell
ACME Math–III
ENGLISH Practice Book GR–10 (Teacher Manual) 7

HF_Acme_ENG_TM_G10.indd 7 5/22/2021 5:58:38 PM


8. (c) President is clearly in a dilemma over the farm laws.
9. (i) Option (a) 10. (d) Destiny 11. (c) Recede
12. (a) One should be self aware and seek knowledge.

Passage 9
1. Our forefathers had simple lifestyle, very different from the lifestyle we are leading
today. Forefathers lived in cohesive joint families whereas we live in nuclear family.
2. The term ‘happiness’ has a variety of meanings. It stands for being lucky, fortunate,
content, glad, etc. Its beauty lies in the mind of an individual.
3. People are ready to migrate to the West to enjoy a comfortable life. They want to
become successful in foreign lands and lead a good life.
4. A joint family system facilitates development of a child’s personality and provides a
proper environment to grow. The values of responsibility, respect, tolerance, integrity,
etc. are embedded in the child.
5. (b) Family 6. (b) Detachment from relatives
7. (a) Material wealth 8. (a) joint
9. (c) Happiness and Material Wealth
10. (d) People should focus on developing inner peace and satisfaction and not run after
money
11. (d) Detachment 12. (ii) Option (b)

Passage 10
1. The writer’s concerns himself about falling values of trust and faith in humans in this
passage.
2. The most detestable form of deceit is the one carried under the garb of friendship.
There can be no greater deceit than first to give assurance, and then deceive the
person.
3. Society is meant to promote the good of man, and to provide him dignity, refinement,
and intelligence. But due to deceit, it works mainly as a cunning device by enabling
people to pretend to have unreal virtues and to hide real defect. This defeats the
purpose of society.
4. The nature of every man is a mixture of bad and good, so all men are like actors on
a stage; they take on whatever appearance may suit their present aim, and they act
in keeping with this character.
5. (b) When they have an enemy
6. (c) verbal show
7. (iv) F—(a), (d) and O—(b), (c)
8. (iii) (b), (d), (e)
9. (d) Evil brought out stops to appear as evil as it is in reality.
10. (b) Refinement 11. (iii) Option (c) 12. (b) Potent

8 ACME English Practice Book GR–10 (Teacher Manual)

HF_Acme_ENG_TM_G10.indd 8 5/22/2021 5:58:38 PM


Section-B (B1: Writing)

Chapter 1 Formal Letters


ANSWER KEY

Letter to Editor
1. 35, Mall Road
Kanpur

6th June 20xx

The Editor
The Times of India
ITO
New Delhi

Sub: Role of government and public in development of tourism

Sir
Through the columns of your esteemed newspaper, I would like to give my opinion
on how the government and the public can contribute to the development of tourism.
Development of tourism infrastructure requires heavy investment in building a variety
of sites, hotels and a good network of transportation. Many of these activities are
in the form of public utilities like roads, power, water, etc. The government should
judiciously use funds to build and maintain these facilities. Apart from investing in
provision of basic facilities at tourism sites, launching awareness programmes and
undertaking requisite preservation efforts should be done by the government. General
public too must ensure safety and comfort of tourists by being cooperative with them.
They must also ensure preservation of tourist attractions by not spoiling them and
keeping their surroundings clean. People can also use social media to promote their
tourist destinations. They can share pictures, videos, events announcements, and
blogs across social media channels. Tourism contributes a lot in improving economic
condition of an area and hence, suitable steps must be taken to ensure continuation
and development of tourism.

Thanking you
Yours sincerely
Rajesh

Learnwell
ACME Math–III
ENGLISH Practice Book GR–10 (Teacher Manual) 9

HF_Acme_ENG_TM_G10.indd 9 5/22/2021 5:58:38 PM


2. M-3, Karol Bagh
New Delhi

6th June 20xx

The Editor
The Times of India
ITO
New Delhi

Sub: Consequence of climate change happening due to various activities of man

Sir
Through the columns of your esteemed newspaper, I would like to draw the
attention of masses towards the consequence of climate change happening due
to various activities of man. Human induced activities are adversely affecting the
ecological balance of earth, leading to large scale droughts, earthquakes, tsunamis,
landslides and other natural disasters. Some of the human activities like burning
fossil fuels, releasing chemicals into the atmosphere, unplanned urbanisation and
industrialisation, deforestation, the rapid expansion of agriculture lands, dumping
industrial and agricultural wastes into water sources, encroachment of land and water
are contaminating environment and adversely affecting the balance of the ecosystem
and climate. Climate change leads to melting of polar ice caps, rising sea levels,
extreme weather, shifting rainfall patterns, etc. Consequences of climate change will
be highly detrimental to survival of humans. It will also endanger most animals and
marine life. If some serious steps are not taken to control climate change, it will put
a threat to the lives on earth.

Thanking you
Yours sincerely
Yakshi

3. M-31, Connaught Street


New Delhi

6th June 20xx

The Editor
The Times of India
ITO
New Delhi

Sub: Consequence of rising number of vehicles and suggestions to control

Sir
Through the columns of your esteemed newspaper, I would like to draw the attention
of masses towards the consequences of growing number of vehicles on the roads.

10 ACME English Practice Book GR–10 (Teacher Manual)

HF_Acme_ENG_TM_G10.indd 10 5/22/2021 5:58:38 PM


Excessive traffic on the road causes not only a great inconvenience to the people
but has also become a health hazard for them. Smoke produced by road transport
is a major contributor to air pollution. It is highly harmful for human health and the
environment. Vehicles emit a range of pollutants including nitrogen oxides (NOx),
carbon monoxide and other harmful gases. Regular exposure to heavy traffic can
lead to various adverse health consequences. It has been confirmed that exposure to
pollution can significantly increase the risk for diseases, such as high blood pressure,
heart attack, anxiety, etc. There can be only one solution to this problem—public
transport. Public transportation can transport a large number of people in less space
than individual modes of transportation, which helps reduce traffic jams, which in turn
reduces air pollution. Public transportation produces significantly less air pollution
per passenger than a single car carrying one or two people. So, we must adopt
the habit of using public transport to help protect environment and human health.

Thanking you
Yours sincerely
XYZ

4. 11, Noida
Uttar Pradesh

10th June 20xx

The Editor
The Tribune
Mandi House
New Delhi

Sub: To express concern over unrestrained killing of animals

Sir
Through the columns of your esteemed newspaper, I would like to generate awareness
about unrestrained killing of animals for fun, fur and fashion. The craze for products
made of leather and fur leads to killing of thousands of foxes, deer, elephants,
crocodiles, tigers, snakes, etc. for their skin and fur. People don’t hesitate to use
cosmetics, chessboard pieces, belts, purses, fur caps and jewellery made from animal
body parts. This unchecked hunting of animals is disturbing the ecological balance of
the earth. Indiscriminate hunting of animals will disturb the food chain as many animals
have already become extinct or are at the verge of extinction. It’s high time we put a
check on animal hunting for fulfilling personal greed. NGOs, government agencies
and public should join hands together to stop animal killing. Government should
make stringent laws and must ensure their strict adherence. Exemplary punishment
should be given to those who indulge in animal hunting and poaching. Government
should create more awareness programmes like ‘Save the Tiger’ for other animals.

Learnwell
ACME Math–III
ENGLISH Practice Book GR–10 (Teacher Manual) 11

HF_Acme_ENG_TM_G10.indd 11 5/22/2021 5:58:38 PM


General public can help the government by refusing to buy animal products. With
collective efforts, we can stop this heinous practice.

Thanking you
Yours sincerely
Krishankant

5. 22, XYZ Street


ABC City

6th June 20xx

The Editor
The Tribune
Mandi House
New Delhi

Sub: Regarding the unhygienic condition of Prashad Nagar park area

Sir
Through the columns of your esteemed newspaper, I would like to bring into notice
the unhygienic condition of the garden near the Prashad Lake. I have been visiting
the lake for many years. Of late I have observed that the lake area is increasingly
becoming unclean. I was appalled by the foul smelling water in the lake and the failing
cleanliness standards around the surrounding area. There’s an urgent need to improve
the cleanliness of the park. The residents of the area, who had earlier bought houses
around the area with the hope of enjoying nature, are extremely disappointed by the
foul smell, dirtiness, and unhygienic condition of the lake. They do not send children
to play in the park nor do they go for walking or exercising fearing that they or their
children will catch some disease due to unhygienic and filthy conditions there. The
piles of garbage littered around the park attract mosquitoes and flies that pose a big
threat to the well-being of the residents. The local authorities should look into the
matter and take action to improve the hygiene of the place. Parks are important for
a community for recreation and it’s quite necessary that people should have clean
and healthy atmosphere.

Thanking you
Yours sincerely
Anmol

6. 65, Avenue Road


New Delhi

11th May 20xx

12 ACME English Practice Book GR–10 (Teacher Manual)

HF_Acme_ENG_TM_G10.indd 12 5/22/2021 5:58:38 PM


The Editor
The Tribune
Mandi House
New Delhi

Sub: To raise awareness about the value of girl child

Sir
Through the columns of your esteemed newspaper, I would like to generate awareness
about the need for equality and how it can be achieved by giving value to the girl-child.
As the president of ‘Girls Association of India’, I want to make the people realise that
we can correct the gender imbalance by providing for our daughters as we do for our
sons. A large part of the Indian society generally considers girls inferior to boys and
hence they are given secondary treatment. This leads to a number of gender issues
such as high drop-out rates among girls from school, widows being seen as a burden,
female foeticide, unequal pay, etc. We can eradicate the root cause of discrimination
only by changing our attitudes towards the girl child. The society needs to consider
a girl child as deserving as a male child. Women status in the society can only be
improved by providing them education, equal opportunities to grow and prosper,
and an atmosphere where they feel valued and can hold up their heads with dignity
and enjoy the freedom that men do.
Our goodwill ambassadors for “Save the Girl Child Campaign” are the tennis sensation,
Sania Mirza; the Junior Squash Champion, Joshna Chinappa and the All India Topper,
Aruna Kesavan. We all are working together tirelessly to bring about a change in the
society. We want to appeal to the public to join us in this noble cause.

Thanking you
Yours sincerely
Sabha Khan

7. 70, Karnal Road


Kaithal

12th May 20xx

The Editor
The Times of India
Mandi House
New Delhi

Sub: To raise awareness about the need to save water

Sir
Through the columns of your esteemed newspaper, I would like to generate awareness
about the need to save water and raise the water table. Water is the most precious

Learnwell
ACME Math–III
ENGLISH Practice Book GR–10 (Teacher Manual) 13

HF_Acme_ENG_TM_G10.indd 13 5/22/2021 5:58:38 PM


commodity on earth after oxygen. Underground water is at dangerously low level.
Human activities are endangering its availability. Bottled water is a thriving business
in India. Companies who are engaged in selling water pay no attention to the need
of conserving water. Water pollution is rampant everywhere. No heed is paid by the
factories before dumping factory waste into rivers and other sources of water. Rivers
are getting polluted, making water unsafe for drinking. Water resources are facing a
serious threat of depletion. Rain cycle is disturbed due to increasing pollution and
rampant deforestation. Wells, hand pumps and tube-wells have been sucked dry
because of less seepage of water and overuse. If water table falls below the normal
level, farmers will be unable to cultivate crops. This will lead to shortage of food
throughout the world. The entire human race will face danger of extinction if we go
on wasting and polluting water. Stringent and urgent steps should be taken to save
water such as rain water harvesting, drip irrigation, plantation of trees, check on water
pollution, etc. Both the public and the government should come together to check
the misuse of water.

Thanking you
Yours sincerely
Rekha

8. Sector 44
Gurugram

13th June 20xx

The Editor
The Times of India
Mandi House
New Delhi

Sub: Suggestions to make educational curriculum more flexible

Sir
Through the columns of your esteemed newspaper, I would like to suggest some ways
through which our educational curriculum could be made more flexible and relevant
for the contemporary society. The educational curriculum should be made keeping in
mind the changing requirements and environment of our society. Rote learning and
rigid syllabus doesn’t give students a chance to reach their full capabilities. The students
should be given not only theoretical knowledge but practical one too. Tasks which
involve doing researches, project work, surveys, etc. should be incorporated in the
syllabus to make learning real and experience based. Skill-based and aptitude based
teaching should be promoted in schools. Vocational training based on instructional
programmes or courses that focus on the skills required for a particular job should
be provided in educational institutes. Vocational education provides students edge

14 ACME English Practice Book GR–10 (Teacher Manual)

HF_Acme_ENG_TM_G10.indd 14 5/22/2021 5:58:38 PM


in job searches, since they acquire certifiable knowledge that they need to enter
the field. School should focus more on skills like work ethic, creativity, teamwork,
critical thinking, problem solving, emotional intelligence, interpersonal intelligence,
etc. Education should be made more learner centric that gives value to curiosity,
individuality and creativity. It should also be fun, exciting, more about the learning
and preparation for the real world after school.

Thanking you
Yours truly
Saksham

9. 36-D, Ram Nagar


Lucknow

4 April 20xx

The Editor
The Times of India
Mandi House
New Delhi

Sub: Expressing views over allotment of land for residential colony

Sir
Through the columns of your esteemed newspaper, I would like to express my
dismay over allotment of a 65 acre plot of land reserved for garden and green belt
for residential colony by Lucknow Development Authority. This decision of the
authority is facing a strong opposition from the residents of the nearby areas and
the environmentalists. Forest land conversion will lead to ecological imbalance. The
forested area has grassland, bushes and marshes. It is also home to many kinds of
wild animals, migratory birds, snakes, butterflies and many more species. The area has
taken hundreds of years to build up and cannot be re-planted overnight. Destruction
of the green belt can cause changes in water quality and wildlife habitat, and quality
of soil and air. It will also rob enthusiasts of one of their favourite cycling or trekking
venues. So, the decision by Lucknow authorities should be rolled back. Moreover,
locals and environmentalists are planning to go on indefinite hunger strike to protest
against the conversion of the green belt.

Thanking you
Yours truly
Rajesh Mishra

Learnwell
ACME Math–III
ENGLISH Practice Book GR–10 (Teacher Manual) 15

HF_Acme_ENG_TM_G10.indd 15 5/22/2021 5:58:38 PM


10. 36, Chawri Bazar
New Delhi

4 April 20xx

The Editor
The Times of India
Mandi House
New Delhi

Sub: Request to increase the number of ‘University Special’ buses

Sir
Through the columns of your esteemed newspaper, I would like to request the
transport administration to increase the number of ‘University Special’ buses on
different routes of the city. Due to expansion of education, more and more students
are coming from different parts of the country to get university education. It has led
to increase in demands for more number of buses and other means of transportation
to reach the educational institutions. But the availability of buses is inadequate in
number. In such a situation, students face difficulties to reach their colleges, especially
during exams when they have to reach their examination centres on time. This has
led to several problems like strikes, road blocks, fights and even increase in crimes.
This is especially unsafe for female students and teachers as they have to walk long
distance or wait at the bus stops for a long period of time. There is an urgent need
of providing more ‘University Special’ buses on different routes so that college going
students can travel easily to their institutions. Overcrowded buses are unsafe for all
passengers as well as a few accidents have also been reported where people have
fallen down from the buses while travelling on the footboards. I hope that through
your newspaper the authorities will become aware of the issue and take swift remedial
action.

Thanking you
Yours truly
ABC

Letter of Complaints
1. Sector 20
Ambala

20th March 20xx

Kamal Electronics
Mall Road
Karnal

16 ACME English Practice Book GR–10 (Teacher Manual)

HF_Acme_ENG_TM_G10.indd 16 5/22/2021 5:58:38 PM


Subject: Complaint against defective refrigerator

Dear sir
I am writing to express my utter dismay at the poor quality of the refrigerator supplied
by you in response to my order dated 12th March 20xx, vide cash memo number
25896/96.
The order was for Whirlpool Refrigerator of 300 litres. Within a month, the freezer of
the refrigerator has stopped working. It fails to do the cooling properly due to which
a lot of our food items have gone waste. Also, it produces a loud, clanking sound at
times. Kamal Electronics is a leading dealer in electrical appliances and the company
enjoys a good reputation and goodwill in the market. This kind of product quality is
not expected from your end. As the purchase is within the warranty period, I hope
you will replace the defective refrigerator at the earliest.

Thanking you
Yours sincerely
Arun

2. 712, Worli
Mumbai

30th March 20xx

The Superintendent of Police


Worli, Mumbai

Sub: To register complaint about the poor law and order situation

Dear sir
I am writing to express my utter dismay at the poor law and order situation in our
area. Incidents of chain snatching, eve teasing, bullying, robbery and looting of shops
have become so common that people are afraid of stepping outside their houses
or keeping their shops open after 7 pm. Residents of the area hesitate to celebrate
festivals and organise functions because of the fear of hooligans. They have no safety
and peace of mind due the prevailing law and order scenario. In spite of filing several
complaints to the local police station, no action has been initiated till now. The local
police station should be instructed to take quick and timely action to resolve residents’
grievances. Police patrolling should be increased in the area, especially during night
time. Lady constables should be deployed at busy places to ensure safety of women.
They should also be deployed in public transport. Security of shopkeepers should
be ensured by appointing more policemen in the area. I hope my suggestions will be
considered as serious and acted upon in an urgent manner.

Thanking you
Yours sincerely
Chirag

Learnwell
ACME Math–III
ENGLISH Practice Book GR–10 (Teacher Manual) 17

HF_Acme_ENG_TM_G10.indd 17 5/22/2021 5:58:38 PM


3. St Mary’s School
New Delhi

10th April 20xx

The Manager
Royal Tourist Service
New Delhi

Sub: Complaint about poor services

Dear sir
On February 27, 20xx, I bought a tour package for 30 students and 5 teachers from
your tourist service agency. Unfortunately, your services were not as promised under
the package deal. The arrangements done by the agency were sub-standard. The
accommodation and food arrangements were poor. The hotel in the tour package was
supposed to be a three-star hotel but the one provided was a two-star hotel. Also, to
our utmost disappointment, transport facility was not made available as mentioned
in our tour package. We had to hire our own mode of transportation leading to extra
expenditure and discomfort. The quality of food at the hotel was very poor. Some of
the students refused to eat food there. Most of the activities listed in the tour package
were either not available or were of sub-standard quality due to which we had to
suffer a lot and we had to spend extra money too. As an aware customer, I demand
the compensation for services you failed to provide us. Kindly resolve the matter at
the earliest.

Thanking you
Yours sincerely
Sameer

Letters of Placement of An Order


1. Rosemary Public School
D – 85, Preet Vihar
Yamuna Nagar

9th January 20xx

The Manager
M/s Babbar and Sons
Daryaganj
New Delhi

Subject: Placing an order for books

18 ACME English Practice Book GR–10 (Teacher Manual)

HF_Acme_ENG_TM_G10.indd 18 5/22/2021 5:58:38 PM


Dear sir
We wish to order some books for our school’s library. The number of items and their
quantities for the purchase are mentioned below:
S. no Particulars Quantity
1. Holy Faith English Grammar 100
2. Super Refresher English, Grade-9 150
3. Outlook Reasoning and Logic 100
4. ABC Vyakaran 200
5. Modern ABC Science, Grade-12 150
You are requested to supply the desired items within this week. As we are an
educational institution, we would expect a special discount that dealers generally give
to such institutions. You are requested to send the order through your own transport.
They should be neatly packed. Payment of the purchase will be done digitally once
the books are received.
Looking forward to hearing from you.

Thanking you
Yours truly
Manisha
The librarian

2. St Thomas School
New Delhi

1st March 20xx

The Sales Manager


Mahogony Sales, Karol Bagh
New Delhi 1100XX

Subject: Placing an order for electronic appliances

Dear sir
We are pleased to place an order for the items listed below. Kindly send these at the
above mentioned address as per the already negotiated prices.
Name of the Items No. of Items Brand
Fans 48″ 50 Bajaj
Microwave oven 25 LG
Bulbs 60 W 100 Philips
Tubelights 40 W 100 Nova
Kindly ensure that all the items are in good condition and packed properly. The
items must be delivered within 15 days of the issue of this letter. Any damage during
transportation will be borne by you. Kindly note that the cost of transportation will be

Learnwell
ACME Math–III
ENGLISH Practice Book GR–10 (Teacher Manual) 19

HF_Acme_ENG_TM_G10.indd 19 5/22/2021 5:58:38 PM


borne by you. Kindly find enclosed the cheque (No. 654219 dated 1 March, 20 xx)
for half the payment drawn on XYZ Bank, being the advance payment against the
above order. The remaining half will be paid on receiving the order.

Thanking you
Yours sincerely
Sandhya Verma

3. K 43, Shankar Road


New Delhi

21st December 20xx

M/S Prakash Furniture


Kirti Nagar
New Delhi

Sub: Placing an order for school furniture

Dear sir
I am writing this letter with reference to the quotation dated 19 December 20XX given
by your sales agent. We agree to it and hope that 10% discount that is permissible
to educational institutes will be given on our order. Please send the following items
on the above mentioned address.
Name of the Items No. of Items
Round Tables 10
Chairs 100
Benches 100
Conference tables 10
Kindly ensure that all the items should be in good condition and packed properly.
We expect them to be delivered within a week. The payment will be made by cash
once the items reach us. Any damage during transportation will be your responsibility.
Further please note that transportation chargers shall be borne by you.

Thanking you
Yours truly
Nitin Sharma

20 ACME English Practice Book GR–10 (Teacher Manual)

HF_Acme_ENG_TM_G10.indd 20 5/22/2021 5:58:38 PM


Letter of Inquiry
1. ABC School
Bengaluru

1st December 20xx

The Manager
Maharashtra Tour and Travels
Bengaluru

Sub: Seeking inquiry for an excursion tour to Ajanta and Ellora Caves

Dear sir
Our school is planning to organise a four-day tour to the Ajanta and Ellora Caves
during the school’s winter vacation. We are a group of two hundred and fifty students,
seven teachers and four peons. The planned dates for the tour are 26-29 December.
I want to enquire the charges for a comfortable stay of students and teachers. We
expect an affordable and hygienic accommodation in a good hotel. I want you to
furnish the following details regarding the charges and services provided by you.
1. Itinerary
2. Hotel details
3. Arrangement of breakfast, lunch and dinner and the charge per head
4. Arrangement of deluxe buses and their charges
5. Any other relevant information regarding the tour
Kindly provide us the details at the earliest. We also expect some discount that is
normally given to such tours consisting of a large number of tourists.

Thanking you
Yours truly
Maya
Teacher Incharge

2. 564, Model Town


Delhi

23rd January 20xx

The Sales Head


Jain Kitchen Equipment
Peeragarhi
New Delhi

Sub: Seeking inquiry for cabinets and equipment for kitchen

Learnwell
ACME Math–III
ENGLISH Practice Book GR–10 (Teacher Manual) 21

HF_Acme_ENG_TM_G10.indd 21 5/22/2021 5:58:38 PM


Dear sir
With reference to your advertisement no. 34 dated 20th January in The Times of India,
we want to order some kitchen cabinets and equipment for our newly constructed
home kitchen. Kindly inform us whether you have the availability and means to
transport us the items listed below. If you can, please send your catalogue or rate
lists with applicable terms and conditions.
1. Orpat Electric Blender 2. Inalsa Toaster 3. Divine Geyser
4. Kaff chimney 5. Cabinets (colours available)
As soon as we receive the information, we will place an order with you for the above
mentioned items. Kindly inform us of the available discounts, if any.

Thanking you
Yours truly
Rajni

3. A-51, Sector-21
Noida

23rd April 20xx

The Course Director


A TO Z in English
Gandhi Nagar
New Delhi

Sub: Seeking inquiry about TOEFL training programme

Dear sir
I am writing to you in reference to your advertisement in The Tribune dated 15th
April 20xx about a training programme for TOEFL—Test of English as a Foreign
Language being provided at your institute. I have just cleared my Class XII Board
Examination with an aggregate of 84%. I wish to go to England for higher studies.
Kindly send me the following details about the course:
• Duration of the course and batch availability
• Course content
• Fees for the course and the details of payment
• Boarding/Lodging facility, if available
Kindly mail prospectus, if possible. Looking forward to an early response.

Thanking you
Yours faithfully
Tanisha

22 ACME English Practice Book GR–10 (Teacher Manual)

HF_Acme_ENG_TM_G10.indd 22 5/22/2021 5:58:38 PM


Additional Letters
1. 674, Model Town
Delhi

16th February 20xx

The Editor
The Times of India
ITO
New Delhi

Sub: To raise concern over the outdated education system

Sir
Through the columns of your esteemed newspaper, I would like to show my concern
over the outdated education system which has not helped in creating more jobs in
India. Our education system still suffers from colonial influence and is not suitable
for the needs and demands of the contemporary times. Knowledge is confined to
reading, cramming and clearing the examinations. The syllabus focuses on teaching
what is given in the textbooks which make the education system theoretical. This
kind of education is not at all job oriented. Vocational training is the need of the hour.
The education system should be revamped to provide practical education apart from
academic knowledge. It is important to inculcate moral and value based education in
schools and colleges to deal with the challenges being faced by our modern society.
The evaluation system needs to be designed to recognise students’ creativity, problem-
solving, innovative and leadership skills. The education system in India needs to be
remolded to produce world-class leaders rather than producing cheap labor for the
developed nations.

Thanking you
Yours truly
ABC

2. 45-D, West Patel Nagar


Delhi

30th April 20xx

The Editor
The Times of India
ITO
New Delhi

Sub: Seeking complete ban on the use of tobacco products

Sir
Through the columns of your esteemed newspaper, I would like to draw the attention
of concerned authorities towards the rising consumption of tobacco and its products in
Learnwell
ACME Math–III
ENGLISH Practice Book GR–10 (Teacher Manual) 23

HF_Acme_ENG_TM_G10.indd 23 5/22/2021 5:58:38 PM


India despite the ban on tobacco-related advertisements and the law against smoking
in public places. I am of the opinion that only putting a ban on such advertisements
will not serve the purpose. The government should come out with a complete ban on
the use of tobacco products and the closure of tobacco industries. Only such drastic
measures will eliminate this evil. Smoking in public is still quite common as people
don’t bother to follow the laws due to ineffective laws and lenient implementation.
Exemplary punishment should be given to those who break laws regarding smoking.
Consumption of tobacco and its products has resulted in alarming rise in the cases
of oral and lung cancer. It’s high time the government take some drastic measures
to save people. The prices of tobacco products should be increased and the tobacco
producing industry should be made to pay high taxes. Awareness campaigns, online
and otherwise, should also be organised on large scale to make people aware of the
negative effects to tobacco consumption. I hope my words will find place in your
newspaper.

Thanking you
Yours truly
Kamal

3. 65-E, Nirman Vihar


Delhi

24th June 20xx

The Editor
The Tribune
New Delhi

Sub: Raising concern at the rapidly mushrooming illegal colonies and slums in metros

Sir
Through the columns of your esteemed newspaper, I would like to express my concern
over the rapidly mushrooming illegal colonies and slums in the metros and want to
suggest some remedial measures to stop them. Migrants from other states come to
metropolitan cities in search of work and start living illegally in slums due to lack of
affordable housing options. These slums have no provision for potable water, drainage
system and electricity which leads to the problems of water logging, insanitary living
conditions, spread of diseases, etc. Due to unavailability of garbage disposal systems,
these slums are breeding grounds for mosquitoes and other deadly diseases. These
diseases then spread all over the city. Authorities must make arrangements for people
living here and should provide basic amenities to the people living in slums. The
authorities should also maintain proper record of these people so that benefits of
government schemes might reach to them.

Thanking you
Yours truly
ABC

24 ACME English Practice Book GR–10 (Teacher Manual)

HF_Acme_ENG_TM_G10.indd 24 5/22/2021 5:58:38 PM


4. 90-B, Sector 24
Gurugram

23rd March 20xx

The Editor
Hindustan Times
Sector-63, Gurugram

Sub: Expressing concern about the problems created by stray animals

Sir
Through the columns of your esteemed newspaper, I would like to highlight the
nuisance created by stray animals on the roads.
A large number of stray cattle keep roaming on the busy roads at all hours of the
day. Many a time, they have been the cause behind traffic jams as well as accidents.
They create an unsafe environment for children and elderly people as they fear
being attacked by the stray cattle. Children feel afraid to go out to play. Elderly can’t
enjoy walking in the nearby parks. Sometimes fight amongst bulls lead to destruction
of public property. They also dirty the roads and park. This gives a very ugly look
to the whole area. We had complained to the municipal authorities on several
occasions, but no action was taken to deal with this menace. I request the concerned
authorities through the medium of your paper to take immediate action to deal with
this problem.

Thanking you
Yours sincerely
ABC

5. A.V. Road
Bengaluru

10th January 20xx

The Editor
Deccan Herald
Bengaluru

Sub: Raising demand for night shelters for homeless people

Sir
Through the column of your prestigious national daily, I want to highlight the need
for construction of more night shelters for homeless people. India has the maximum
number of homeless poor people in the world. Millions of people in India are forced

Learnwell
ACME Math–III
ENGLISH Practice Book GR–10 (Teacher Manual) 25

HF_Acme_ENG_TM_G10.indd 25 5/22/2021 5:58:38 PM


to spend their nights in the open due to lack of facilities like night shelter. Many
underprivileged people have to struggle for a little space on the pavements. The
situation is even worse during winters. One can find hundreds of people taking shelter
under bridges, at railway stations, bus stops or in under-construction buildings. There
have been numerous instances of theft, robbery, murder, kidnapping and molestation
due to unavailability of shelters at night. I am sure the number of these cases is much
higher as most of them go unreported.
The administration must look into the plight of these homeless people. The state
government must open adequate night shelters and provide free lodging so no one
is forced to sleep on pavements at night. It will help a great deal in saving hundreds
of homeless people from dying due to cold and crime.

Thanking you
Yours truly
Karuna

6. Sector 45
Chandigarh

10th March 20xx

The Editor
The Tribune
Chandigarh

Sub: Complaint against noise pollution caused due to religious celebrations

Sir
Through the column of your prestigious national daily, I want to highlight the
problem of noise pollution caused by religious celebrations. There’s an urgent need
of imposing blanket ban on the use of loudspeakers during religious celebrations. Use
of loudspeakers at religious processions, festival celebrations, holy songs blasting at
full volume during early mornings and late evenings have become the cause of great
nuisance to the residents of our area. Patients, elderly people, babies and students
are exposed to a great deal of suffering due to the loudspeakers blaring music at odd
hours.
Excessive noise adversely affects both physical as well as mental health. It is proven
that continuous exposure to loud noise causes a number of health ailments like
hypertension, stress, loss of hearing, disturbed sleep, etc. It is also disturbing for
students, especially those preparing for exams. I hope the concerned authorities will
understand the gravity of the problem and take strict steps to deal with this menace.

Thanking you
Yours faithfully
Karan Singh

26 ACME English Practice Book GR–10 (Teacher Manual)

HF_Acme_ENG_TM_G10.indd 26 5/22/2021 5:58:38 PM


7. 234, E Pocket
Dilshad Garden, Delhi

3rd January 20xx

The Editor
Hindustan Times
New Delhi

Sub: Raising concern over the problem of child abuse

Sir
Through the column of your prestigious newspaper, I want to highlight the problem
of child abuse. Abusing innocent children is one of the gravest problems prevailing in
the world. Children being small, vulnerable and afraid to speak are often physically,
mentally, emotionally and sexually abused. We find a lot of stories on TV, radio or
in newspaper about children being abused. Rise in nuclear families, easy access to
violent and adult content, lack of value education in schools, lack of strict laws and
their implementation, lack of social support, fear of stigma, lethargic dispensation
of justice, etc. are some of the factors that contribute to rising cases of child abuse.
Abuse of any kind leaves deep psychological scars on a child’s mind. Children are
the future of the nation. Child abuse not only endangers the health and welfare of the
abused children but also adversely affects the nation’s growth. Parents must educate
children about sexual advances or threats and protect them from abuse through
the concept of “good and bad touch”. People should donate to NGOs that provide
dedicated services fighting child sex abuse, and providing rehabilitation to victims of
abuse. People should report to police any kind of abuse they witness. General public
should help the government agencies to fight against child abuse.

Thanking you
Yours sincerely
Ankit Arora

8. Sector 71
Gurugram

3rd March 20xx

The Editor
Indian Express
New Delhi

Sub: Raising concern about the living conditions in slums

Sir
Though the column of your prestigious national daily, I express my dismay at the fact
that many young children from slums live and grow up in unhygienic surroundings.

Learnwell
ACME Math–III
ENGLISH Practice Book GR–10 (Teacher Manual) 27

HF_Acme_ENG_TM_G10.indd 27 5/22/2021 5:58:38 PM


They are under-nourished and illiterate. Millions of slum children are living a miserable
life in slum areas that lack even the basic civic necessities of life. They don’t have
access to civic amenities. Pure drinking water is not available to them. There is no
provision for the sewerage or availability of drainage system. Water-logging is very
common in slums leading to spread of a number of diseases. These children don’t
have access to schools. Even when schools are available, they lack the basic facilities
for children. Most of the times, parents of slum children do not send them to school as
they expect them to work. Life in a slum is gruelling. Quick and efficient steps need
to be taken by the government for improving education, hygiene and civic amenities
in slum areas. The government and social agencies should work together to provide
better facilities to slum children.

Thanking you
Yours sincerely
Trisha

9. F-14
Dayal Bagh Colony
Agra

2nd March 20xx

The MLA
North Agra

Sub: Complaint regarding the poor water supply

Dear Sir
I would like to bring to your kind attention that the water supply in our locality is very
poor. There are around 20000 people living in this area. We are facing problems of
poor quality and insufficient quantity of water supply for the last six months. Water
is supplied for only fixed two hours per day. It is very difficult to fill water in such less
time for the entire day, especially for those who have bigger families. It has also led
to almost daily clashes among people. With the approaching summers it would be
impossible for us to survive. Moreover since the last few days the quality of water
has deteriorated. People are forced to buy bottled water at exorbitant prices. Those
who cannot afford to buy water are forced to drink the muddy water. If something is
not done soon the health of the residents would be at risk. I request you to consider
our demands as high priority as our life is at stake. We had complained many times
to the local authorities but no remedial action has been taken. I, therefore, request
you to look into this matter and take necessary action immediately.

Thanking you
Yours sincerely
Gopal Pandey

28 ACME English Practice Book GR–10 (Teacher Manual)

HF_Acme_ENG_TM_G10.indd 28 5/22/2021 5:58:38 PM


10. 44, Bansi Garden
Jodhpur

10th March 20xx

The Manager
Vijay Electronics
Sikandri Bazaar
Jodhpur

Subject: Complaint about defective TV

Sir
I am writing in reference to the purchase of a Samsung LED television from your
outlet against the Cash Memo no. 123 dated Feb 2, 20xx. It worked well for about a
month. But for last few days, it has started to have problems. We have noticed issues
with sound and picture quality. The screen gets blurred every now and then. The
sound is not clear even at a high volume. The product has a two-year warranty against
any technical defect. Claiming that warranty, I request you to send your engineer to
check the fault and rectify the problem. If any manufacturing defect is discovered by
the engineer, I would like you to replace the set with a new one at the earliest.

Yours sincerely
Saumil

11. 115, Preet Vihar


New Delhi

10th March 20xx

The Manager
Chroma
Daryaganj

Subject: Complaint regarding defective washing machine

Dear sir
A month ago I purchased a fully automatic Whirlpool washing machine from your
outlet vide Cash Memo No. 0702 dated February 1, 20xx. For about a month, the
machine worked satisfactory. But for the last one week, it has started developing
issues. It makes a very loud noise on the spinning mode. Sometimes it stops on
its own all of a sudden. The machine does not wash the clothes properly. Once it
gave a minor electric shock to my sister while it was in use. Since this machine falls
well within the warranty period, I request you to replace it at the earliest. Your store
is a reputed store and I have been your regular customer. I have purchased many
appliances from your store before and have not faced any issues with any of them.
I hope you send an immediate replacement of the washing machine.

Yours sincerely
Kapil Sharma
Learnwell
ACME Math–III
ENGLISH Practice Book GR–10 (Teacher Manual) 29

HF_Acme_ENG_TM_G10.indd 29 5/22/2021 5:58:38 PM


12. 45-B
Nirman Vihar
Delhi

7th February 20xx

The Manager
Sheetal Electronics
Nirmal Vihar, Delhi

Sub: Complaint regarding a defective hair dryer

Sir
A week ago I purchased a ‘Philips Hair Dryer’ from your outlet vide Cash Memo No.
0702 dated February 1, 20xx. The demo piece had worked satisfactory when I tried it
at your store but when I used the hair dryer at home after three days of the purchase,
it stopped working on its own all of a sudden. I have read the manual and tried to
troubleshoot it as per given instructions but my efforts were in vain. Since this hair
dryer is well within the warranty period, I request you to replace it at the earliest. I
have been your regular customer and have purchased many articles from your store
on your recommendation and good faith. So, I hope for an immediate replacement
of the hair dryer.

Yours sincerely
Anisha

13. Room Number 25


Subhash Hostel

17th February 20xx

The Principal
Saint Luke School
Saket, Delhi

Subject: To raise complaint against bullying

Dear Sir
I am a student of class X B, staying in Subhash Hostel, room number-20 for the last
6 months. I have been facing a lot of bullying because of being a new student in the
hostel. I am teased on regular basis by a group of students. I come from a rural area
and look and speak differently from the other students. These bullies make snide
remarks every time I come across them. They have heckled me and at times have

30 ACME English Practice Book GR–10 (Teacher Manual)

HF_Acme_ENG_TM_G10.indd 30 5/22/2021 5:58:39 PM


snatched my lunch and money from me. As a result, I am undergoing a lot of stress.
I am not able to concentrate on my studies. I have even been threatened of serious
consequences by these bullies if I complain. I request you to take immediate action
against these bullies so that I can stay here peacefully.

Thanking you
Yours faithfully
Madhur
10 B

14. Sector 6
Birla Apartments
Rohini, New Delhi

7th April 20xx

The Manager
Arora Stationery Mart
Connaught Place
New Delhi

Sub: Placing an order for stationery item

Sir
We want to place an order for some stationery items needed for our coaching institute.
Kindly provide us the below mentioned stationery as soon as possible.
• Ball points – 50 packets
• Gel pens – 30 packets
• Printer papers A 4 size – 50 bundles
• Glue sticks – 10
• File covers – 50
• Scotch tape – 10
We have been your regular customer for the past few years. So, kindly process our
request on urgent basis. We expect a regular discount of ten per cent. Delivery charges
will be borne at your end. Payment will be done through online transaction once the
products are received.

Thanking you
Yours sincerely
Anupama Chhabra

Learnwell
ACME Math–III
ENGLISH Practice Book GR–10 (Teacher Manual) 31

HF_Acme_ENG_TM_G10.indd 31 5/22/2021 5:58:39 PM


15. Pragati Public School
Green Park, New Delhi

27th February 20xx

The Manager
Bajaj and Sons
Sadar Bazaar, New Delhi

Subject: Order for sports items and accessories

Sir
I would like to place an order for the following sports items and accessories, on the
behalf of Pragati Public School, Green Park.
Volleyball nets 2 pieces
Cricket bats 4 pieces
Tennis racquets 4 pieces
Tennis balls 2 pieces
Footballs 6 pieces
Leg-guards 12 pieces
Gloves 6 pairs
Kindly make sure that the items are of high quality. A reasonable discount which
you give to educational institutions is expected from you. We want the delivery to
be made within 15 days of the placement of order. The payment would be made
through bank draft within two days after the products are received.

Yours sincerely
Preeti Lakra
Sports Incharge

16. 365, F pocket


Shahdara, Delhi

6th March 20xx

The Manager
Sargam Electronics
M-63, Dilshad Garden
New Delhi

Sub: Placing an order for air conditioners

32 ACME English Practice Book GR–10 (Teacher Manual)

HF_Acme_ENG_TM_G10.indd 32 5/22/2021 5:58:39 PM


Sir
We want to place an order for five air conditioners for our newly opened office. The
brand along with the capacity are mentioned below:
Brand Capacity
1. Lyyod (Split AC) 1.5 ton
2. General (Window AC) 2 ton
3. Samsung (Window AC) 2 ton
4. Haier (Split AC) 2 ton
5. LG (Window AC) 1 ton
All the ACs should be delivered in proper condition, well bound and packed properly.
The items must be delivered by end of the week of the issue of this letter. The cost of
transportation would be borne by you. Any damage during transportation would also
be borne by you. The payment will be made within 24 hours of receipt of delivery
through online transaction.

Thanking you
Yours truly
ABC

17. Green Fields Public School


GTB Enclave

9th July 20xx

The Manager
Mehta Science Equipment
D-78, Jagatpuri

Sub: Placing an order for microscopes

Sir
As you are a trusted name in the supply of scientific goods for educational institutions,
we want to place an order for 20 microscopes for our school’s laboratory. We expect
competitive rates for the products. We expect the general 10% discount that you
provide on the purchase of scientific goods. We also hope that you will provide an
extra 5% discount usually given to educational institutions. All the microscopes must
reach us in proper condition. The cost for transportation as well as any damages during
it will be borne by you. The payment will be done through cheque after receiving the
order. Please send the products within 7 days of receiving this letter.

Yours truly
ABC

Learnwell
ACME Math–III
ENGLISH Practice Book GR–10 (Teacher Manual) 33

HF_Acme_ENG_TM_G10.indd 33 5/22/2021 5:58:39 PM


18. 879, B Block
Pragati Vihar

25th February 20xx

The Manager
Khanna Bicycles
B-110, Gaffar Nagar
Delhi

Sub: Placing an order for bicycles

Sir
I am writing in reference to our telephonic conversation on 22 February 20xx regarding
the purchase of bicycles for our newly opened shop. We want you to supply us with
10 bicycles. Details of the order are given below:
Name Type Quantity
Atlas Sports Cycle 3
Hero Ranger Gearless 4
HMT Fat bike 3
We expect a usual discount plus special discount on the ground that we are new in
business. We hope to share a long term business relations with you, if the deal is
found satisfactory. I request you to send us the above mentioned items with bill within
15 days of placement of the order. Payment will be done through cheque once the
order is received. Please note that the cost of transportation will be borne by you.

Yours faithfully
ABC

19. St Mary’s School


Chandigarh

10th January 20xx

The General Manager


Paradise Hotels and Resorts
Chandigarh

Sub: Inquiry about charges for a farewell party

Sir
As the coordinator of the organising committee for conducting a farewell party for
class 12th, I want to make some enquiries regarding the cost of the party. There

34 ACME English Practice Book GR–10 (Teacher Manual)

HF_Acme_ENG_TM_G10.indd 34 5/22/2021 5:58:39 PM


will be approximately 250 students and 50 teachers. We want a buffet system for
lunch that must include both vegetarian and non-vegetarian options. Please send
the details about the menu and sitting arrangement. Also, I want to know about the
food and beverage charges per head, taxes applicable and service charges. We expect
the discount given to educational institutes for such events. Kindly provide timely
information as the party is scheduled to happen on 28th February 20xx.

Yours truly
Nikhil
Coordinator

20. Mayur Vihar


New Delhi

10th March 20xx

The Director
Web Training Academy
Gurugram

Sub: Inquiry about web designing course

Sir
I came across your advertisement in The Times of India dated 3rd March 20xx about
a one-month web-designing course being offered in your institution. I would like to
know the details of the course offered by your institute. I am a class X student and
wish to pursue a course in web designing during the summer vacation after my board
exams. I want to know the following details:
• Syllabus
• Fee structure
• Eligibility criteria
• Timings
Kindly send me the brochure of the institute at the above mentioned address. The
payment for the brochure shall be made at the time of the delivery.
Thanking you in anticipation

Yours faithfully
Shweta Rajput

Learnwell
ACME Math–III
ENGLISH Practice Book GR–10 (Teacher Manual) 35

HF_Acme_ENG_TM_G10.indd 35 5/22/2021 5:58:39 PM


21. 345, Sector 51
Meerut

10th March 20xx

The Manager
Ayur Herbal Solutions
Ghaziabad

Sub: Inquiry regarding course in ayurvedic medicine

Sir
I am writing this letter in reference to your advertisement in Meerut Chronicles, dated
5th March regarding courses in ayurvedic medicine offered by your reputable institute.
I am currently in class XII and preparing for my final exam. I am very interested in
ayurveda and want to pursue a career in this field. I am interested in pursuing a course
in ayurvedic medicine from your institution. Kindly send me the prospectus and the
application form. I would like to be provided information for the following points:
• Course details
• Fee structure
• Scholarships availability
• Admission procedure
• Eligibility criteria
• Hostel facilities
Kindly send me the brochure of our institution also so that I could know more about
your institute.

Yours sincerely
Arjun Sinha

22. 45-A, Sector-21


Dwarka, Delhi

21st November 20xx

The Manager
Lost and Found Department
Delhi Metro Service
New Delhi

Sub: Loss of baggage on train no. 565

Sir
I travelled on 20th November by Delhi Metro. I boarded the train from Karol Bagh for
Noida Sector 15 at 8 pm. It was a night journey. The train reached the destination at

36 ACME English Practice Book GR–10 (Teacher Manual)

HF_Acme_ENG_TM_G10.indd 36 5/22/2021 5:58:39 PM


8:30 pm. After I got down at the station from the train, I was surprised to find that one
of my bags was missing. It was a regular sized American Tourister red and black duffle
bag. It contained some important documents besides my clothes and accessories. I
may have left it at the entry station Karol Bagh during the check-in. I want to enquire
if your department has any such bag in your possession. If yes, kindly contact me on
the above mentioned address. I will be indebted for your help.

Thanking you
Yours faithfully
ABC

23. House Number 56


Sector-16, Noida

13th September 20xx

The Sales Manager


Octopus Books
Daryaganj
Delhi

Subject: Enquiry for books by Ruskin Bond

Sir,
I want to inquire about the availability of some books by Ruskin Bond. I wish to
purchase the below mentioned sets of these books for a newly opened public library
in our society.
Name of books Quantity
1. The Room on the Roof 5 copies
2. The Blue Umbrella 4 copies
3. The Night Train at Deoli and Other Stories 3 copies
4. Delhi Is Not Far 7 copies
5. Rusty, The Boy From The Hills 5 copies
6. Time Stops at Shamli and Other Stories 5 copies
7. Our Trees Still Grow In Dehra 6 copies
8. Roads To Mussoorie 6 copies
Please also send the details about the prices, discount, mode of payment and mode of
delivery at the above mentioned address. Prompt response will be much appreciated.

Thanking you
Yours truly
Abc

Learnwell
ACME Math–III
ENGLISH Practice Book GR–10 (Teacher Manual) 37

HF_Acme_ENG_TM_G10.indd 37 5/22/2021 5:58:39 PM


Analytical
2… … … .
Chapter Paragraphs

ANSWER KEY

1. The map describes the growth of a fishing town from 1995 to present day. One can
see significant changes have taken place in the fishing town’s infrastructure between
the time periods.
It can be observed that the farmland and the forest park located in the northeast of
the fishing town have been cleared and converted into a golf course and a tennis
court. The housing area situated on both the sides of the main road in the north-west
has not undergone any major changes. A new road connecting to the main road has
been added from the west direction to provide connectivity to the newly constructed
houses on that side.
Earlier there were shops and a fish market on the south western side of the town
near the fishing port. These shops and fish market have been replaced by a line of
restaurants and apartments. On the south eastern part of the town, a car park has
been built near the hotel. In 1995, a fishing port used to exist on the southern side
of the village but it has been removed now.
2. The graph shows the percentage of profits earned by two companies between the
years 2000-2005. It can be deducted clearly that Company A has shown more
steady growth than Company B, especially during the last years of analyses. During
2000, Company A was in loss. It incurred around 10% losses in this year. On the
other hand Company B registered a profit of 10%. Both the companies were on the
opposite ends of the spectrum in 2000. In 2001, Company A remained steady with
no profits and no loss while company B incurred a loss of 5%. An equal increase
of 5% in profits is observed in both companies in the year 2002. In the year 2003,
Company B registered a sharp decline of 10 % while Company A achieved a profit
of 5%. In 2004, Company B showed substantial improvement by registering 15%
profits while Company A registered a profit of 10% in the same year.
In conclusion, the graph shows a tough competition between the two companies with
Company A being a little ahead in terms of profit.
3. The table shows information about the number of people who visited various museums
and galleries in Britain in 1981, 1991 and 2001. The pie chart shows the distribution
of different kinds of visitors who visited five different museums and galleries in Britain
in three different years.
British Museum received the highest proportion of visitors while Tate Gallery received
the lowest numbers of visitors in the years the survey was conducted. National History
Museum received the highest number of visitors in 1991 among all the museums and
galleries. The number of visitors continued to drop in the next few years. In 1981,
British Museum was visited by the second highest number of people after the National
History Museum. It continued to show upward trend in the next two years. Science
Museum received around 3.4 million visitors in 1981. The number dropped to 2.3
millions in 1991 and 1.7 millions in 2001. National History Museum and Science
Museum are the two places which have shown drops in number of visitors. National
Gallery received 2.5 million visitors in 1981. The number increased to 3.7 in 1991 and
5.4 million in 2001. Among those who visited these museums, the highest percentage

38 ACME English Practice Book GR–10 (Teacher Manual)

HF_Acme_ENG_TM_G10.indd 38 5/22/2021 5:58:39 PM


belongs to students, followed by researchers and teachers (both form around 20 per
cent of total visitors), followed by ordinary people and tourist. Tourists are the lowest
contributors to the number of people who visit the museums and galleries in England.
4. It is a report by IPCC about the increase in global warming and its consequences.
According to the report the global warming limit is set at 1.5 degrees Celsius but due
to the current rate of emissions of pollutants, the world is about to breach this global
warming limit very soon. It is revealed by the IPCC Special Report that the set limit
of 1.5 degrees Celsius will be crossed between 2030 and 2052. This special report
was commissioned to specifically explore the scientific feasibility of the 1.5°C goal set
in the Paris Agreement. The report suggests that continual deterioration of extreme
weather events will result in temperature rise. According to the report it is mandatory
to achieve net-zero emissions by 2050 to limit global warming to 1.5°C. It further
states that emissions would need to be drastically cut by at least 45 per cent by 2030.
It is also included in the report that the corresponding rates of emission reduction
required to limit warming to 2°C would be a 20 per cent reduction by 2030 and net-
zero emissions by 2075.
5. The chart below shows the changes in students’ population studying under different
courses in a college between years 2005 and 2007. The chart illustrates the changes
in the choice of courses over a period of three years.
According to the graph, in the year 2005 the highest number of students opted for
Arts in comparison to other subjects. The number of students choosing Arts was more
than double the number of students choosing Law and Commerce. The number of
students who chose Science was approximately 200 less than the number of students
who chose Arts.
In the year 2006, the number of students studying Arts and Science were almost the
same with a little gap of only 10 students. The same gap is reflected between the
number of students studying Law and Commerce. In the year 2007, the situation
changed as the number of students studying Science exceeded the number of
students studying Arts by almost 150 numbers. The number of students studying Law
showed an increase of 100 students and became more than the number of students
in Commerce. The number of students studying Commerce remained the same in
2006 and 2007.
6. The chart given above provides information about the percentage of students choosing
various sports as a career. The highest number of students wished to take up football
as a career. The percentage of students who wish to opt for football is more than two
third of the total number of students. There is a huge difference between football
as a career and other sports like handball, basketball, volleyball and other sports.
The chart illustrates that 71% students consider football as a good option for career
whereas the rest of the students (29%) think other sports to be good for career. Nine
per cent students wish to play handball. Volleyball and basketball are the preferred
sports for the same percentage of students, that is, 6 per cent. Eight per cent of the
students have chosen other sports. Analysing this pie chart, we can conclude that
football is the most preferred game among students.
7. The chart describes the process of digestion. The first stage of the digestion begins
in the mouth. First of all, ingestion takes place. It refers to the entry of food through
the mouth. Next, the mechanical and chemical breakdown of food happens when
the food is chewed. Then the food moves to the oesophagus which facilitates the
movement of food from the mouth to the stomach.

Learnwell
ACME Math–III
ENGLISH Practice Book GR–10 (Teacher Manual) 39

HF_Acme_ENG_TM_G10.indd 39 5/22/2021 5:58:39 PM


The third stage of digestion happens in the stomach. The chemicals present in the
stomach break down the food. Mechanical digestion and chemical digestion takes
place in the stomach. The third stage of digestion happens in the small intestine where
chemical digestion of food takes place. The nutrients from the food are absorbed.
The remaining food enters the large intestine where absorption of water takes place.
The undigested part of the food becomes waste. The large intestine propels food
towards the rectum. The rectum and anus enables waste products to come out of
body through the process of egestion.
8. The line graph indicates the amounts of goods transported using four different types
of transportation– road, water, rail and pipeline in Britain between the year 1974
and 2002. Overall, it is clearly noticeable that the amounts of goods transported in
Britain by all the four different types of transportation increased over the given period.
Also, there was a significant rise in the quantities of goods transported by road in
Britain from 1974 to 2002. From about 70 million tonnes in 1974, the quantities
of goods being transported via road increased to 100 million tonnes in 2002. The
same type of ascending pattern can be observed in goods transported by water and
pipeline. Initially transport through water was about 40 million tonnes in 1970. By
2002 it has increased to 60 million tones after witnessing a drop in 1998. Transport
through pipelines was about 5 million tonnes and rose to around 20 million tonnes
by 2002.
Transportation by rail has seen significant ups and downs. It was 40 million tonnes
in 1974 and declined to about 20 million tonnes in 1987. Then it witnessed a lot of
ups and downs between 1985 and 1994. It rose considerably until the year 2002
and reached its peak at just over 40 million tonnes.
9. The bar graph gives information on participation by boys and girls aged 5-15 in
different leisure activities, namely skateboarding/rollerblading, bike riding, watching
TV or videos, electronic or computer games and art and craft.
Overall boys and girls of all age group show highest interest in watching TV or
videos. Almost all children interviewed in this survey enjoy watching TV or videos.
The next highest interest they showed is in electronic or computer games. Here boys
are more interested in computer games than girls. Bike riding has the third highest
level of interest amongst the interviewed participants. Around 70 per cent boys are
interested in bike riding as opposed to 55 per cent girls. More boys are interested in
skateboarding or rollerblading than girls. The figures stand at 35 per cent and 25 per
cent respectively.
Boys are least interest in Art and Craft. Only 30 per cent boys show interest in art
and craft while girls show least interest in skateboarding/rollerblading.
10. The pie chart shows the market share of different mobile companies in the total number
of mobile phones sold in India in the year 2018. As per the pie chart, Samsung stood
at the first position with a global market share of 22.7%. Huawei settled into the second
position with the market share of 14.6% of the smartphone market. Apple captured
the third position with a market share of 19.1%. Xiaomi took the fourth position with
9.7% of the market share. The least number of mobile phones were sold by Oppo
which captured 8.9% of the market share. Other mobile companies held a huge share
of 31% of the total smartphone market in 2018. Samsung remained to be the most
popular brand in the year 2018. So, it can be concluded that most of the market in
India has been captured by Chinese smartphone companies.

40 ACME English Practice Book GR–10 (Teacher Manual)

HF_Acme_ENG_TM_G10.indd 40 5/22/2021 5:58:39 PM


Additional Exercise
1. The pie chart shows the causes of worldwide degradation of land. As per the pie-chart,
the main causes for land degradation are over-grazing, deforestation, over-cultivation
of crops and some other miscellaneous causes. The over-grazing and deforestation
constitute 65% of degradation globally with contribution of 35% and 30% respectively.
The global degradation of land happened due to over-cultivation of crops is 28%.
Other causes account for 7% of the total land degradation.
The percentage share of these causes of land degradation differs in different regions.
The table shows that Europe is facing as much as 9.8% of degradation due to
deforestation, while the impact of the factor on Oceania and North America is minimal,
with only 1.7% and 0.2% of land degradation respectively. It also suffers from over-
cultivation (7.7%) and overgrazing (5.5%). Europe has the highest overall percentage
of land degradation (23%). Oceania suffers from 13% land degradation which is
caused majorly due to over-grazing (11.3%). North America has comparatively a
lower proportion of degraded land which is 5%. It is least affected by land degradation
among the three. The main contributors of land degradation in North America is
over-cultivation (3.3%) followed by over-grazing (1.5%) and deforestation (0.2%).
From the pie chart and the table, we can deduce that Europe has been affected more
from land degradation than the other regions.
2. The table shows the sales figures of various food items at a departmental store over
a period of some months. The table displays the sale amount in dollars for six items
namely grains, milk products, vegetables, fruits, soft drinks and meat products. It
is clear from the table that grains were the biggest selling food item during all the
months. Also, the sale of grains increased over the period of time from 5000 to 7000
between May to October. Milk products have also shown a huge increase of 1000
dollars in sales between May-June and July-August. The increase is meager between
July-August and September-October. Vegetables and soft drinks are the only items in
the table whose sale decreased in September-October. The sale of soft drinks is the
highest among all the items during July-August owing to the hot weather conditions.
Fruits have shown consistent increase in sales over the period of months. Sale of meat
products reduced by 1000 dollars during July-August but then increased sharply by
1500 dollars in September-October. Overall sale of grains, milk products and meat
products have shown an upward trend.
3. The line graph depicts figures of TVs, LCDs and LEDs sold in the United Kingdom
over a period of five years from 2006 to 2011. It can be clearly observed that most
people preferred LED over TV and LCD. A constant increase in sale of LED has
been observed from year 2010 onwards. More LCDs were sold than TV after 2008
till 2010 when their sale witnessed a steady decrease after LEDs were introduced in
the market. The sale of TV witnessed had a sharp fall from approximately 85 million
in 2010 to about 30 million in 2011. The sale of LCD also experienced a steep fall
of near about 50% in 2011 from its peak in 2010. The highest number of TVs was
sold in 2010 at 85 million sets. The highest number of LCDs was sold in 2010 at
100 million sets. The LEDs crossed the 100 million mark within two years of being
launched in the market. Hence, the conclusion can be derived that LEDs became an
instant crowd favourite after its launch in the market and TV and LEDs lost a major
share of the market in the same time.
4. The bar graph shows the birth rates of some countries such as Sweden, Germany,
England, New Zealand, India, and China. The birthrate is shown in numbers per

Learnwell
ACME Math–III
ENGLISH Practice Book GR–10 (Teacher Manual) 41

HF_Acme_ENG_TM_G10.indd 41 5/22/2021 5:58:39 PM


thousands. Overall, it is evident that the birthrates per thousand are lowest in Sweden,
Germany and England. Sweden (15) has the lowest birth rate per thousand followed
by Germany (16) and England (20). On the other hand China and India have the
highest number of birth rate per thousand, with China leading the race.
China has the highest birth rate at 40 per thousand. India has the second highest
birth rate per thousand at 33. According to the bar graph, the third highest populated
region is New Zealand with a birth rate of 30 per thousand. It has double number per
thousand than Sweden’s birth rate. New Zealand and India share almost the same
birthrate per thousand, differing only by 3 per thousand. China has double birthrate
per thousand than England and triple than that of Sweden and Germany.
5. The two graphs show the number of Japanese tourists who visited different countries
between 1985 and 1995 and Australia’s share in the total number of Japanese
tourists who travelled abroad. It is clear from the observation of the first graph that
the number of tourists who travelled abroad has increased steadily over the years.
From the second line graph, it is clear that the percentage of tourists visiting Australia
increased significantly throughout the decade.
Since 1985 the number of Japanese tourists who travelled abroad tripled from
5 millions to 15 millions in 1995. However, the movement of tourists showed a slight
fall by approximately 2 million visitors in the year 1991. But since then the number
of people going abroad has increased. The number of people travelling to Australia
has also shown an upward trend since the beginning, registering a decline in the
year 1989. The travel percentage picked up in 1990 and since then had consistently
increased till 1993 when it suffered another decline.

Section-B (B2: Grammar)

Chapter 1 Tenses

ANSWER KEY

Exercise 1
1. reaches 2. will return 3. will join
4. argues 5. rain 6. weeping
7. are, spending 8. has travelled 9. got
10. do not believe/believe

Exercise 2
1. have, promised 2. will fly 3. forgets
4. were planning 5. was writing 6. had left
7. had looted 8. will, go/did, go 9. will join
10. are preparing

42 ACME English Practice Book GR–10 (Teacher Manual)

HF_Acme_ENG_TM_G10.indd 42 5/28/2021 3:41:06 PM


Exercise 3
1. Are you not living in California?
2. Does his friend come first in every essay-writing competition?
3. Has Deepak not been reading the novel for half an hour?
4. We were not playing basketball when she came.
5. Is she not selling flowers at her flower shop?
6. Did I go to see a movie in cinema yesterday?
7. They have planned to celebrate their wedding anniversary this year.
8. The police have not been searching for the thief for two days.
9. You were even paying attention to the lecture in the class yesterday.
10. His sister was chiding you yesterday.

Integrated Exercise
Exercise – 1
1. (a) (ii) was born (b) (i) were placed (c) (iv) got
(d) (i) began (e) (iii) became (f) (ii) took
2. (a) (iii) had died (b) (ii) came (c) (i) begged
(d) (iv) told (e) (i) added (f) (ii) had not visited
3. (a) will be held (b) will be (c) will put
(d) will guide (e) shall hold (f) will be

Exercise – 2
(a) falling (iv) fell (b) bleed (ii) bled (c) rushing (iv) rushed
(d) struggle (iii) struggled (e) breathe (iv) breathed (f) arriving (ii) arrived

Exercise – 3
(a) (ii) happened (b) (i) was standing
(c) (iii) was being kidnapped (d) (iii) was crying
(e) (ii) shouting (f) (iii) was wearing

Exercise – 4
(a) (ii) Galileo had done (b) (i) Aristotle had stated
(c) (iii) object would fall (d) (iii) It has been
(e) (i) Galileo was the (f) (iii) who proved that

Exercise – 5
(a) It was the (b) films were screened (c) it gave the
(d) festival had a

Learnwell
ACME Math–III
ENGLISH Practice Book GR–10 (Teacher Manual) 43

HF_Acme_ENG_TM_G10.indd 43 5/28/2021 3:43:22 PM


Chapter 2 Modals

ANSWER KEY

Exercise 1
1. would 2. will 3. should 4. May 5. can
6. shall 7. used to 8. May 9. should 10. need

Exercise 2
1. May I come in, sir?
2. Mayank asked his brother when he would go to the market.
3. He could run fast when he was young.
4. I told my sister that I might come to her annual function if I got free on time.
5. We worked hard so that we might get the first position in the examination.
6. I would rather take rest at home than go to the cinema.
7. Invite him to the party lest he should be angry.
8. What will you do after your higher secondary examination?
9. Would that I were a king!
10. How dare you enter my home without my permission?

Integrated Exercise
Exercise 1
1. (a) (i) should (b) (iii) can (c) (iv) will
(d) (ii) will (e) (iii) would (f) (iv) need
2. (a) (ii) must (b) (iii) should (c) (ii) can
(d) (i) must (e) (iii) will (f) (i) must

Exercise 2
1. (a) (i) can (b) (ii) should (c) (iii) can
(d) (iv) may (e) (ii) will (f) (iii) may
2. (a) (i) can (b) (ii) will (c) (i) would
(d) (i) would (e) (ii) should (f) (i) would

Exercise 3
1. (a) may (b) might (c) may
(d) must (e) will (f) should
2. (a) may (b) can (c) need
(d) will (e) can (f) needn’t

44 ACME English Practice Book GR–10 (Teacher Manual)

HF_Acme_ENG_TM_G10.indd 44 5/22/2021 5:58:40 PM


Chapter 3 Use of Passive Voice

ANSWER KEY

Exercise 1
1. Why was the book torn by you?
2. Has his education been completed by Ankit?
3. By whom was your car being washed?
4. The Geeta is read by him daily.
5. The station will have been left by the train by now.
6. You are requested not to make noise.
7. All patients have been examined by the doctor.
8. The ceremony due tomorrow will not be attended by them.
9. You are advised to not break the traffic rules.
10. The car was being driven by him.

Exercise 2
1. She does not play badminton any longer.
2. The police will have nabbed the terrorists.
3. The chartered accountant must check all his accounts.
4. Both males and females in the world wear jeans nowadays.
5. Diwali is being celebrated by the people by lighting candles and burning crackers.
6. The NRI community here celebrates Dussehra with much enthusiasm.
7. What has been eaten by you today?
8. Have they not been invited by him to the coronation ceremony?
9. An operation for kidney will be undergone by his father tomorrow morning.
10. The people in this area speak English.

Exercise 3
1. Some masked men have kidnapped his son from the old market.
2. Why did the police arrest them?
3. She will withdraw money from your account.
4. Are the students singing the national song?
5. Manish has paid all the installments to the bank.
6. Rajesh sent this message for you.
7. Where did you steal this purse?
8. Can I be taught how to fix a leaking roof by you?
9. How many questions have been attempted by you in today’s exam?
10. Why are you not invited by him to his home?
11. You are always praised by me in front of other people.
12. Why am I being criticised by you now?
Learnwell
ACME Math–III
ENGLISH Practice Book GR–10 (Teacher Manual) 45

HF_Acme_ENG_TM_G10.indd 45 5/22/2021 5:58:40 PM


Integrated Exercise
Exercise–1
1. (a) (iii) are held (b) (iii) are made (c) (iv) are set
(d) (ii) are appointed (e) (iii) are deployed (f) (iv) is taken
2. (a) (iv) has been used (b) (iii) is used (c) (i) are prepared
(d) (i) are made (e) (ii) is cooked (f) (ii) is considered
3. (a) (iii) is held (b) (iv) was celebrated (c) (i) was decorated
(d) (i) was organised (e) (iv) was inaugurated (f) (iii) was presented

Exercise–2
1. (a) (ii) is taken (b) (ii) is dug (c) (i) is taken out
(d) (i) is mixed (e) (ii) are added (f) (i) are placed
2. (a) (i) are separated (b) (i) are dissolved (c) (i) are allowed
(d) (ii) are rubbed (e) (ii) are rinsed (f) (ii) is washed off
3. (a) (i) are taken (b) (ii) is added (c) (i) are mixed
(d) (iii) is heated (e) (i) is poured

Chapter 4 Subject-Verb Concord

ANSWER KEY

Exercise 1
1. does 2. has 3. likes 4. requires 5. works
6. was 7. have 8. are 9. guide 10. goes
11. have 12. have

Exercise 2
1.
Rakesh does not get up early in the morning.
2.
It is he who helps me every time.
3.
The people who live in glass houses should not throw stones on others.
4.
Anand Kumar, as well as his students, has come for the ceremony.
5.
The principal and the teachers have participated in the annual function of the college.
6.
One of my relatives lives in England.
7.
A lot of people do not know about the laws regarding the functioning of the
government.
8. Brown bread is a good source of carbohydrates to include in breakfast.
9. Measles is spreading in many parts of the country at an alarming rate.
10. Your spectacles look very good on your face.

46 ACME English Practice Book GR–10 (Teacher Manual)

HF_Acme_ENG_TM_G10.indd 46 5/22/2021 5:58:40 PM


Integrated Exercises
Exercise 1
1. (a) (i) are (b) (ii) is (c) (i) are
(d) (ii) has (e) (i) preserve (f) (ii) helps
2. (a) (i) is (b) (ii) has (c) (i) has been
(d) (ii) is (e) (ii) are (f) (ii) is

Exercise 2
1. (a) (i) has (b) (iii) is (c) (i) is caused
(d) (iv) benefit (e) (iii) want (f) (iv) are considered
2. (a) (i) was waiting (b) (iii) was wearing (c) (iii) looked
(d) (ii) saw (e) (iv) was saying (f) (i) was making

Exercise 3
1. (a) originate (b) emanates (c) is
(d) is (e) is (f) changes
2. (a) was (b) were (c) was
(d) were located (e) was (f) were exchanged

Chapter 5 Reported Speech

ANSWER KEY

Exercise 1
1. His father told him that he had to go to Delhi then.
2. She told me that if I gave her a wrist watch, she would give me a digital camera.
3. Seema told her that she had been studying for two hours.
4. They asked me if they did not teach me well enough.
5. They asked me if he had ever been convicted of stealing.
6. I replied to his inquiry in negative.
7. Ravi asked her where her brother was living those days.
8. We asked each other if the Indian Cricket Team would win the match that day.
9. She asked him why he was not returning her book.
10. The saint exclaimed with contentment that they lived in a wonderful world.

Exercise 2
1. The prime minister told the people neither to indulge in nor to tolerate corruption.
2. He suggested me that we should go to watch a movie.
3. The teacher suggested his students to suppose that to be a square.

Learnwell
ACME Math–III
ENGLISH Practice Book GR–10 (Teacher Manual) 47

HF_Acme_ENG_TM_G10.indd 47 5/28/2021 3:44:43 PM


4. Ram told me that he was going to Delhi that day.
5. I asked him why he was disturbing me.
6. Ajay asked me if I had finished my project the previous day.
7. His friends asked him if they had been working in Infosys before that.
8. The teacher asked me why I had not completed my homework.
9. They asked him if he would go with them to the circus that night.
10. The official asked Lata why she was waiting there.

Integrated Exercise
Exercise–1
1. (a) (i) why she had been shouting at her
(b) (i) he had been teasing her
(c) (i) whether he had teased his sister
2. (a) (iii) why he had moved out
(b) (iv) that it had been
(c) (iv) if he believed
3. (a) (ii) how her father was then
(b) (i) that he was running a high temperature
(c) (iv) to bring him there
4. (a) (ii) why she was sitting all alone
(b) (iii) that she was not feeling well
(c) (i) to go to the dispensary
5. (a) (ii) if she had solved (b) (i) Mita to sit down
(c) (iv) she could leave only when she solved all of them
6. (a) (iii) that they should go (b) (iv) she must reach home
(c) (ii) why she was in such a hurry
7. (i) if he might go out to play cricket then
(b) (ii) if he had completed (c) (iii) to complete his work first
8. (a) (ii) why he was making (b) (iii) how he could make it muddy
(c) (i) not to answer him back

Chapter 6 Clauses

ANSWER KEY

Exercise 1
1. Can you tell me the reason why he is not obeying my order?
2. You will not pass if you do not work hard.
3. The girl who is in red dress is the head-girl of the school.

48 ACME English Practice Book GR–10 (Teacher Manual)

HF_Acme_ENG_TM_G10.indd 48 5/22/2021 5:58:40 PM


4. The man who she was singing with is my brother.
5. He did not follow the commands which the teacher gave him.
6. Walk fast otherwise you will miss your flight.
7. Ask the school manager when the school bus will come.
8. Please tell me where your class teacher is.
9. I purchased a new car which was white in colour.
10. I know Mr Jaiswal whose bike was stolen in this market.

Exercise 2
1. I didn’t know where he had gone.
2. He believes that God is omnipresent.
3. The police questioned the eye witness who was present at the accident spot.
4. The minister met the principal who was awarded the Nobel Peace Prize.
5. I waited for my brother till he returned.
6. Tell me where you found my phone.
7. No one knows what is going on in his mind.
8. The doctor came after the patient had died.
9. The girl who is wearing a red saree is my sister.
10. The room which is to be painted is upstairs.
(NOTE: Students can add any other appropriate clauses of their choice to complete these
sentences.)

Integrated Exercise
Exercise–1
1. (a) (iv) how I should solve this problem?
(b) (i) which the teacher told us yesterday.
(c) (iv) where I committed an error.
2. (a) (ii) how you developed it.
(b) (i) that is needed. (c) (iii) how much time I should devote to it every day.

Exercise–2
1. (a) (i) legend that it (b) (ii) know what can
(c) (iii) believe that this (d) (ii) B.C. when slaves
(e) (i) liberty because he (f) (i) wit. As Aesop
2. (a) (ii) unaware that writing (b) (i) on how you
(c) (iii) know that organisation
(d) (ii) down what you (e) (iii) yourself if the
(f) (iii) clearly what you

Exercise–3
1. (a) (ii) when you won it. (b) (ii) which was organized
(c) (iii) how many teams participated in it.
Learnwell
ACME Math–III
ENGLISH Practice Book GR–10 (Teacher Manual) 49

HF_Acme_ENG_TM_G10.indd 49 5/22/2021 5:58:40 PM


2. (a) (iii) why I should get medicines from this shop only?
(b) (iv) which supplies genuine medicines.
(c) (iv) what medicines I should buy?

Exercise–4
(a) which (b) who (c) that
(d) which (e) which (f) which

Exercise–5
(a) (iii) which (b) (i) which (c) (iv) who
(d) (i) whom (e) (ii) that (f) (iv) where

Chapter 7 Determiners

ANSWER KEY

Exercise–1
1. many 2. many 3. Much 4. a 5. X
6. a 7. x, the 8. little 9. little 10. Few

Exercise–2
1. any 2. some 3. any 4. much 5. enough
6. Many 7. an 8. the, a 9. the 10. The

Integrated Grammar
Exercise–1
1. (a) X (b) the (c) an
(d) a (e) the (f) x
2. (a) an (b) the (c) the
(d) many (e) the (f) an
3. (a) other (b) some (c) the
(d) a (e) the (f) the
4. (a) a (b) x (c) x
(d) the (e) A (f) a

Exercise–2
(a) (ii) A (b) (i) a (c) (i) the
(d) (iii) The (e) (ii) the (f) (iii) the

50 ACME English Practice Book GR–10 (Teacher Manual)

HF_Acme_ENG_TM_G10.indd 50 5/28/2021 3:46:05 PM


Exercise–3
(a) (i) These (b) (ii) little (c) (ii) an
(d) (i) any (e) (i) some (f) (iii) our

Exercise–4
(a) (ii) to the picturesque (b) (ii) for a trip (c) (iii) for some days
(d) (ii) made a lot (e) (i) sold all of (f) (ii) At the end

Exercise–5
(a) (ii) ours, our (b) (i) a, the (c) (ii) much, many
(d) (iii) the, a (e) (i) these, this (f) (iii) Any, Some

Chapter 8 Preposition

ANSWER KEY

Exercise 1
1. of 2. beside 3. up 4. on 5. since
6. of 7. of 8. with 9. for 10. before

Exercise 2
1. Everyone congratulated me on my achievement.
2. He has been preparing for CAT examinations for 2 years.
3. Our mother prays to god for our good future.
4. The frog is jumping into the well.
5. I can borrow money from the bank.
6. They stopped him from committing the error of judgment.
7. He died due to severe pneumonia.
8. Ramesh deals in old and new cars.
9. Insects breed more in the summer season in comparison to the winter.
10. A lizard is creeping on the wall.

Integrated Exercise
Exercise–1
1. (a) (ii) since (b) (i) over (c) (i) in
(d) (iii) into (e) (ii) above (f) (iii) until
2. (a) (ii) about (b) (ii) with (c) (ii) for
(d) (i) to (e) (iii) with (f) (iii) with

Learnwell
ACME Math–III
ENGLISH Practice Book GR–10 (Teacher Manual) 51

HF_Acme_ENG_TM_G10.indd 51 5/22/2021 5:58:41 PM


Exercise–2
(a) for (b) in (c) about
(d) of (e) From (f) of

Exercise–3
1. (a) (ii) man on land (b) (i) flies in the
(c) (iii) organism to organism(d) (i) breed in water
(e) (iii) equa in Latin (f) (ii) propagate on land
2. (a) (i) reported from several (b) (ii) due to conflicts
(c) (iii) official and the (d) (i) living in or
(e) (ii) in Bandipur (f) (i) among tribals

Exercise–4
1. (a) (i) on (b) (ii) for (c) (iii) in
(d) (ii) with (e) (i) in (f) (ii) of
2. (a) (iii) in (b) (i) of (c) (iii) after
(d) (iv) in (e) (i) of (f) (ii) of
3. (a) (iii) of (b) (iii) in (c) (i) of
(d) (iv) With (e) (i) into (f) (i) Before

Section-B (B3: Integrated Grammar)

Chapter 1 Cloze Passages

ANSWER KEY

1. (a) (i) up (b) (iii) the (c) (ii) to


(d) (ii) with (e) (i) a (f) (iii) been
(g) (i) a (h) (ii) can
2. (a) (ii) have (b) (iii) to (c) (ii) have
(d) (iii) to (e) (i) a (f) (iii) are
(g) (i) for (h) (ii) are
3. (a) (i) in (b) (i) a (c) (iii) took
(d) (i) to (e) (ii) till (f) (iii) the
(g) (iii) which (h) (iv) of
4. (a) (iii) to (b) (iii) the (c) (iii) the
(d) (i) along (e) (i) be (f) (i) with
(g) (iii) any (h) (i) and

52 ACME English Practice Book GR–10 (Teacher Manual)

HF_Acme_ENG_TM_G10.indd 52 5/28/2021 3:47:43 PM


5. (a) (iii) more (b) (i) As (c) (iii) the
(d) (ii) to (e) (i) with (f) (ii) only
(g) (ii) while (h) (ii) an
6. (a) (ii) too (b) (ii) much (c) (ii) are
(d) (i) understand (e) (iii) the (f) (i) before
(g) (i) with (h) (iii) my
7. (a) (i) a (b) (i) is (c) (ii) who
(d) (iii) that (e) (ii) are (f) (ii) an
(g) (iii) in (h) (ii) and
8. (a) (iii) as (b) (ii) are (c) (ii) to
(d) (i) be (e) (iv) only (f) (iv) with
(g) (iv) x (h) (ii) to
9. (a) (ii) a (b) (iv) x (c) (i) for
(d) (ii) been (e) (i) among (f) (i) the
(g) (iv) x (h) (iii) of
10. (a) (i) the (b) (iii) in (c) (ii) from
(d) (iii) the (e) (i) is (f) (ii) and
(g) (i) a (h) (iii) are
11. (a) (iii) to (b) (iv) have (c) (iv) X
(d) (ii) are (e) (ii) of (f) (i) is
(g) (ii) helps (h) (ii) be
12. (a) (ii) of (b) (iii) still (c) (ii) against
(d) (i) is (e) (iii) has (f) (iii) to
(g) (i) forward (h) (ii) have
13. (a) (iii) all (b) (ii) to (c) (i) a
(d) (iv) in (e) (ii) being (f) (ii) an
(g) (ii) of (h) (i) dry
14. (a) (iii) have (b) (i) being (c) (ii) both
(d) (iv) a (e) (i) can’t (f) (i) of
(g) (ii) producing (h) (i) to
15. (a) (i) being (b) (i) and (c) (iii) are
(d) (iii) the (e) (i) with (f) (i) will
(g) (iv) for (h) (i) threaten
16. (a) (i) a (b) (i) is (c) (iii) of
(d) (iv) in (e) (ii) as (f) (iv) in
(g) (i) offer (h) (i) and
17. (a) (iii) been (b) (ii) its (c) (ii) yet
(d) (ii) like (e) (ii) more (f) (i) a
(g) (iv) it (h) (ii) that
18. (a) (ii) its (b) (iv) from (c) (iii) the
(d) (ii) their (e) (i) for (f) (ii) have
(g) (iii) often (h) (ii) with

Learnwell
ACME Math–III
ENGLISH Practice Book GR–10 (Teacher Manual) 53

HF_Acme_ENG_TM_G10.indd 53 5/22/2021 5:58:41 PM


19. (a) (i) a (b) (iv) beyond (c) (iii) been
(d) (iii) of (e) (i) for (f) (ii) to
(g) (iii) the (h) (ii) by
20. (a) (ii) was (b) (iii) the (c) (iii) at
(d) (ii) by (e) (ii) covers (f) (ii) other
(g) (i) is (h) (iv) around

Chapter 2 Gap Filling

ANSWER KEY

1. (a) with (b) is (c) their (d) and (e) the


2. (a) is (b) to (c) with (d) yourself (e) hear
3. (a) are (b) of (c) an (d) in (e) is
4. (a) in (b) would (c) running (d) so (e) were
5. (a) during (b) Often (c) is (d) for (e) with
(f) Once
6. (a) is (b) of (c) hotly (d) will (e) be
(f) Either
7. (a) into (b) everything (c) never (d) nor (e) with
(f) who
8. (a) with (b) with (c) the (d) for (e) great
(f) gathered
9. (a) for (b) few (c) on (d) of (e) to
(f) over
10. (a) was (b) where (c) a (d) often (e) upon
(f) above
11. (a) and (b) to (c) who (d) on (e) to
(f) by (g) as (h) of
12. (a) a (b) as (c) times (d) more (e) from
(f) of (h) and
13. (a) of (b) either (c) it (d) as (e) are
(f) to (g) away (h) The (h) According
14. (a) has (b) were (c) their (d) too (e) and
(f) then (g) no (h) a
15. (a) in (b) to (c) be (d) quite (e) for
(f) have (g) which (h) off
16. (a) of (b) too (c) a (d) up (e) away
(f) itself (g) It (h) finds
17. (a) for (b) at (c) of (d) of (e) The
(f) to (g) to (h) to

54 ACME English Practice Book GR–10 (Teacher Manual)

HF_Acme_ENG_TM_G10.indd 54 5/22/2021 5:58:41 PM


18. (a) so (b) don’t (c) the (d) instead (e) at
(f) all (g) at (h) to
19. (a) of (b) were (c) those (d) but (e) of
(f) been (g) over (h) of
20. (a) in (b) in (c) of (d) into (e) by
(f) the (g) with (h) most
21. (a) to (b) can (c) upon (d) the (e) and
(f) have (g) be (h) a
22. (a) to (b) be (c) of (d) to (e) of
(f) a (g) such (h) within

Chapter 3 Editing

ANSWER KEY

1. (a) a, the (b) on, around (c) don’t, didn’t (d) an, a (e) at, in
(f) for, with (g) a, an (h) over, above
2. (a) Of, For (b) a, the (c) a, the (d) his, their (e) was, is
(f) to, by (g) was, is (h) to, for
3. (a) has, have (b) at, on (c) claim, claims (d) a, an
(e) to, on (f) let, lets (g) at, in (h) in, on
4. (a) on, from (b) whose, which
(c) entertain, entertainment (d) which, where (e) so, as
(f) entertain, entertainment (g) hour, hours (h) or, and
5. (a) epic, epics (b) take, took (c) for, of (d) on, in
(e) there, their (f) and, but (g) book, books
(h) exercises, exercised
6. (a) till, to (b) in, to (c) at, in (d) for, of (e) them, it
(f) on, by (g) women, woman (h) their, his
7. (a) is, are (b) for, to (c) conserve, conserved (d) on, to
(e) of, for (f) have, has (g) by, with (h) by with
8. (a) sphere, spheres (b) is, are (c) ours, our (d) form, forms
(e) percents, percent (f) have, has (g) at, in (h) who, which
9. (a) consider, considered (b) is, has (c) at, in (d) of, about
(e) among, between (f) become, becoming
(g) on, of (h) percents, percent
10. (a) week, weeks (b) at, with (c) life, lives
(d) suggest, suggested (e) their, its (f) trigger, triggered
(g) reader, readers (h) remind, reminded
11. (a) he, him (b) own, owns (c) culture, cultural
(d) line, lines (e) been, being (f) on, in
(g) preach, preached (h) among, between
Learnwell
ACME Math–III
ENGLISH Practice Book GR–10 (Teacher Manual) 55

HF_Acme_ENG_TM_G10.indd 55 5/22/2021 5:58:41 PM


12. (a) became, become (b) for, of (c) put, puts (d) the, a
(e) on, of (f) was, is (g) being, be (h) this, that
13. (a) is, are (b) for, to (c) hunt, hunting (d) part, parts
(e) to, for (f) a, an (g) big, biggest (h) of, with
14. (a) feat, feats (b) being, been (c) are, is (d) is, are
(e) to, of (f) branch, branches (g) or, and (h) on, with
15. (a) he, it (b) for, to (c) have, has (d) to, for
(e) up on, upto (f) on, to (g) of, for (h) is, are
16. (a) don’t, aren’t (b) aren’t, don’t (c) its, their
(d) question, questions (e) are, have (f) by, in (g) his, their
(h) of, for
17. (a) on, with (b) are, is (c) at, in (d) at, in (e) are, is
(f) favour, favours (g) cure, cures (h) boil, boiled
18. (a) amount, amounts (b) observe, observed (c) are, is
(d) been, being (e) for, of (f) has, is (g) the, that
(h) remark, remarked
19. (a) at, with (b) strength, strengthen (c) protect, protected
(d) it, them (e) like, such as (f) being, been (g) consult, consulting
(h) cover, covers
20. (a) for, so (b) participate, participating (c) tree, trees (d) says, said
(e) their, his (f) his, their (g) preserve, preserved
(h) inaugurate, inaugurating

Chapter 4 Dialogue Writing

ANSWER KEY

1. Reena: Mother, Anita has invited me to her birthday party the coming Friday. Can I
go to the party?
Mother: Where is the party and at what time will it begin?
Reena: It will begin at 7 o’clock at the Café Carma.
Mother: At what time will you come back?
Reena: I will be back by 9:30.
Mother: OK. You can go. But please provide me Anita and her parents’ phone numbers,
and their address too.
2. Jessie: Why do you look so upset?
Rohan: My car has broken down for the fourth time in the last fifteen days. I am sick
of getting it fixed again and again.
Jessie: Why are you spending so much on its repair again and again? Why don’t you
buy a new car?
Rohan: I don’t have enough money at present to purchase a car.

56 ACME English Practice Book GR–10 (Teacher Manual)

HF_Acme_ENG_TM_G10.indd 56 5/22/2021 5:58:42 PM


Jessie: Don’t worry about it as you can take a loan from me and pay me later.
Rohan: Thank you. I will think about it.
3. Neeti: Good morning, teacher. I want to speak to you for a moment.
Ms Geeta: Sure. Please take a seat.
Neeti : I have taken part in the dance competition and I need time to practise.
Ms Geeta : When will you need to go for practice?
Neeti : I need to go for the practice in the last class every day.
Ms Geeta : Write an application to the principal asking for her permission.
Neeti : Thank you.
4. Customer: I purchased a mobile phone from your shop a week ago. The phone’s
battery has started to heat up as soon as it is put for charging.
Shopkeeper: I am so sorry for the inconvenience. Can you show me the bill of the
purchase?
Customer: I don’t have the bill of the phone.
Shopkeeper: In that case I won’t be able to help you.
Customer: The phone is still under warranty.
Shopkeeper: You need to show the bill to avail the benefits of the warranty.
5. Rahul : I have discovered a new bookstore in our vicinity.
Shabana : Where is it and how did you find it?
Rahul: I had gone to the shopping complex to buy some fruits and I discovered the
new shop there. The store has all the latest novels.
Shabana : Do they have stationary items as well?
Rahul: Yes. They have a great variety of stuff.
Shabana : Let’s go there in the evening to take a look.
6. Interviewer : Hello, Nisha. Welcome! Please have a seat.
Nisha : Thank you.
Interviewer : Why do you want to join our school?
Nisha : Yours is one of the prestigious schools in the city and I will be honoured to
be a part of it.
Interviewer : Tell something about your job experience.
Nisha : I have been working with Presidium School for the past two years and have
taught classes 3-5.
7. Yash : How are you planning to spend your summer vacations?
Mohan : My parents are planning a trip to Rishikesh.
Yash : Wow! Rishikesh is an amazing place.
Mohan : My father has booked two tent houses for us on the banks of River Ganga.
Yash : You should also do river rafting there as that is very exciting.
Mohan : Thank you for the information. We will definitely try that.
8. Sonam : By what time will the final list of passengers be updated?
TTE: It will be updated half an hour before the train leaves.
Sonam : Where can I find the list?
TTE : You should check the bulletin board near the telephone booth.
Sonam : Is the train on time?
TTE : Yes

Learnwell
ACME Math–III
ENGLISH Practice Book GR–10 (Teacher Manual) 57

HF_Acme_ENG_TM_G10.indd 57 5/22/2021 5:58:42 PM


9. Sneha : My school has organised an NCC camp in Nainital and I want to go.
Mother : What is the duration of the camp and who all will be going there?
Sneha : The camp is for a duration of one week and about twenty students and four
teachers of my school will be taking part in the camp.
Mother : Where will you all stay?
Sneha : We will be staying in camps near Bhimtal.
Mother: OK. Prepare a list of articles you will need. We will get them from the market.
Sneha :Thank you. Please sign the NOC.
10. Mahesh: I am having problem hearing.
Doctor: For how long have you been suffering from the problem?
Mahesh : For the past two days I have not been able to hear properly.
Doctor : It seems that you have an ear infection. Get some medicines from the medical
store.
Mahesh: How long will it take to heal?
Doctor: You will be fine in three–four days.
11. Mother: Priya , how many times have I asked you not to eat junk food?
Priya: The burger that I am eating is given to me by my friend, Meeta.
Mother: Don’t make stupid excuses. Junk food has no nutritional value and can lead
to many lifestyle diseases.
Priya: It is impossible to avoid eating junk food if other friends are eating it.
Mother: You should strengthen your will to say no.
Priya: Okay! I will say no to junk food if anyone offers it to me.
12. Ankush: Hi, Sudha! Where have you been?
Sudha: I have been taking coaching to crack IIT-JEE Main.
Ankush: Have you made up your mind to be an engineer?
Sudha: Yes. What are you doing at present?
Ankush : I have been taking photography classes for some time now.
Sudha: Wow! That’s wonderful.
13. Ankur: I lost the wristwatch you and dad had gifted me on my birthday.
Mother: Oh! Where have you lost it?
Ankur : I have not seen it after the physical education period. So probably it got lost
in the playground.
Mother : Have you put a Lost and Found notice on the school’s notice board?
Ankur : Yes, but there has not been any news.
Mother : It is all right. Don’t fret.
14. Father :You have performed poorly yet again in examination.
Krishna : I couldn’t study properly as I had to prepare for football tournament.
Father : You have to concentrate equally on both your studies and football.
Krishna : Sorry, father. I will try to perform well in the next exam.
Father : You need to make a time table which will help you divide your time efficiently
between both the activities.
Krishna : OK, father. I will do that.

58 ACME English Practice Book GR–10 (Teacher Manual)

HF_Acme_ENG_TM_G10.indd 58 5/22/2021 5:58:42 PM


15. Sagar : Did you attend the seminar on water conservation?
Mitali : Yes, it was an eye-opener for me.
Sagar : Since monsoons are approaching, we can ask our society members to put up
water conservation tanks at strategic places to store rain water.
Mitali : It is a great idea.
Sagar : What else we can do?
Mitali : We can list a few pointers for water conservation and put it on the notice-
board of our apartments. We can also make and distribute flyers.

Chapter 5 Reporting a Dialogue

ANSWER KEY

1. (a) if he was fine. (b) why he was looking so tired.


(c) that Rahul had fallen into a well
(d) he had to pull him out. (e) how he had done that
(f) he had lowered a rope into the well and then pulled it up as Rahul had clung to it.
2. (a) that he had a terrible ache in gum
(b) he was unable to chew his food
(c) that he needed to examine his mouth
(d) to open his mouth wide (e) if there was any serious problem
(f) that he had an infection in his gums
3. (a) if she knew that Meena had met
(b) was in the hospital (c) if she was badly hurt
(d) which hospital she was in
(e) it had been a minor accident. There was nothing to worry.
(f) she was admitted to Kailash
4. (a) if he knew how to play football
(b) in negative (c) he had never had a chance to learn it
(d) that in that school every student had to learn it
(e) he should learn it too (f) he would try to learn it.
5. (a) that a thief had entered the house
(b) why he didn’t bark to awake their master
(c) he ill treated him (d) he didn’t care what happened to him
(e) that was not the time to complain
(f) as he didn’t care for him, he wouldn’t care for him
6. (a) why he had left his previous job
(b) he had wanted to gain more knowledge
(c) how much salary he expected there
(d) that he was expecting `30,000 per month
(e) they could pay him only `25,000 per month
(f) that amount was not acceptable to him

Learnwell
ACME Math–III
ENGLISH Practice Book GR–10 (Teacher Manual) 59

HF_Acme_ENG_TM_G10.indd 59 5/28/2021 3:49:44 PM


7. (a) what had happened (b) he was standing in the farm at that time
(c) he was trying to win a Nobel Prize
(d) what that had to do with the Nobel prize
(e) the Noble Prize is awarded to people outstanding in their field
(f) he would win one too
8. (a) there was a bad news
(b) two of his teeth were decaying and they might have to uproot them
(c) if there wasn’t any other solution
(d) that he could put a crown on the two teeth
(e) what that was
(f) that a crown is a custom-fitted covering that is placed over the tooth to treat
extensive decay
9. (a) she had read his short story
(b) if she liked it
(c) she hadn’t found many mistakes in it
(d) there hadn’t been many mistakes
(e) on the contrary he had done a fantastic job
(f) thanked her for her kind words
10. (a) not to get too close (b) he was a mean one
(c) notice the fierce gleam. He also added that he came from a long line of killers
(d) he was asleep
11. (a) a happy birthday (b) if he didn’t get to blow out any candles
(c) they had not known how old he was so they had left them off
(d) where the cake was
(e) they had not been even sure it was his birthday so they hadn’t brought one
12. (a) if he thought Hobbes was getting a little dirty
(b) both of them could use a bath
(c) oh boy (d) he was taking his bath upstairs
(e) he was a lucky rat (f) try the cold rinse cycle
13. (a) that that was quite a sandwich
(b) that it was too beautiful to eat
(c) beauty is in the eye of the beholder
(d) where his sandwich was (e) beauty was then in the stomach of the beholder
14. (a) his grades were falling (b) what had happened to him
(c) he didn’t have time to study
(d) why that was (e) his mother was suffering from cancer
(f) he had to regularly take her to the hospital.
15. (a) how she was (b) he hadn’t seen her since last month
(c) she’d just come back from her holiday in London
(d) if she had enjoyed her stay
(e) she had loved London and also added that it was a beautiful city
(f) if she had seen the London Bridge

60 ACME English Practice Book GR–10 (Teacher Manual)

HF_Acme_ENG_TM_G10.indd 60 5/22/2021 5:58:42 PM


Section-B (B4: Olympiad)

Olympiad

ANSWER KEY

Olympiad – I
1. (d) colourful 2. (a) longest 3. (a) most horrible
4. (b) more specific 5. (a) strongly 6. (d) cleverly
7. (d) immensely 8. (b) roughly 9. (a) a
10. (b) an 11. (a) A 12. (c) so
13. (c) for 14. (b) magpies 15. (a) clam
16. (d) heart 17. (c) shoes 18. (d) could
19. (d) should 20. (b) interfering 21. (a) Seeing
22. (b) open
23. (b) Because Rekha had no money with her, she could not buy the book.
24. (c) As you pull the chain, the train will stop.
25. (a) Cotton is not as expensive as terrycot.

Olympiad – II
1. (a) at 2. (b) of 3. (a) but
4. (d) is 5. (b) deserves 6. (a) is
7. (c) As you pull the chain, the train will stop. 8. (c) runs
9. (c) were playing 10. (a) had slept 11. (c) the
12. (d) X 13. (c) The, the 14. (a) a
15. (d) x, x 16. (b) so 17. (a) when
18. (a) where 19. (d) either 20. (d) but also
21. (c) than 22. (b) unless
23. (d) Mr and Mrs Sharma are being questioned by the police.
24. (a) The enemy was compelled to surrender.
25. (c) Some of the cargo had been damaged by the sea water.

Olympiad – III
1. (c) as generous as 2. (a) more peaceful 3. (c) shorter
4. (d) a more exciting 5. (b) most wonderful 6. (a) usually
7. (c) frequently 8. (a) silently 9. (c) always
10. (b) my ; I 11. (d) your, I 12. (c) It, my
13. (c) She, us 14. (b) I, them 15. (a) hard
16. (b) peas 17. (b) gold 18. (c) chew

Learnwell
ACME Math–III
ENGLISH Practice Book GR–10 (Teacher Manual) 61

HF_Acme_ENG_TM_G10.indd 61 5/22/2021 5:58:42 PM


19. (c) scarecrow 20. (a) poker 21. (c) speak
22. (b) to drive 23. (d) broken 24. (b) to teach
25. (b) cutting

Olympiad – IV
1. (b) has not been swimming 2. (b) has been working 3. (b) is downloading
4. (b) has been studying 5. (a) has 6. (d) were
7. (a) is 8. (d) were 9. (a) are
10. (d) at 11. (a) on 12. (d) with
13. (a) of 14. (a) in 15. (c) if
16. (b) until 17. (d) Unless 18. (c) for
19. (a) As far as 20. (d) x, x
21. (d) I was annoyed with his behaviour.
22. (a) What is being done by you today?
23. (b) By whom was the door being knocked at?
24. (d) Milk is contained in the jug.
25. (c) Food will have been eaten by Tina by evening.

Olympiad – V
1. (d) He, us 2. (d) They, their 3. (c) You, her
4. (d) me 5. (a) we, we 6. (a) more patient
7. (b) more committed 8. (b) hardest 9. (d) nice
10. (d) youngest 11. (b) kindly 12. (a) sometimes
13. (d) slightly 14. (b) brightly 15. (b) greatly
16. (b) niece 17. (a) congregation 18. (b) directors
19. (c) boss 20. (c) manager 21. (c) must
22. (b) Where was this pen found by you?
23. (b) By whom were you called names?
24. (c) Am I loved by you? 25. (b) He is disgusted by dirty lanes.

Olympiad – VI
1. (c) has done 2. (b) saw 3. (b) has been
4. (c) have completed 5. (a) is 6. (c) are located
7. (a) has 8. (a) is 9. (c) has
10. (a) at 11. (a) in 12. (b) to
13. (b) in 14. (a) to 15. (c) should
16. (a) May 17. (b) might 18. (c) No sooner
19. (b) before 20. (c) after 21. (a) in order that
22. (a) or
23. (b) The cheque was taken from the customer by the accountant.
24. (a) He was refused admittance by the gatekeeper.
25. (a) The political leaders were being interviewed by Sohan.

62 ACME English Practice Book GR–10 (Teacher Manual)

HF_Acme_ENG_TM_G10.indd 62 5/22/2021 5:58:42 PM


Olympiad – VII
1. (a) Blame 2. (c) Insolvent 3. (a) Capable
4. (a) Huge 5. (d) Surplus
6. (d) most poisonous 7. (c) mysterious 8. (a) sweeter
9. (d) silly 10. (c) more appropriate 11. (a) foolishly
12. (a) undoubtedly 13. (d) repeatedly 14. (c) rapidly
15. (a) often 16. (b) our 17. (a) I, We
18. (a) my, me 19. (b) her, her 20. (d) against
21. (d) to 22. (a) on
23. (b) A perfect dam has been built by the builders across the stream.
24. (b) All the instructions should be carefully followed by them.
25. (c) Here they teach us English.

Olympiad – VIII
1. (d) x, x 2. (c) The 3. (a) a, a
4. (c) the 5. (c) The, the 6. (a) and
7. (b) and 8. (b) but 9. (c) so
10. (d) Unless 11. (c) will 12. (a) will
13. (a) should 14. (d) will 15. (c) Shall
16. (b) into 17. (b) to 18. (a) to
19. (a) before 20. (b) to 21. (a) were
22. (b) was 23. (a) were 24. (a) flows
25. (b) boils

Olympiad – IX
1. (c) many 2. (a) much 3. (b) Many
4. (a) Much, many 5. (d) a little 6. (b) strangely
7. (b) rarely 8. (a) always 9. (b) clumsily
10. (c) partially 11. (a) class 12. (c) spectators
13. (a) singers 14. (b) hostess 15. (c) employer
16. (d) mine, yours 17. (d) you, my 18. (c) my, my
19. (c) you, me 20. (a) myself, she 21. (a) at
22. (c) against 23. (c) at 24. (a) from
25. (c) with

Olympiad – X
1. (a) a, a 2. (d) x 3. (b) an
4. (d) x 5. (d) x 6. (c) for
7. (a) As far as 8. (c) No sooner 9. (b) before
10. (a) and 11. (b) used to 12. (a) Would
13. (d) May 14. (c) may 15. (b) Would
16. (d) with 17. (c) by 18. (a) to

Learnwell
ACME Math–III
ENGLISH Practice Book GR–10 (Teacher Manual) 63

HF_Acme_ENG_TM_G10.indd 63 5/22/2021 5:58:42 PM


19. (a) of 20. (b) to
21. (a) No moss is gathered by a rolling stone.
22. (d) The minister’s speech was loudly cheered by the audience.
23. (a) My pocket has been picked.
24. (a) By whom were you taught such tricks as these?
25. (b) He was found guilty of robbery by them.

Section-C (C2: Prose (First Flight))

Chapter 1 A Letter to God

ANSWER KEY

Passage Based Questions


1. (a) The field of ripe corn dotted with flowers promised a good harvest.
(b) Lencho saw the sky towards the north-east because he was waiting for rainfall
that would ensure that he would get a good crop.
2. (a) During the meal, as Lencho had predicted, big drops of rain began to fall. In the
north-east, huge mountains of clouds could be seen approaching. The air was
fresh and sweet.
(b) The rain brought with it hailstones that destroyed the crops. Thus, the pleasant
rain turned into an evil one for Lencho.
3. (a) Lencho was satisfied because he thought the raindrops would ensure a good crop
which would eventually help them earn good money.
(b) All of a sudden, a strong wind began to blow and along with the rain very large
hailstones began to fall which eventually destroyed the crops.
4. (a) Lencho expected that the raindrops would ensure a good crop which would
eventually help earn good money. But what he expected didn’t happen as huge
hailstones started falling and destroyed his crop.
(b) The hailstones covered the entire valley. The field looked as white as salt. The
leaves had fallen from the trees, corns were smashed and flowers had fallen from
the plants.
5. (a) (i) A farmer (b) (ii), (iii), (iv)
(c) (iv) He was irritated by post office employees.
(d) (iv) All the above (e) (iii) Crooks
6. (a) b) F—i, iii and O—ii, iv (b) (ii) Through a letter
(c) (iii) It was addressed to God.
(d) (i) Hardworking and strong person
(e) a) option (i)

64 ACME English Practice Book GR–10 (Teacher Manual)

HF_Acme_ENG_TM_G10.indd 64 5/22/2021 5:58:43 PM


Short Answer Type Questions
1. In the end Lencho called the post office employees ‘a bunch of crooks’ because he
believed that they had taken his money. He was unaware that they contributed money
for him. Therefore, the ending of the story is ironical.
2. Lencho said that the raindrops were like new coins. The crops would benefit from
these raindrops. It would provide good harvest which would help him earn a lot of
money.
3. No, Lencho didn’t try to find out who had sent the money. He had unshakable faith
in God and believed that God had helped me.
4. Being a generous person, the postmaster asked everyone in the post office to contribute
towards helping Lencho. He collected the money, put it in an envelope and signed
it ‘God’. He did this so that Lencho’s faith is not shaken.
5. Lencho’s family members were sorrowful at the damage done to the crops. They felt
that all their hard work had been futile. They thought they would have to go hungry
that year.
6. Lencho had unwavering faith in God. He was hard-working and unlike other farmers,
he knew how to write. He was naïve, innocent and expected to recover the loss of
crops by asking for help from God.
7. In his letter, Lencho wrote that if God didn’t help him and his family, they go hungry
that year. He needed a hundred pesos in order to sow his field again and to live until
the crop comes because the hailstorm had destroyed his standing crop.
8. The postmaster is a generous and helpful person. He also respected others’ faith. He
collected 70 pesos to help Lencho and not let his faith shake in God.

Long Answer Type Questions


1. Lencho had great faith in God but much less faith in men. Being a farmer, Lencho
was completely dependent upon his crops. Due to the hailstorm his annual crop
was completely destroyed. He had deep faith in God and believed that God would
help him. He wrote a letter to God asking him for 100 pesos. When he got seventy
pesos, which were actually collected by post office employees, he blamed them for
cheating. He could not believe that God had made a mistake. So, he wrote another
letter to God asking him to send the rest of the money. He requested him not to send
it through mail because he believed that the post office employees were ‘a bunch of
crooks’. This shows his positive attitude towards God and negative attitude towards
humans.
2. When Lencho received 70 pesos, he called the post office employees “a bunch of
crooks”. He thought that the employees have kept the remaining thirty pesos. He
was unaware that the money was sent by them. As soon as the postmaster received
Lencho’s first letter, he read it and expressed his concern towards Lencho. He gave
up a part of his salary and asked all other employees to help. Thus, he managed to
collect 70 pesos and sent them to Lencho in an envelope. He even wrote God on
the letter. Hence, we can say that the postmaster and the employees were generous
and helpful people. They were not a bunch of crooks.

Learnwell
ACME Math–III
ENGLISH Practice Book GR–10 (Teacher Manual) 65

HF_Acme_ENG_TM_G10.indd 65 5/22/2021 5:58:43 PM


Nelson Mandela:
Chapter 2
Long Walk to Freedom
ANSWER KEY

Passage Based Questions


1. (a) Mandela’s believed that if people can be taught hatred, they can also be taught
love and brotherhood. People are not born with hatred in their hearts. They learn
to hate others while growing up as they learn to discriminate and hate from others
in the society.
(b) Love comes naturally to the human heart. It is inborn. People are born with love
and innocence in their heart. They learn to hate others from their environment.
2. (a) It was impossible for the Blacks in South Africa to fulfill their responsibility towards
the family as well as the society. It was because in South Africa, a man of colour
who attempted to live as a human being was punished and isolated.
(b) A man has obligations to his family, to his parents, to his wife and children; and
he has an obligation to his people, his community, and his country. These are the
twin obligations he has.
3. (a) This hunger transformed the speaker completely. He was a brave man who
transformed from a lawyer to the one breaking the law, a family-man to a man
without a home and a lively man to a monk.
(b) Mandela was aware about the demands of his people. All they wanted was
freedom. He started working to attain freedom for his people. So, he was forced
to live the life of a monk.
4. (a) According to the writer, the oppressor as well as the oppressed needs freedom.
The oppressed needs freedom from restrictions and the oppressor needs freedom
from hatred.
(b) Mandela was of the opinion that a man who takes away another man’s freedom
is a prisoner of hatred because he himself is a victim of prejudice and narrow-
mindedness. He justified it by saying the oppressor and the oppressed are alike
because one takes away someone else’s freedom and the other’s freedom is taken
away.
5. (a) d) (iv), (v) (b) (iii) His countrymen were not free.
(c) (iv) Chains on himself (d) (i) Freedom (e) c) option iii
6. (a) (iv) Its people (b) (iv) Minerals (c) (ii) His comrades
(d) b) F - (i), (ii), (iii) and O -(iv) (e) (iv) toughness

Short Answer Type Questions


1. Mandela remembered the history of racial discrimination against Black people as “an
extraordinary human disaster”. Black people of South Africa who were natives of
the country were deprived of their freedom and treated like a second-class citizen.
2. Before Mandela, thousands of freedom fighter had struggled to gain freedom for
Blacks in South Africa. They had made great sacrifices during their fight for freedom.

66 ACME English Practice Book GR–10 (Teacher Manual)

HF_Acme_ENG_TM_G10.indd 66 5/22/2021 5:58:43 PM


He said that these sacrifices could not be repaid. He considered himself as the sum
of all of those African patriots.
3. The policy of apartheid created a deep and lasting wound in South African Blacks.
The policy segregated the natives on the basis of the race and colour. This led to
discriminatory behaviour towards the Blacks who were treated like a second class
citizen and didn’t have access to basic facilities of life. This also led to animosity
between the Blacks and the White. This policy had so deeply wounded the people
of his country that it would take centuries to heal.
4. The policy of apartheid became instrumental in producing patriots of extraordinary
courage, wisdom and resilience. Many great men like Oliver Tambos, Walter Sisulu,
Yusuf Dadoo, Bram Fischer, etc. became well known for their fight against brutality
and oppression. They were men of great character.
5. In Mandela’s opinion, the greatest wealth of a country is not minerals or gems. The
greatest wealth is its people who struggle together to bring in change in the system.
6. According to Mandela, courage means to triumph over fear. A brave man is not he
who does not feel afraid, but he who conquers that fear. Mandela learnt the true
meaning of courage from his comrades.
7. The inauguration was attended by a large number of international leaders. The
international community came to support the end of apartheid. It signified the triumph
of South African people who struggled against evil.
8. Nelson Mandela realised his hunger for freedom when he found his boyhood freedom
was an illusion. As a young man, he was stripped off of his freedom.

Long Answer Type Questions


1. On the day of the oath-taking ceremony, the two anthems were sung. Nkosi Sikelel-i-
Afrika was the anthem of the whites and Die stem was the old anthem of the Republic
sung by the blacks. Singing both the anthems epitomises the equality of rights between
the whites and the blacks. In South Africa, a brutal practice named apartheid was
followed which deprived dark-skinned people of their basic rights. Mandela along with
his comrades struggled to get freedom for their people and create a society where
no one would be discriminated on the basis of caste, colour, race, age or gender.
Although no one knew the lyrics of the anthem they once despised, Mandela believed
that everyone would soon learn it.
2. Being a great patriot, Nelson Mandela has sacrificed a lot for the welfare of his
country and its people. Considered an outlaw by the government, his struggle against
apartheid kept him away from his family. He was locked and tortured in jail. This did
not deter his conviction. He never resorted to violence and encouraged his comrades
and followers to adopt non-violence in their struggle for freedom. Mandela recalled
his prison days when he was tortured, he could see glimmer of humanity in some
guards. It was the essential goodness of human heart which could not be wiped out.
He believed in the goodness of human nature. Man is not born with a desire to hate
anyone. As he can be taught to hate, he can also be taught to love. Hence, it shows
that Mandela followed the principles of non-violence and peace.

Learnwell
ACME Math–III
ENGLISH Practice Book GR–10 (Teacher Manual) 67

HF_Acme_ENG_TM_G10.indd 67 5/22/2021 5:58:43 PM


Chapter 3 Two Stories about Flying

ANSWER KEY

I. His First Flight


Passage Based Questions
1. (a) Young seagull could not move. He feared to take the first plunge to be able to fly.
(b) His parents motivated him to put efforts to fly. His father and mother had come
around calling to him shrilly, upbraiding him, threatening to let him starve on his
ledge unless he flew away.
2. (a) Nobody had come near the young seagull for twenty-four hours because they
wanted him to fly towards them. They wanted him to learn to fly.
(b) He had watched his parents flying about with his brothers and sister. They were
perfecting them in the art of flight, teaching them how to skim the waves and
how to dive for fish. His older brother had caught his first herring and his parents
circled around him, praising him.
3. (a) When the young seagull saw his mother flying across to him with a piece of fish
in her mouth, he uttered a joyful scream. He was begging her for food for long
before this.
(b) The young seagull’s mother enticed him with food to make him fly. She knew that
he was very hungry and would jump towards her on seeing the food. Maddened
by hunger, the seagull dived at the piece of fish his mother was carrying in her
beak.
4. (a) The young seagull’s mother was aware of his latent potential to fly without any
difficulty.
(b) The young seagull screamed in horror. Then a monstrous terror seized him and
his heart stood still. He could hear nothing. But it only lasted a minute. The next
moment he felt his wings spread outwards. Soon he learnt to flap them and soar
upwards.
5. (a) (iii) Young Seagull (b) (iv) Flying
(c) (i) For diving and soaring
(d) a) (i), (ii), (iv) (e) b) Option (ii)
6. (a) d) F - (ii), (iii), (iv) and O - (i)
(b) (iv) Calling to him (c) (i) Because of hunger and exhaustion
(d) (i) Bravery (e) (ii) calling

Short Answer Type Questions


1. The young seagull was maddened with hunger. When he saw his mother flying
towards him with a piece of fish in her mouth, he screamed with joy. He jumped
at her but she moved back. The young seagull fell down from the cliff. But soon he
started flapping his wings and started flying.

68 ACME English Practice Book GR–10 (Teacher Manual)

HF_Acme_ENG_TM_G10.indd 68 5/22/2021 5:58:43 PM


2. The young seagull’s flight was important. It would teach him to be independent. He
would not have to depend on his parents for food.
3. To grab the attention of his family, the young seagull stood on one leg at the edge of
the ledge. He closed his eyes pretending to have fallen asleep.
4. The seagull was at first very scared and frightened. He flapped his wings to rise. But
he was tired and weak with hunger and he could not rise. But soon his belly touched
the green sea and he began floating.
5. The seagull’s father and mother wanted young seagull to be independent. They
threatened and cajoled him so that he could learn to fly.
6. When the young seagull fell down the ledge, he uttered a loud scream. He could
hear nothing. But soon he began to flap his wings and started rising up. He began
soaring gradually downwards and outwards. He was no longer afraid. He just felt a
bit dizzy.
7. Yes, the young seagull was less fearless in comparison to his brothers and little sister
as both his siblings learnt to fly before him. His parents had to put in a lot of efforts
to make him fly. But ultimately he overcame his fear and learnt to fly.
8. The young seagull’s family was walking on the plateau to motivate him to fly. They
were also taunting him for his cowardice.

Long Answer Type Questions


1. The young seagull was frightened to take his first flight because of the vast sea below.
His parents tried everything to make him overcome his fear. They screamed, scolded,
cajoled and threatened to let him starve on the ledge unless he flew. They left him
alone at the ledge. The hungry young seagull felt miserable on the ledge. He begged
his mother for food. They ignored him for a long period. After some time, his mother
carried a piece of fish in her mouth and flew towards him. The sight of fish maddened
him. In order to grab the fish, he jumped at her. His mother moved away from the
edge of the cliff and the young seagull fell downwards. He was terrified. But soon he
learnt to flap his wings and began soaring upwards. He took control of his movement
and started flying in the sky. So, jumping from the edge to satisfy hunger was the
prime factor that made the young seagull fly.
2. Yes, I can sympathise with the young seagull. To jump off a cliff was no mean feat
for anyone. Everyone is afraid of something or the other. A baby is also afraid to
take its first step. A child is afraid to take his/her first jump. It’s natural. Some people
learn early, some take their time. The stern treatment meted out to him for his sake
is justified to a certain extent. The young seagull was scared of flying. His brothers
and sister had already learned to fly but he could not find courage to fly. His parents
scolded him shrilly, left him alone and threatened to let him starve on the ledge in an
effort to make him fly. After some time, his mother took a piece of fish in her mouth
and intentionally flew across to him and halted at the edge. Maddened by the sight
of food, the young seagull dived at the fish. Her mother aware of his step swooped
upwards. He fell outwards and downwards. Gradually his fear of flying was over and
he enjoyed it now. At first he was terrified but soon he started flapping his wings and
soared upwards.

Learnwell
ACME Math–III
ENGLISH Practice Book GR–10 (Teacher Manual) 69

HF_Acme_ENG_TM_G10.indd 69 5/22/2021 5:58:43 PM


II. Black Aeroplane
Passage Based Questions
1. (a) No, the sky didn’t remain clear the whole night as the narrator saw black clouds
later.
(b) The narrator was happy because he was going to England to spend the holidays
with his family.
2. (a) The narrator was flying to England to spend his holidays with his family at home.
He wanted to have breakfast with his family.
(b) The narrator said ‘It was an easy flight.’ Soon, he encountered a storm which put
his life in danger. Therefore, the statement turned ironic.
3. (a) The narrator’s aeroplane was caught amidst a storm of black clouds. Therefore,
he couldn’t see outside the aeroplane. His aeroplane was jumping and twisting
in the clouds.
(b) The narrator couldn’t believe his eyes when he saw a black aeroplane which had
no lights on its wings. He saw the pilot waving at and signalling him to follow him
to get out of the storm.
4. (a) The narrator got no answer from the Paris Control Room as his instruments had
stopped working. He was not able to reach them through his plane.
(b) He saw an aeroplane with no lights on its wings, flying next to him. He could see
the pilot’s face, who signalled him to follow him.
5. (a) (ii) Pilot of black aeroplane
(b) (iv) Black aeroplane (c) d) F - (ii), (iii), (iv) and O - (i)
(d) (i) He followed the plane like an obedient child.
(e) (i) Strange
6. (a) d) (iv), (vi) (b) (i) Seeing two straight lines of lights
(c) (i) Black aeroplane (d) (iii) The pilot of black aeroplane
(e) d) Option (iv)

Short Answer Type Questions


1. The narrator was eager to meet his family. When he realised he did not have enough
fuel to fly around the black clouds, he didn’t go back to Paris. He took the risk of flying
straight into the storm because he desired to get home and have a good breakfast
with his family.
2. The narrator had faced a horrific experience flying the Dakota plane in the storm.
He had quite an adventure and after landing, he was not sorry to walk away from
the old Dakota. He had landed the plane safely.
3. Inside the clouds, everything was suddenly black. It was impossible to see anything
outside the aeroplane. The old aeroplane jumped and twisted in the air. The compass
and the other instruments were dead.
4. The narrator was thinking of having a satisfactory breakfast at home with his family
when he checked the map and the compass after taking off. He believed everything
was going well.
5. The black aeroplane and its pilot disappeared after coming out of the storm. The
narrator was told by the woman at the control centre that there was no other plane
flying that night except the narrator’s.
70 ACME English Practice Book GR–10 (Teacher Manual)

HF_Acme_ENG_TM_G10.indd 70 5/22/2021 5:58:43 PM


6. When the narrator started his flight, the weather was clear, the moon was up in the
east, the stars were shining and there wasn’t any cloud in the sky.
7. A pilot in the black aeroplane helped the narrator to land safely amidst the storm.
Unfortunately, the pilot and the plane disappeared. The narrator wanted to thank
the pilot therefore, he enquired about the pilot at the control room. The lady looked
at him strangely and informed him that there was no other plane flying on the radar
except his plane.
8. The narrator was compelled to follow another aeroplane because he had lost his way
in the storm and was unable to see anything. The pilot of another aeroplane was
helping him to get out of the storm and land safely.

Long Answer Type Questions


1. The narrator took the risk of flying straight into the storm clouds. Inside the clouds
everything suddenly went back. He was not able to see anything outside the plane.
His plane was swirling and twisting among the clouds. The compass was dead. Other
instruments also stopped functioning. The narrator was lost in the storm clouds.
Suddenly, a strange black plane appeared near him. The pilot of the black plane
waved his hand and signalled him to follow him. The narrator followed him like an
obedient child. He helped the narrator to land safely. The black aeroplane rescued
the Dakota pilot.
2. The narrator was rescued by a pilot of the black plane amidst the storm. He landed
safely with the pilot’s help. The narrator was thankful to the pilot for saving his life
and wanted to meet him and show his gratitude. But to his surprise, the woman in
the control room told him that his was the only aeroplane on the radar that night.
The narrator was perplexed.
I think the black aeroplane never existed. The mysterious ending of the story compels
us to think that perhaps it was the narrator’s imagination. But if this was his imagination
then how could he land safely without a compass and other instruments! I think it
was the narrator’s intuitive power which created an imaginary vision of plane and
pilot to help him in landing safely.

Chapter 4 From the Diary of Anne Frank

ANSWER KEY

Passage Based Questions


1. (a) The note the speaker made as a reminder for the essay she had to write caught
her eye. The note was the topic ‘A Chatterbox’ on which she had to write an essay
as a punishment for talking in the class.
(b) The speaker began to think about the subject of the ‘note’ as she wanted to present
concrete arguments in support of being a chatterbox.
2. (a) The speaker wanted to come up with convincing arguments as she thought anyone
could fill pages for the sake of writing. She wanted to present herself better.

Learnwell
ACME Math–III
ENGLISH Practice Book GR–10 (Teacher Manual) 71

HF_Acme_ENG_TM_G10.indd 71 5/22/2021 5:58:44 PM


(b) The subject of the argument is the necessity of talking. These lines tell us that the
speaker was talkative and wanted to give a reasonable logic for being so.
3. (a) Mr Keesing had been giving Anne essays to justify her talkative nature. That’s
why she felt that Mr Keesing was trying to play a joke on her.
(b) Anne wrote a poem about a mother duck and a father swan with three baby
ducklings. In the story the father beats the three ducklings to death because they
quack a lot. Mr Keesing understood the joke.
4. (a) Anne wrote a poem about a mother duck and a father swan with three baby
ducklings. The father had beaten the ducklings to death because they quacked
too much. Mr Keesing understood the joke and she was allowed to talk in the
class.
(b) Mr Keesing asked Anne to write an essay on the subject ‘A Chatterbox’. On the
second occasion, she was asked to write an essay on the subject ‘An Incorrigible
Chatterbox’ as she talked a lot in class.
5. (a) (ii) 13 (b) (iii) No one would be interested in them.
(c) b) F - (i) and O - (ii), (iii), (iv)
(d) (iii) Writing a diary (e) c) Option (iii)
6. (a) (i) Anne Frank (b) (i) It doesn’t complain.
(c) (iv) A diary (d) (iv) A real friend (e) c) Option (iii)

Short Answer Type Questions


1. Unlike other writers, Anne Frank decided to make her diary her friend. Kitty was the
name given by Anne to her diary.
2. Anne provided a brief sketch of her life in her diary as she considered it her friend.
She introduced herself and her family to the diary.
3. Anne wrote that she had loving parents and aunts and a sixteen-year-old sister. They
are a loving family but she couldn’t confide in them.
4. Anne stayed with her grandmother for a while when her parents were away in
Holland. They shared a warm relationship and loved each other deeply. When her
grandmother died, Anne felt sad. She missed her immensely.
5. Just before the announcement of results, the whole class was shaking with fear. Half
of the class was making bets about the result. The fate of the students was in the
teachers’ hands.
6. It is true that the story “From the Diary of Anne Frank” describes the teacher-student
relationship in a positive light. Mr Keesing asked Anne to write essays as a punishment
because he wanted her to focus on studies. They both tried to make the other
understand their point of view using humour. They both accepted each other the
way they are ultimately. This depicts a very healthy relationship between the teacher
and the student.
7. Anne shared cordial relationship with her teachers. Anne and her childhood
headmistress Mrs Kuperus had a tearful farewell. Even though Mr Keesing gave her
assignments as punishment, she never disrespected him. She took it in a positive
manner and put forward convincing arguments. They both found humour in the
situation ultimately.
8. A chatter box is a person who likes to chatter or talk a lot. Mr Keesing gave Anne an
essay entitled ‘A Chatterbox’ to be written as a punishment because she used to talk
a lot in class.

72 ACME English Practice Book GR–10 (Teacher Manual)

HF_Acme_ENG_TM_G10.indd 72 5/22/2021 5:58:44 PM


Long Answer Type Questions
1. When Mr Keesing punished Anne for her talkativeness repeatedly by asking her to
write essays, she thought that he was playing a joke on her. She took his assignments
as punishment but put forward her opinion honestly. But in her last assignment, she
composed a poem on the topic ‘The Incorrigible Chatterbox’ and gave a message
through it to the teacher. Mr Keesing, who was punishing Anne repeatedly, got so
impressed by her poem that he decided not to punish her further. He took the poem
in the right way and understood the joke. After this incident he never assigned any
extra homework to Anne for talking in the class. On the contrary, he himself started
having fun and making jokes in class. He also showed her poem to other classes.
This instance shows that teachers are unpredictable creatures.
2. Anne considered Kitty her true friend in which she could confide her inner most
feelings. Anne received the diary on her thirteenth birthday. She decided to make it
her friend and confide her feelings in it because she did not have any confidant. She
believed that it would be a great source of comfort and support to pour her heart out
in the diary. At times, she felt lonely and longed for companionship. Though Anne
had many friends and admirers because of her playful nature, she never had any real
friend to be able to open up to them.
Anne and her friends talked only about trivial things. Therefore, Anne’s found comfort
in her diary which helped her through insecure, isolated and fearful time in the hiding.

Chapter 5 The Hundred Dresses-I

ANSWER KEY

Passage Based Questions


1. (a) Wanda didn’t have any friends because she was quiet and rarely said anything.
She sat in the last row where noisy boy sat. She was also different from other
students as she belonged to a different nationality and had a different name.
(b) Wanda always wore a faded blue dress because she was poor and had no other
dress.
2. (a) Wanda did not have any friend. So, to impress other girls of the school Wanda
used to lie that she had a hundred dresses. Also the other girls used to tease her
about her blue faded dress. So to counter them she told this lie.
(b) Peggy thought Wanda was too dumb to say that she had a hundred dresses
because everybody knew she was poor.
3. (a) ‘She’ refers to Maddie. She wanted to write to Peggy that whatever they were
doing to Wanda was not fair and they should stop doing it.
(b) Maddie did not have the courage to write the ‘note’ to Peggy because she was
scared that Peggy might divert her bullying to her.
4. (a) It was the result day of the drawing competition and light shower was falling.
Peggy and Maddie ran quickly to school without waiting for Wanda.
(b) Peggy and Maddie could not afford to get late as it was an important day because
the result of the drawing competition was to be announced.

Learnwell
ACME Math–III
ENGLISH Practice Book GR–10 (Teacher Manual) 73

HF_Acme_ENG_TM_G10.indd 73 5/22/2021 5:58:44 PM


5. (a) (ii) Teacher (b) (iii) Drawing competition
(c) (i) Peggy could draw beautifully.
(d) a) option (i) and (ii) (e) b) Option (ii)
6. (a) (iii) Wanda (b) (iv) For students’ exhibition
(c) (ii) Students of the class (d) b) F - (i), (iii) and O -(ii), (iv)
(e) c) Option (iii)

Short Answer Type Questions


1. Peggy belonged to a wealthy family. She owned many pretty dresses. According to
Maddie, she was kind-hearted girl who protected the young children from bullies.
But she didn’t treat Wanda well. She made fun of her because she thought her to be
dumb.
2. Peggy used to sit in the front of the class as she was a pretty and the most popular
girl in her school.
3. Peggy came from a wealthy family and was the most popular girl in class. On the
contrary, Wanda was a poor girl who lived in Boggins Heights and always wore a
faded blue dress which looked tidy but was never ironed properly.
4. No, Wanda did not have any friends in the school. She came to school alone, was
a very reserved girl who sat in the last row next to noisy boys. Peggy would always
tease her for lying about owning a hundred dresses.
5. Wanda lived at Boggins Heights. The place seemed to be far away from her school.
It was a poor locality. The area had a lot of dry mud and Wanda’s caked shoes were
a proof of it.
6. Wanda lived at Boggins Heights which was not a rich locality. Her shoes were often
caked with mud because the area had a lot of dry mud. She wore the same faded
blue dress to school. With these examples, we can infer that Wanda was a poor girl.
7. Peggy did not feel guilty for teasing Wanda as she thought Wanda was dumb to lie
about her hundred dresses and sixty pairs of shoes when everybody knew she was
poor.
8. Peggy and Maddie would wait to tease Wanda at school. Peggy and the other girls
would surround her and ask her about the number of dresses she had in her closet.
Wanda would reply that there were a hundred. They would exclaim incredulously,
break into a peal of laughter and disperse.

Long Answer Type Questions


1. Peggy and Maddie would wait to tease Wanda at school. Peggy and the other girls
would surround her and ask her about the number of dresses she had in her closet.
Wanda would reply that there were a hundred. They would exclaim incredulously,
break into a peal of laughter and disperse. It was not innocent action on the part of
Peggy. Teasing someone for fun even though we know that the other person is lying
is not an appropriate behaviour. Even though she used to protect animals and young
children from being mistreated, she was not capable of identifying misdeed of teasing
and how hurtful it was.
2. Everyone believed that Peggy would win the drawing and colouring competition.
When Miss Mason announced Wanda as the deserving winner because she drew a
hundred dresses, all unique, everyone was surprised. Despite being poor, ignored

74 ACME English Practice Book GR–10 (Teacher Manual)

HF_Acme_ENG_TM_G10.indd 74 5/22/2021 5:58:44 PM


and having no proper resources, Wanda displayed extraordinary talent. Everyone
applauded and gathered around the room to have a look at the drawing. Maddie
and Peggy saw the blue dress and the green dress Wanda had talked about. They
were surprised to see those dresses. Her talent was appreciated by the entire class.

Chapter 6 The Hundred Dresses-II

ANSWER KEY

Passage Based Questions


1. (a) Peggy became very sad. She felt ashamed about her behaviour towards Wanda
and wanted to apologise to her.
(b) According to Peggy, Wanda was probably getting good ideas for her dresses when
she teased Wanda about her dresses. It might be true that Wanda got inspired by
the regular teasing and vented out creatively by drawing the hundred dresses.
2. (a) When Maddie did not find Wanda at her home, she was disappointed. She could
not sleep the whole night because she was guilty of being a part of Wanda’s
teasing. She wanted to apologise for her behaviour.
(b) The important conclusion Maddie reached was that she would not tolerate bullying
of any sort. She would speak up against anyone who picks up on someone for
their looks or funny name.
3. (a) Maddie finally decided that she would speak against anyone picking on someone
for his/her looks or funny name. She would not witness it silently. She would never
make anyone unhappy.
(b) These lines tell that Maddie was an empathetic girl. She was not cruel and was
regretful for having hurt Wanda.
4. (a) Peggy was a rich girl and the most popular girl in school. She protected young
children and animals from being bullied but made fun of Wanda. Maddie was a
poor girl who wore Peggy’s hand-me-down clothes. She did not like Peggy teasing
Wanda, but she could not stand up against it.
(b) Maddie would try to protect Wanda if someone was trying to tease her and would
stand up against the teasing.
5. (a) (iii) Wanda painted them in her drawings.
(b) (i) She missed Wanda. (c) (i) Remorseful (d) d) (iv) and (v)
(e) c) Option (iii)
6. (a) (iii) Wanda liked them. (b) a) (i), (ii), (v)
(c) (ii) She would stand silently.
(d) (iv) Wanda’s drawings (e) c) Option (iii)

Short Answer Type Questions


1. No, Maddie wasn’t a bad person. She couldn’t stand up against Peggy’s bullying
because she was afraid that Peggy would divert her bullying towards her. Later she
decided to stand for people being teased.

Learnwell
ACME Math–III
ENGLISH Practice Book GR–10 (Teacher Manual) 75

HF_Acme_ENG_TM_G10.indd 75 5/22/2021 5:58:44 PM


2. No, Peggy didn’t accept that she mistreated Wanda. She believed that she never
called Wanda a foreigner or made fun of her name. She never thought Wanda had
even the sense to know that they were making fun of her. She thought Wanda was
too dumb.
3. Miss Mason received a letter from Jon Petronski, Wanda’s father. In that letter,
Mr Petronski had written that they have left for good due to incessant bullying. Thus,
Peggy and Maddie came to know that the Petronskis had left.
4. Wanda’s house is compared to her blue dress because both looked shabby but were
clean. Also her house was in a poor neighbourhood and Wanda’s dress reflected that
she was poor.
5. Maddie was right when she thought that her silence was as bad as Peggy’s teasing.
Maddie’s silence encouraged Peggy’s teasing. She realised that the person who silently
witnesses the offence is equally responsible for the offence as the offender.
6. In the letter, Wanda asked the girls of Room Number Thirteen to keep those ‘hundred
dresses’. She asked the teacher to give the drawing of the green dress to Peggy and
drawing of the blue dress to Maddie.
7. After listening to the letter, both Maddie and Peggy became very sad. They both
felt ashamed for bullying Wanda and wanted to apologise to her. Maddie felt sick in
her stomach. She couldn’t concentrate in her study. Maddie was more regretful than
Peggy.
8. When Maddie carefully looked at the drawing she realised that the blue dress had a
face that looked like hers. The face on the green dress resembled Peggy’s. This is when
the girls realised that Wanda had liked them in spite of their teasing. She decided to
go to Boggins Heights to apologise to Wanda.

Long Answer Type Questions


1. Maddie was guilty of being a silent witness to Wanda’s teasing. She wanted to apologise
to Wanda and went to Boggins Heights. When she didn’t find Wanda there, Maddie
was disappointed. She could not concentrate in her studies nor could she sleep that
night. Her mind was engrossed by the drawings of Wanda’s hundred dresses, her
faded blue dress and her house. She contemplated the whole night and took an
important decision. She concluded that she would never stand by and support any
teasing. She wouldn’t stand anyone’s insult and oppose bullying. She would never
let anybody else feel inferior again.
2. Peggy tried to justify her actions by calling Wanda dumb. She thought that Wanda did
not have sense enough to sense that they were making fun of her. But her interpretation
was incorrect. Wanda’s letter to school revealed Wanda’s feelings which made Peggy
change her opinion about Wanda’s. When Wanda missed the school, Peggy noted her
absence. When there wasn’t any reply to their letter, they felt that Wanda was angry
with them for their ill treatment. She even decided to go to Boggins Heights along
with Maddie to apologise to Wanda. Peggy realised later that Wanda was sensitive
and aware of all the certain things which made Peggy miss her and feel sorry for her.

76 ACME English Practice Book GR–10 (Teacher Manual)

HF_Acme_ENG_TM_G10.indd 76 5/28/2021 3:50:36 PM


Chapter 7 Glimpses of India

ANSWER KEY

I. A Baker from Goa


Passage Based Questions
1. (a) Those ‘eaters of loaves’ were the Portuguese Goan people. They might have
vanished due to the change in the demographics of the area.
(b) Goan village bakers were the ‘makers’. The ‘makers’ are still there to bake loaves.
They have adapted themselves to the new circumstances and become an important
part of everyday life.
2. (a) The baker uses the traditional baker’s bamboo to announce his arrival in the village
in the morning. It is used to attract people to buy the different kinds of breads he
was carrying.
(b) Bread-making is the ‘family profession’ referred to in this extract. This family
profession has a lot of significance in the area as no Goan festival is complete
without bakery products.
3. (a) Children were woken up by the jingling thud of the bamboo. No, they didn’t mind
being woken up as they longed for bread-bangles.
(b) They ran to meet and greet the baker to buy bread-bangles.
4. (a) A Goan festival is incomplete without bakery products—be it marriages,
engagements or any other ceremony. Traditional sweet bread, known as ‘bol’ is
given with marriage gifts. At Christmas ‘bolinhas’ and cakes are a must.
(b) The baker’s furnace in the village was very important as bakery products were
quite popular and essentials for all festivals and celebrations.
5. (a) (i) a single long frock reaching to knees (b) (iii) kabai
(c) d) (ii), (iii), (iv) (d) (iv) One who wears a half pant
(e) a) Option (i)
6. (a) (iv) He collected his bills. (b) a) F - (i), (ii) and O - (iii), (iv)
(c) Prosperous (d) (i) Their plump physique (e) (ii) Plump

Short Answer Type Questions


1. The baker wore a single piece of long frock reaching down the knees known as kabai.
It was a single piece of long frock reaching down the knees. Later during the writer’s
childhood, baker started wearing a shirt and trousers which were shorter than full
length trousers and longer than half pants.
2. The children were mildly rebuked because as soon as the baker arrived, they hurried
outside to peep in the basket. They were drawn to the smell of the bread. This made
the elders angry.
3. The jingling thud of the pader and his musical entry in the morning would wake
children up from their sleep. They would excitedly run to greet him and peep into
his basket. They would forget to brush their teeth or wash their mouths.

Learnwell
ACME Math–III
ENGLISH Practice Book GR–10 (Teacher Manual) 77

HF_Acme_ENG_TM_G10.indd 77 5/22/2021 5:58:45 PM


4. The baker usually collected his bills at the end of the month. He recorded his monthly
accounts on some wall with pencil.
5. Baking was a profitable profession as any Goan festival was incomplete without bread.
This profession helped bakers earn well. They kept servants and no one starved. Their
round bellies were proof of their prosperity.
6. The baker would come at least twice a day for selling his bread. He announced his
arrival with the help of the thud and jingle of the specially made bamboo staff. All
the children would wake up and surround him hoping to get bread. They were mildly
rebuked to move aside so that the elders could buy the bread.
7. Bread is an essential commodity in any Goan function. Be it marriages, engagements
or any other ceremony, breads hold an important place. Traditional sweet bread,
known as ‘bol’ is given with marriage gifts. At Christmas ‘bolinhas’ and cakes are a
must.
8. The bakers in earlier times wore half pants and were known as pader. So, anyone
who wears a half-pant which reaches just below the knees invites the comment that
“he is dressed like a pader”.

Long Answer Type Questions


1. Bread baking is an old and a prosperous profession. There are customs and traditions
related to it, which makes the Goan elders nostalgic about the Portuguese and their
loaves of bread. The bakers still have the mixtures, moulders and time-tested furnaces
to bake loaves. They would announce their arrival by the jingling thud of bamboo.
The sweet aroma of the bread would attract children. In olden day, the bakers used to
wear a particular dress called ‘Kabai’. It was a single-piece long frock reaching down
to the knees. Bread was an essential commodity in Goan functions which made the
bakers a prosperous community. These old customs and traditions of bakery are still
prevalent and trace their lineage to the Portuguese.
2. During the writer’s childhood, traditional bakers were quite common and popular
in the village. They used to visit twice and wake the children up with their jingling
thud of the bamboo. The children loved the aroma of the bread and were fascinated
with bread bangles. They would rush outside to peep inside the basket of the bakers,
without even brushing their teeth or washing their face. The writer remembered those
days when he was hushed and pushed by the elders for interrupting their daily affairs
with the bakers. He recalled the days when bakers enjoyed respect and love of the
people because they were essential part of the Goan festival. They lived a prosperous
life as they were indispensable part of the community.

II. Coorg
Passage Based Questions
1. (a) Coorg is referred to as ‘a piece of heaven’. It is situated midway between Mysore
and Mangalore.
(b) The place is famous for evergreen forests, spices and coffee plantations.
2. (a) A part of Alexander’s army settled in Coorg because returning to their places
seemed impractical. They settled here and married among the locals.

78 ACME English Practice Book GR–10 (Teacher Manual)

HF_Acme_ENG_TM_G10.indd 78 5/22/2021 5:58:45 PM


(b) The similarity of the martial traditions, marriage and religious rites between the
people of Coorg and the Greek traditions supports the Greek theory about the
origins of the Coorgi people.
3. (a) Coorgi houses have a tradition of hospitality. They feel proud to share the
courageous tales of their sons and fathers.
(b) Kodavus are the only people in India permitted to carry firearms without a licence.
4. (a) The author is talking about Coorg. A panoramic view is seen from the Brahmagiri
Hills.
(b) One can reach Nisargadhama Island by a walk across the rope bridge. It is a
sixty-four-acre island.
5. (a) (iv) Langur (b) b) (iii), (vi)
(c) (ii) With river rafting, canoeing, rock climbing
(d) (iii) trekkers (e) a) Option (i)
6. (a) (i) Heavy rain keeps the visitors away.
(b) (i) Season of tourism (c) (iv) Perfect with some showers
(d) b) option (ii) and (iii) (e) (i) Tiring

Short Answer Type Questions


1. The people of Coorg are famous for their valour and fierce attitude. The Coorg
Regiment is one of the most decorated in the Indian Army. Coorgis take immense
pride in recounting the tales of valour of their sons and fathers.
2. Squirrels and langurs throw partial eaten fruit into the water to enjoy the splash and
the ripple effect of the water. They do this to satisfy their instincts of enjoyment and
mischief.
3. Coorg is situated midway between Mysore and the coastal town of Mangalore. It
seems to be a piece of heaven fallen from the kingdom of God. Rolling hills and rich
variety of flora and fauna make Coorg a great attraction for tourists.
4. Coorg is home to a variety of species of birds and animals. One can found Mahaseer,
a large freshwater fish, in abundance here. Kingfishers dive for their catch, while
squirrels and langurs drop partially eaten fruit to enjoy ripple effects in clean water.
Elephants are bathed and scrubbed in the river by the mahout.
5. It is believed that Kodavu people are of Greek or Arabic origin. According to one
story, some of Alexander’s armymen settled here, marrying amongst the locals. The
martial traditions, marriage and religious rites are similar to the Greeks. Another story
draws support from the embroidered waist belt worn by the Kodavus that resembles
that of Arabs.
6. According to the author, from the Brahmagiri Hills one can enjoy a panoramic view
of the entire landscape of Coorg. There is a sixty-four-acre island of Nisargadhama
nearby. It can be reached through a rope bridge. Therefore, a climb to the Brahmagiri
Hills is a must.
7. The months of September and March are considered to be the season of happiness
and tourism. This is the perfect time to visit Coorg as the weather is great and various
water sports like rafting, canoeing, rappelling rock climbing, etc. can also be enjoyed.
8. The Coorgis are fiercely independent and courageous. Their houses have a tradition
of hospitality. The Coorg Regiment is one of the most decorated in the Indian Army.
They proudly share the courageous tales of their sons and fathers.

Learnwell
ACME Math–III
ENGLISH Practice Book GR–10 (Teacher Manual) 79

HF_Acme_ENG_TM_G10.indd 79 5/22/2021 5:58:45 PM


Long Answer Type Questions
1. The people of Coorg are courageous, hospitable and fiercely independent. They are
assumed to be of Greek or Arabic descent. According to the Greek theory, Alexander’s
army had to settle in Coorg as they found it difficult to return to their places. They
married among the locals and settled there. The Coorgis are hospitable and have
many tales of bravery. They do not shy away in recounting the brave tales of their
sons and fathers. The Coorg Regiment is one of the most decorated in the Indian
army and they can carry firearms without any license. So, the origin of Coorgis can
be said to be responsible for their valour.
2. The author called Coorg ‘a piece of heaven’ for many reasons. It is situated in
Karnataka midway between Mysore and the coastal town of Mangalore. Its beauty
forces us to believe that it must have drifted from the kingdom of God. This is a land of
rolling hills inhabited by martial men, beautiful women and a variety of wild animals.
It is home to great variety of plants and many different species of birds and animals.
The river Kaveri flows through Coorg. Brahmagiri Hills provide panoramic view of
the entire misty landscape of Coorg. One can visit the place between September and
March to enjoy the view with some showers.

III. Tea from Assam


Passage Based Questions
1. (a) The awesome scenery outside diverted Rajvir’s mind from reading detective
stories.
(b) Rajvir was amazed to see greenery everywhere. He saw the green paddy fields
first and then the green tea bushes that were stretched as far as the eyes could see.
2. (a) Though there are many legends, no one really knows who discovered tea.
(b) The emperor always boiled water before drinking it. Tea leaves gave the water ‘a
delicious flavour’.
3. (a) Bodhidharma, an ancient Buddhist ascetic cut off his eyelids so that he didn’t feel
sleepy during meditations.
(b) Ten tea plants grew out of the eyelids. The leaves of these plants when put in hot
water and drunk banished sleep.
4. (a) The building was called ugly because smoke was billowing out of its tall chimneys.
(b) Rajvir cried excitedly on seeing a tea garden. Pranjol did not share the same
excitement because he was born and brought up on a plantation. The garden
and greenery were a common sight for him.
5. (a) (i) Pranjol and Rajvir (b) (iv) To go to Pranjol’s home in Assam
(c) (iii) School hostel (d) (ii) Pranjol’s home (e) c) Option (iii)
6. (a) (ii) Rajvir
(b) (i) Rajvir seemed to have done a lot of research before coming.
(c) b) (i), (ii), (v) (d) (iii) Tea yield periods (e) c) Option (iii)

Short Answer Type Questions


1. There is a Chinese story related to tea. A Chinese emperor always drank boiled water.
Once while boiling the water some leaves of the burning twigs fell into the water. The
water tasted delicious. Those leaves were tea-leaves.

80 ACME English Practice Book GR–10 (Teacher Manual)

HF_Acme_ENG_TM_G10.indd 80 5/22/2021 5:58:45 PM


2. Tea was introduced to Europe in the sixteenth century. It was considered to have
medicinal properties and used for treatment.
3. According to the Indian legend, tea was discovered by Bodhidharma, a Buddhist
ascetic. Once when he cut off his eyelids to get rid of sleep during meditation, ten
tea plants grew out of it. The leaves of these plants when put in hot water and drunk
were believed to keep sleep away.
4. Pranjol and Rajvir reached Dhekiabari Tea Estate by travelling by train. On their way,
majestic green forests, paddy fields and tea bushes greeted them.
5. Dhekiabari Tea Estate is a tea estate situated in the Upper Assam. On both sides of
the gravel-road were acre upon acre of tea bushes, all neatly pruned to the same
height. Groups of tea-pluckers, with bamboo baskets on their backs, wearing plastic
aprons, were plucking the newly sprouted leaves.
6. Yes, Rajvir is a knowledgeable child. He had researched about the sprouting period
beforehand. He told Pranjol’s father about the second sprouting period which yields
the best tea. Pranjol’s father was surprised by Rajvir’s inquisitiveness and knowledge.
7. According to a Chinese legend, tea was discovered when a few leaves of a twig burning
under the pot fell into boiling water. The Indian legend says that ten tea plants grew
out of a Buddhist ascetic, Bodhidharma’s eyelids, when he cut them off to get rid of
sleep. The leaves of these plants were used for making tea. In Europe, tea came in
the sixteenth and was drunk for medicinal purposes.
8. Rajvir saw large fields of tea bushes as far as eyes could see. He observed that almost
all of them were neatly pruned to the same height. Tea-pluckers were carrying bamboo
baskets on their backs for keeping the leaves newly sprouted leaves after plucking
them.

Long Answer Type Questions


1. There are two popular legends related to the origin of tea – Chinese and Indian.
According to the Chinese legend, a Chinese emperor always boiled water before
drinking it. One day while he was boiling water, some leaves from the twigs burning
under the pot fell into it. On drinking, he found the taste to be delicious. It is said that
they were tea leaves.
According to the Indian legend, tea leaves grew out of an ancient Buddhist ascetic
named Bodhidharma’s eyelids. It is believed that Bodhidharma cut off his eyelids to
avoid sleep during meditation. Ten tea plants grew out of his eyelids. The leaves of
these plants when put in hot water and drunk banished sleep.
2. Rajvir was a curious and knowledgeable child. His urge for knowledge brought him
to Assam. He had never visited a tea estate ever before in his life. His inquisitiveness
made him research about the tea garden and the sprouting season beforehand. He
displayed his knowledge of tea garden before Pranjol and his father. He had been
reading about it a lot. He also narrated two legends about the discovery of tea in
Indian and China. Rajvir shared his knowledge about the second-flush or sprouting
period when he saw a tractor pulling a trailer load of tea leaves. He also told Pranjol’s
father about the yielding period that continues from May and to July. Pranjol’s father
was impressed and astonished by Rajvir’s deep knowledge about tea-leaves and its
sprouting season.

Learnwell
ACME Math–III
ENGLISH Practice Book GR–10 (Teacher Manual) 81

HF_Acme_ENG_TM_G10.indd 81 5/22/2021 5:58:45 PM


Chapter 8 Mijbil the Otter

ANSWER KEY

Passage Based Questions


1. (a) Mij spent most of his time in play. He spent hours shuffling the rubber ball round
the room, like a four-footed soccer player using all four feet to dribble the ball,
and he could also throw it, with a powerful flick of the neck, to a surprising height
and distance.
(b) Mij would lie on his back rolling two or more of marbles up and down his wide,
flat belly without ever dropping one to the floor.
2. (a) As the British Airlines did not allow pets, the narrator had to book a ticket on a
different airline. The airline directed the narrator to pack Mijbil in a box of not
more than 18 inches square. He also had to change two flights to get back to
London with Mijbil.
(b) The narrator put Mij into the box an hour before the flight so that he would become
accustomed to it and not create a scene at the airport or in the plane.
3. (a) The woman stood up on her seat screaming that there was a rat. The otter hid
himself beneath the legs of an Indian passenger.
(b) The air hostess asked the narrator to resume his seat. She assured him that she
would find the animal and bring it to him.
4. (a) Mij passed his time in London by playing for hours with ping-pong balls, marbles,
rubber fruit, and a terrapin shell.
(b) Mij invented a game with ping-pong ball. He would put the ball at one end of a
sloping lid and then grab it as it ran to the other end.
5. (a) (ii) Like a dog (b) (iv) In the London streets
(c) (iii) Certain compulsive habits
(d) b) F - (i), (iii) and O -(ii), (iv) (e) d) Option (iv)
6. (a) (iii) People’s guesses about otters
(b) (iv) Otters were not commonly kept as pet.
(c) c) (ii), (iii), (iv) (d) (iv) Mustellines (e) a) Option (i)

Short Answer Type Questions


1. When the narrator saw the otter for the first time, he couldn’t recognise him as otter.
He thought him to be a very small medievally-conceived dragon. He found him very
soft looking as he was covered with soft velvet fur, like that of a chocolate-brown
mole.
2. Mijbil the otter was a small creature that looked like a small dragon. From head to
the tip of his tail, he was coated with symmetrical pointed scales of mud armor. He
had a soft velvet fur like that of a chocolate-brown mole.
3. On the second night, Mijbil slipped on to the author’s bed. He remained asleep in
the crook of his knees until the servant brought tea the next morning.

82 ACME English Practice Book GR–10 (Teacher Manual)

HF_Acme_ENG_TM_G10.indd 82 5/22/2021 5:58:45 PM


4. Mijbil went wild with joy on seeing water in the bathroom. He jumped up and down
the length of the bathtub, plunging and rolling in it.
5. When the narrator entered his bedroom, he observed two Arab men sitting with a
sack which twisted itself again and again. They handed him a note sent by his friend
which stated that the otter was a gift from him.
6. Mijbil was an intelligent animal. He had invented a game with a ping-pong ball
and narrator’s old bag. He invented a game with the ping pong and the narrator’s
damaged suitcase whose lid would look like a slope on closing. He would place the
ball at the high end and it would run down the length of the suitcase. Mij would run
and grab it at the opposite end. He could also turn on the tap with ease.
7. Most of the times, Mijbil would easily turn the tap and enjoy the full flow. But
sometimes he would tighten the tap by twisting it in the wrong direction. He would
be disappointed to see no flow.
8. Mijbil would spend hours shuffling a rubber ball round the room like a four-footed
soccer player. He used all four feet to dribble the ball, and he threw it with a powerful
flick of the neck.

Long Answer Type Questions


1. Maxwell visited Basra along with his friend to the Consulate-General (of Iraq) to
collect and answer their mail from Europe. Though his friend received the mail on
time, his hadn’t arrived. He cabled to England. He waited for three days but nothing
happened. He then tried to telephone. The call needed to be booked twenty-four
hours in advance. On the first day, the line was out of order. On the second day the
exchange was closed for a religious holiday. On the third day too, there was another
breakdown. By then his friends had already left. He waited for five days until he
received his mail.
2. Mijbil was a fun-loving and intelligent animal. Once when Maxwell took him to the
bathroom, he squealed with joy. He loved water and stayed in the bathroom for a
long time. Maxwell soon realised Mijbil’s love for water. Mijbil would jump up and
down in the water. He would splash and spread water everywhere. The next day,
Mijbil sneaked into the bathroom without Maxwell’s knowledge. He struggled with
the tap till it turned on and water flowed in the tub. Sometimes he would turn the
tap in the wrong direction tightening the grip. That would disappoint him. His love
for water was quite evident from his actions.

Chapter 9 Madam Rides the Bus

ANSWER KEY

Passage Based Questions


1. (a) Valli saved every penny by resisting every temptation to buy peppermints, toys,
balloons, etc. With this conviction she managed to save sixty paise.
(b) Looking from the doorway of her house or sometimes even venturing out into
the village became Valli’s excursion.

Learnwell
ACME Math–III
ENGLISH Practice Book GR–10 (Teacher Manual) 83

HF_Acme_ENG_TM_G10.indd 83 5/22/2021 5:58:45 PM


2. (a) Valli clapped her hands in delight seeing a young cow, tail high in the air, running
very fast, right in the middle of the road and in front of the bus.
(b) The honking frightened the animal. The more the driver honked, the more
frightened the animal became and the faster it galloped in front of the bus.
3. (a) Valli could see a speck of a train in the distance, growing bigger and bigger as it
drew near the bus. When the bus came to a railroad crossing, a train rushed past
the crossing gate with tremendous roar and rattle, shaking the bus.
(b) Valli saw big, bright-looking shops with glittering displays of clothes in the
thoroughfare. She was surprised at the big crowd that had gathered there.
4. (a) Valli felt excited during her return journey. She wasn’t bored to the slightest degree
and greeted everything with the same excitement she’d felt the first time.
(b) Valli saw a young cow lying dead by the roadside. It had perhaps been struck by
some fast-moving vehicle.
5. (a) (ii) The temporariness of life
(b) (ii) A young cow (c) (i) She saw a young cow lying dead.
(d) (ii) It was transformed into a lifeless one.
(e) c) Option (iii)
6. (a) (iv) She was immature. (b) (iv) She smiled.
(c) (i) Valli poked her nose in their conversation.
(d) c) Option (iii) (e) b) (iii), (iv)

Short Answer Type Questions


1. The woman sitting beside Valli was chewing betel nut and its juice was almost spilling
about her lips. This sight disgusted Valli and she developed hatred towards the woman.
2. Valli calculated the bus fare and saved every penny that came her way. She had to
resist every temptation of buying peppermints, toys, balloons, etc. She also had to
make the roundabout journey during the time her mother took afternoon nap.
3. Valli noticed that the bus made a halt at a railroad crossing. She could see a tiny dot
approaching towards the bus which grew larger. The train passed with a tremendous
roar. She could feel the bus shaking.
4. On taking a turn, the bus reached a thoroughfare. It was a well laid out shopping
street with big and brightly lit shops on the road that displayed different articles for
sale. Valli was mesmerised by the glittering displays of clothes and other merchandise.
She was surprised by the crowd.
5. No, Valli didn’t feel uncomfortable while riding in the bus. Valli exuded an air about
herself. She was commanding and did not respond when the conductor tried to help.
6. No, Valli didn’t face any problem when she reached home after the bus ride. When
she returned home, her mother was awake and talking to one of her aunts. They
didn’t even notice that she was missing for such a long time.
7. From the window of the bus, Valli saw the palm trees standing along the bank of a
canal with mountains and the blue sky covering the background. On the opposite
side, the vast green fields soothed her sight.
8. No, Valli didn’t like to be called a child. She behaved like a lady and refused any
help from the conductor. The conductor called her ‘Madam’ because she acted like
a grown-up.

84 ACME English Practice Book GR–10 (Teacher Manual)

HF_Acme_ENG_TM_G10.indd 84 5/22/2021 5:58:46 PM


Long Answer Type Questions
1. Valli found the elderly woman in the bus absolutely repulsive. She immediately
developed a strong dislike for her. She noticed big holes in the lady’s earlobes and
found her earrings ugly. She was disgusted by the sight of the woman chewing betel
nut and the juice almost spilling over her lips. She was also annoyed by the woman’s
concern for her. The woman repeatedly asked about her whereabouts and if it was
proper for her to travel alone. Valli showed her dislike towards the woman openly and
curtly replied her back saying that she was capable of travelling alone. She started
looking outside the window to avoid further conversation with the woman.
2. Valli was an eight-year-old girl. She was inquisitive, a meticulous planner and mature
beyond her age. She planned her trip carefully saving every penny. Her silence after
the death of the young cow showed her sensitivity. At such tender age, she knew
how to grieve and understood the temporariness of life. When she returned home,
she saw her mother and aunt talking. She gave her affirmation to their conversation
when they talked about endless possibilities in the world one is unaware of. Though
she justified her reaction as a casual reaction, she smiled to show the readers that she
could understand the depth of the statement sentence. Valli, being a small girl, took
the bus ride and the way she managed everything showed her in a mature light.

Chapter 10 The Sermon at Benares

ANSWER KEY

Passage Based Questions


1. (a) ‘He’ refers to Siddhartha Gautama. He witnessed a sick man, then an aged man,
then a funeral procession, and finally a monk begging for alms. He went out into
the world to seek enlightenment concerning the sorrows he had witnessed.
(b) The tree was named as the Bodhi Tree because Siddhartha Gautama attained
salvation while sitting under this tree after seven days. Bodhi Tree means the tree
of wisdom.
2. (a) Kisa Gotami carried the dead child to all her neighbours, asking them for medicine
to cure the child. She couldn’t come to terms with her child’s death and lost her
rationality.
(b) The neighbours thought that she had lost her senses. She was unable to bear the
loss of her child and lost her sense of reason.
3. (a) The people were reminded of their own grief of losing their dear ones after Kisa
Gotami made them the request. They told her that the living are few but the dead
are many.
(b) The people said there were many dead in their family and Kisa Gotami was
reminding them of their grief. They asked her not to remind them of their grief.
4. (a) Gotami became weary and hopeless as she couldn’t find any family in which no
one had ever died.
(b) Not finding any family without dead people made her realise that death is common
to all. She came to terms with the death of her child and realised that she had
been irrational in her quest for medicine to bring him back.
Learnwell
ACME Math–III
ENGLISH Practice Book GR–10 (Teacher Manual) 85

HF_Acme_ENG_TM_G10.indd 85 5/22/2021 5:58:46 PM


5. (a) (iv) Troubled, brief and combined with pain (b) (i) dying
(c) b) F - (i), (iii) and O -(ii), (iv)
(d) (iii) Ripe fruits and earthen vessels (e) c) Option (iii)
6. (a) (iii) Weeping and lamentation make pain greater and body suffers.
(b) (i) Stop lamenting and become composed (c) d) (i), (v), (vi)
(d) (i) Who overcomes sorrow (e) b) Option (ii)

Short Answer Type Questions


1. The Buddha asked Kisa Gotami to bring a handful of mustard seeds. He said that
the seeds should only be procured from a house that had never experienced death
or grief. The method was successful because she couldn’t find any such house. Thus,
she realised the inevitability of death.
2. Yes, the meeting with the Buddha turned Kisa Gotami wiser than she was before.
She realised the inevitability of death, stopped lamenting and became calm and
composed. She came to terms with the death of her son.
3. Yes, Kisa Gotami was being selfish in her grief. She looked for medicines in her
neighbourhood that could revive her son from death. Again, she went from home to
home to collect mustard seeds, reminding them of their grief.
4. According to Lord Buddha, one should accept what is bound to happen. Lamenting
and weeping would make the body suffer. To attain salvation, one must stop grieving
and lamenting.
5. Gautama Buddha delivered the Sermon at Benares. In ‘The Sermon at Benares’,
the Buddha preached about the inevitability of death and the need to overcome the
grief that follows. To attain salvation, one must stop grieving and lamenting.
6. The most important lesson that one can learn from the sermon is the inevitability of
death. One who has come to earth is bound to die and there is no escape from it.
Grief weakens our path of salvation. Lamenting and weeping would make the body
suffer. To attain salvation, one must stop lamenting.
7. Siddhartha sought enlightenment after he saw the realities of the world. These sights so
moved him that he at once went out into the world to seek enlightenment concerning
the sorrows he had witnessed. He wandered for seven years and finally sat down
under a peepal tree, where he vowed to stay until enlightenment came. Enlightened
after seven days, he renamed the tree the Bodhi Tree and began to teach and share
his new understandings. Thus, he came to be known as the Buddha or the ‘awakened
one’.
8. No, Kisa Gotami didn’t get a handful of mustard-seed as directed by the Buddha.
The Buddha had asked her to collect the seeds from a house where no one had ever
died. But Kisa could not find any such house.

Long Answer Type Questions


1. Gautama Buddha was born as Siddhartha in a royal family. At the age of 12, he was
sent away for schooling in Hindu sacred scriptures. Four years later he got married
to a princess. Once on his way, Prince Siddhartha saw a sick man, an aged man,
a funeral procession and a monk begging for alms. He was touched by the misery
and pain of the world. It impacted him deeply and he left the palace and became a
monk. He wandered for seven years before finally sitting down under a peepal tree.

86 ACME English Practice Book GR–10 (Teacher Manual)

HF_Acme_ENG_TM_G10.indd 86 5/22/2021 5:58:46 PM


He was enlightened after seven days and named the tree as the ‘Bodhi Tree’. He
started teaching and sharing his knowledge and understanding of life and became
famous with the name of the Buddha. He preached his first sermon at the city of
Benares.
2. Kisa Gotami was full of grief when her only son died. She carried her dead son from
door to door to all her neighbours. From each she asked for medicine that could revive
her son. People believed that Kisa had lost her senses. On someone’s suggestion she
went to the Buddha to request him to revive her son. The Buddha asked her to bring
him a handful of mustard seeds. The seeds, the Buddha said, should be brought from
a house where no one had ever died. Blinded by her son’s love, she went from house
to house to get a handful of mustard seed. She easily got mustard seeds but didn’t
find any house where someone had not died. Thus, she realised the inevitability of
death. At last, the Buddha was able to explain to Kisa Gotami the ultimate truth of
life.

Chapter 11 The Proposal

ANSWER KEY

Passage Based Questions


1. (a) The listener Natalya and the speaker Lomov were neighbours. The speaker told
the listener that their families had known each other since long and have always
had a cordial relationship.
(b) The listener and the speaker shared a friendly and cooperative relationship that
went back to the times of their parents and relatives.
2. (a) Lomov is the speaker. He wanted to show his documents to prove his claim over
Oxen Meadows when Natalya told him that the property did not belong to him.
(b) Natalya’s father’s grandfather enjoyed the free use of Oxen Meadows for 40 years
in return for making bricks for Lomov’s aunt’s grandmother.
3. (a) Natalya is the speaker. Lomov’s claim over the Meadows surprised her as according
to her Oxen Meadows was her property.
(b) Natalya considered her family as the owner of the Meadows. So, Lomov’s claim
over the Meadows appears unfair to her.
4. (a) Natalya wanted to present the Meadows to Lomov. She said it sarcastically as she
considered herself the rightful owner of the meadows.
(b) According to Natalya not being grateful for help and Lomov claiming her property
as his own is not neighbourly.
5. (a) (ii) Lomov (b) (i) He was arguing to claim the Meadows.
(c) (i) Chubukov (d) (iii) To give them to peasants
(e) a) Option (i)
6. (a) (ii) Guess (b) a) i and v
(c) (iii) Who is better between Squeezer and Guess?
(d) (iv) Dogs like it could be found under every bush. (e) c) Option (iii)

Learnwell
ACME Math–III
ENGLISH Practice Book GR–10 (Teacher Manual) 87

HF_Acme_ENG_TM_G10.indd 87 5/22/2021 5:58:46 PM


Short Answer Type Questions
1. Natalya noticed Lomov’s evening dress. She complimented him and asked him
about the occasion for dressing up in such formal manner. This shows that she got
impressed.
2. The first quarrel that ensued between Lomov and Natalya was about the ownership
rights of Oxen Meadows. Both claimed themselves as the rightful owners of the
Meadows. To prove the point Natalya called her father and Lomov decided to show
the papers.
3. Lomov told Natalya and Chubukov that Chubukov’s grandfather’s peasants were
given the Meadows by his aunt’s grandmother for temporary and free use. Therefore,
he is the rightful owner of the meadows.
4. According to Natalya, Oxen Meadows belonged to her family for the last three
hundred years. She emphasised over her claim by putting forward an argument that
her grandfather and great grandfather thought that their land extended to the Burnt
Marsh, which meant that the Oxen Meadows belonged to them.
5. Lomov said it because Natalya was blaming him for claiming her land as his. He
thought Natalya was accusing him of being a landgrabber.
6. Natalya talked about sending her mowers and grass cutters to Oxen Meadows to
prove her ownership. This angered Lomov and he threatened to hit them.
7. Chubukov objected to Lomov’s manner of speech as Chubukov thought Lomov was
shouting. He told Lomov that yelling wouldn’t prove anything.
8. Natalya was adamant on her claim over Oxen Meadows. Therefore, Lomov threatened
her to take the matter to court and prove it to them that he was the real owner.

Long Answer Type Questions


1. After having a strong dispute over the ownership of Meadows, Natalya and Lomov
fought over the superiority of their dogs-Squeezer and Guess. Natalya being Squeezer’s
owner tried to prove that her dog was better than Guess, Lomov’s dog. She pointed
out several shortcomings in Guess. She said that Guess was old, ugly and as worn
out as a cab horse. Lomov also tried to defend his dog, Guess by stating that his dog
had become lame since its leg had been bitten by some other dog. Though Lomov
had come there with the intention of proposing Natalya for marriage but they got
involved in trivial issues and silly arguments. Chubukov played an important role in
aggravating the arguments by supporting his daughter in every petty issue.
2. On Lomov’s visit, Chubukov suspected him to have come to borrow money. When
Chubukov asked Lomov about the occasion for wearing evening dress, Lomov
informed him that he had come to ask for Natalya’s hand in marriage. Chubukov
immediately claimed Lomov to be like his own dear son. He was not honest and
sincere in saying those words. He did not hold a good opinion about Lomov. He
changed his stand when he got to know about Lomov’s intention. Even when Natalya
was arguing with him over petty issues, Chubukov supported the argument. Both
the parties hurled abuses at each other. Chubukov’s behaviour was not in agreement
with what he had said earlier.

88 ACME English Practice Book GR–10 (Teacher Manual)

HF_Acme_ENG_TM_G10.indd 88 5/22/2021 5:58:46 PM


Section-C (C3: Poetry (First Flight))

Chapter 1 Dust of Snow

ANSWER KEY

Passage Based Questions


1. (a) The poet is standing under a hemlock tree. A crow is sitting on the hemlock tree.
It shakes the tree in such a way that the dust of snow falls down from the tree
upon the poet.
(b) The poet is talking about an incident when he was standing under a hemlock tree.
He was upset. A crow sitting on a hemlock tree shook the tree in such a way that
the dust of snow fell down from the tree upon the poet and changed his mood.
2. (a) (i) gloomy (b) (iii) Delight (c) (iii) abab
(d) (ii) He overcame his sadness and became happy.
(e) (i) The bird changed his mood.

Short Answer Type Questions


1. The dust of snow symbolises natural enthusiasm, joy and energy. The poet was
experiencing sadness and depression. The dust of snow brought about a change in
his mood, and made the poet enjoy the time full of joy and fills him with optimism.
2. The very instant that the dust of snow from the hemlock tree fell on the poet, he
experienced a complete change of mood. Earlier, he was hopeless and depressed
but now he came out of his sorrowful state of mind. The dust of snow made him feel
relieved.
3. Although both crow and hemlock tree are considered inauspicious. But shaking of
dust of snow from hemlock by the crow changed the poet’s mood. As the crow shook
down the dust of snow on the poet, he was filled with optimism and energy. Now, he
could use his entire day in a fruitful way which would otherwise have gone wasted
because of his pensive mood.
4. The poem deals with the theme that even the little experiences and small things help
in changing one’s attitude and behaviour. The poet also highlights the beauty of the
nature in the form of snowflakes, which filled the poet with optimism and saved many
precious moments from going waste.
5. The poet Robert Frost gives a vivid picture of the nature. The branches of the hemlock
tree are laden with snow and a crow is sitting on one of its branches. The crow’s
movements on the tree branches cause the snow to fall down from the tree branch
upon the poet who is sitting under the tree. This causes the poet’s mood to be changed
from pensive to happy. The poet also emphasises that various aspects of nature are
so influencing that they can change people’s mood.

Learnwell
ACME Math–III
ENGLISH Practice Book GR–10 (Teacher Manual) 89

HF_Acme_ENG_TM_G10.indd 89 5/22/2021 5:58:46 PM


Long Answer Type Questions
1. Both crow and the hemlock tree are normally considered symbols of sorrow. They
are generally considered to be inauspicious. Still, Robert Frost has used both these
natural elements as the source of optimism and positivity. He has represented both
these pessimistic symbols as carriers of energy, joy and strength that transformed his
mood and saved his day from being wasted. By not using birds like sparrow and
nightingale and trees like maple, oak or a pine, the poet has tried to break down
all the preconceived notions we have about certain agents of nature. The use of
such unconventional ideas in his poem makes us realise that humans observe the
environment not as how it is, but as how they want to see it. He sees the crow and
the hemlock tree together as transformers that made him happy. The dust of snow
stands for joy. This is an unconventional treatment of nature by Robert Frost in ‘Dust
of Snow’
2. Normally, people consider both crow and hemlock tree as inauspicious. But when
the dust of snow from the hemlock tree fell on the poet, it brought about a positive
change. He was depressed and sorrowful but the moment the crow shook the hemlock
tree and the dust of snow fell on him, he felt relieved. There are times when we are
extremely sad or sorrowful. But our happiness is renewed by certain incidents in
our life. Even some small and insignificant moments of life can really infuse us with
positivity. Here, the poet wants to say that we must also realise that on every cloud
there is a silver lining. This means that every sorrowful moment is followed by a
period of joy and happiness. Thus, the poet advocates keeping a positive outlook
amidst difficulties and problems.

Chapter 2 Fire and Ice

ANSWER KEY

Passage Based Questions


1. (a) The two diverse opinions of people regarding the end of the world are that the
world will end either by fire or by ice. Some believe that the earth would be
destroyed by its fiery core, while others believe that life would vanish from the
earth on the onset of ice age.
(b) In spite of having contradictory traits, both ice and fire are similar in the sense
that both of them have the power to cause destruction on earth and bring about
an end to life.
2. (a) (ii) hatred (b) (iv) Ice represents greed. (c) (ii) hatred
(d) (i) Ice is capable of destroying the world. (e) (i) perish

Short Answer Type Questions


1. Poet says according to the scientists, the destruction of the world will be caused by
either the earth’s fiery core or the ice age. The word ‘fire’ symbolises desire. The word
‘ice’ in the poem indicates hatred and indifference. The people on earth will destroy

90 ACME English Practice Book GR–10 (Teacher Manual)

HF_Acme_ENG_TM_G10.indd 90 5/22/2021 5:58:47 PM


their own existence in pursuit of their desires. So, the end by fire seems to be more
probable according to me.
2. According to the poet, fire, i.e. human desires, would cause the end of the world. He
says so as he thinks that human desires are powerful and would lead to a quick end.
He is of the opinion that desires lead the people to destroy others. So, he concludes
that the world will end in fire.
3. The symbol ‘ice’ represents hatred and indifference which is as cold as ‘ice’. These
emotions according to the poet are so powerful and dangerous that they have the
potential to bring the world to an end.
4. The poem has been written symbolically. The poet says that there are mainly two
opinions about the causes of destruction of the world. One cause is fire and another is
ice. This is true both literally and symbolically. The symbols - ‘Fire’ and ‘Ice’ have been
used for human emotions like desire and hatred respectively that are so dangerous
that they can end the world. In literal terms, world may be destroyed due to global
warming or onset of ice age.
5. The poet says that there are mainly two opinions about the causes of destruction
of the world. According to the poet, the world will be destroyed by the ‘fire’, which
symbolises desire. But if the world has to end twice then it will be due to ice which
symbolises hatred and indifference.
6. The poet talks about the two different beliefs regarding the end of this world. Some
people say that the world will end in fire while others say it will end in ice. So, the
poet repeats the line ‘some people say’ to emphasize the inevitability of destruction
of the world.

Long Answer Type Questions


1. The poem is based on the theme of probable causes of destruction of world. The
mystery is whether the world would be frozen to death or burnt to death. The poet
determines which of the forces is powerful enough to cause the destruction of the
world. So, he selects two darkest traits—desire and hatred as the probable cause of
the end of the world. The poem answers the question of whether the world will end
in fire or in ice. ‘Fire’ symbolises desire and ‘Ice’ symbolises hatred. Desire is intense
greed that makes one solely focused on accumulating and possessing material goods.
Hate is just as powerful as desire. While desire is very quick in consuming, hate keeps
on lingering in people’s minds and hearts for years and sometimes even lifetimes.
So, the poet warns the people against the two greatest dangers to human existence
which can lead to human extinction.
2. The poet says that there are mainly two opinions about the causes of destruction
of the world. According to the poet, the world will be destroyed by the ‘fire’, which
symbolises desire. But if the world has to end twice then it will be due to the ice which
symbolises hatred. The poet feels that there is so much hatred among the people
that they don’t bother about other people. This hatred will end life on the surface of
the earth one day. The poet’s assertion that the world will end by ice seems justified
as hate can occur and linger in people’s minds and hearts for years and sometimes
even lifetimes. Hate is like a two sided sword which consumes both the hater and
the person being hated. It disturbs the mind of the hater even more than it affects
the person being hated, and it can ruin lives. Hate can, thus, be very destructive and
lead to destruction of the world.

Learnwell
ACME Math–III
ENGLISH Practice Book GR–10 (Teacher Manual) 91

HF_Acme_ENG_TM_G10.indd 91 5/22/2021 5:58:47 PM


Chapter 3 A Tiger in the Zoo

ANSWER KEY

Passage Based Questions


1. (a) ‘He’ refers to the tiger. ‘He’ lurks in shadow to catch its prey.
(b) The plump deer would pass through the water hole. It will do so to drink water
in the water hole.
2. (a) The tiger should be snarling on the outskirts of the jungle around houses and
terrifying villagers. These lines suggest that the tiger should roam freely in jungle
and terrorise people who try to infringe upon its natural habitat.
(b) The tiger terrorises the villagers by growling around the houses of villagers which
were at Jungle’s edge, and by baring his white fangs and his claws.
3. (a) (i) Cage in zoo (b) (i) Going through an area
(c) (ii) His strength behind bars
(d) a) F—(i), (ii), (iii) and O—(iv) (e) (i) alliteration
4. (a) (iv) repetition (b) (ii) Tiger (c) (iv) patrolling cars
(d) (ii) stars (e) c) F—(i), (ii) and O—(iii), (iv)

Short Answer Type Questions


1. The poet says that the tiger is very beautiful and is walking in his little cage. He has
beautiful stripes on his skin and has velvet-like soft paws. The stripes on his body can
be distinguished even from far as they are darker in colour than the rest of his body.
2. The tiger stalks constantly the length of his cage expressing his age quietly. He ignores
the visitors and feels helpless. There is nothing he can do from behind the bars. He
keeps on staring at the brilliant stars in the open sky to express his anger.
3. The poet gives example of a pump deer to express that the tiger’s natural habitat is
in the jungle where he hunts his prey out in the open. No matter how well he is fed
by the zoo authorities, his place is not inside the cage.
4. The tiger hears the sound of the patrolling cars at night that he might not have heard
this in forest. Rather he might have heard voices of various animals and rustling of
leaves. So, the kind of voices he hears in the cage are very different form the voices
he might have heard in the forest.
5. There is a huge difference between the behaviour of the tiger in a zoo and in a forest.
A tiger in a zoo is a caged animal who has no freedom to move beyond the boundaries
of the cage whilst a tiger in a forest is independent and feels powerful and supreme.
6. The tiger looks at the stars at night with hope. He hopes for the day when he would
be able to run free in the wild. The brilliance of the stars, thus, provides him with
some comfort.
7. The tiger keeps on walking along the length of the cage. He ignores visitors, feels
irritated and stares at the brilliant stars shining in the sky. He feels restricted and yearns
for his natural habitat.

92 ACME English Practice Book GR–10 (Teacher Manual)

HF_Acme_ENG_TM_G10.indd 92 5/22/2021 5:58:47 PM


8. The tiger in the wild is majestic. He is independent and free. He lies under the shade
and hunts for its prey. He goes near the water to hunt for his prey because plump
deer and other animals are found in plenty there.

Long Answer Type Questions


1. The poet makes an attempt to highlight the miserable life that the animals in a cage lead
in a zoo. He compares the life of the tiger in the zoo with his life in his natural habitat.
He wants to say that like other creatures, the animals too have the right to freedom
and should not be kept behind bars. They should be allowed to run freely in the wild.
Everyone loves freedom and does not want to live in confinement. Similarly, the tiger
also longed for freedom. He was so frustrated and angry over his condition that he
even ignored the visitors. Being helpless, he could just move to and fro within the
boundaries of his cage. The tiger wanted to escape from the captivity. It is an animal’s
right to enjoy its natural habitat i.e. the forest and live independently and freely in
the wild. So, it’s our moral duty to not put them in the zoo for our entertainment.
2. Wild animals are meant to live in the wild. They are not meant to be caged and
displayed in the zoos. We all know that the majestic species of tiger is on the verge of
extinction. Years ago, there used to be a situation when they were in plenty and used
to live and roam about freely in their natural habitat. They should not be forced to
live a life in confinement. Their right to live freely is as important as all other living
beings’ right to freedom. Confinement leads to depression and misery. Moreover,
their offspring not being born in natural habitat lose their capabilities to hunt. This
would affect their survival abilities as they would not be able to feed themselves.
This is highly dangerous for their long term existence. The whole ecological balance
and food chain is disturbed due to confinement of wild animals. Humans should
understand that animals belong to the forest and not to the cage. So, they should let
the animals live their lives with freedom and without human intervention.

Chapter 4 How to Tell Wild Animals

ANSWER KEY

Passage Based Questions


1. (a) Leopard is the animal being described in the given lines. We can see the dark
spots on the skin of the beast.
(b) The animal will leap upon you at once means that it will jump on you. Crying in
pain will be of no help as it will keep on jumping on you.
2. (a) We might meet a bear while walking around in our yard. The animal is known
for hugging very hard.
(b) The bear is going to hug you hard. If you have doubt or are still alive, he will give
you another tight hug to kill you.
3. (a) (i) tears (b) (i) beasts of prey (c) a) (i), (ii), (iii)
(d) (ii) They are deceptive. (e) (ii) inexperienced

Learnwell
ACME Math–III
ENGLISH Practice Book GR–10 (Teacher Manual) 93

HF_Acme_ENG_TM_G10.indd 93 5/22/2021 5:58:47 PM


4. (a) (iii) Chameleon (b) (ii) paradox (c) (ii) ear and wings
(d) (iv) merge with the background (e) c) (ii), (v)

Short Answer Type Questions


1. There is a very simple rule to identify a Bengal Tiger. It has black stripes on its yellow
skin. It silently attacks its prey.
2. The novice may feel confused in differentiating between a hyena and a crocodile.
The hyena laughs while it swallows its victim while a crocodile cries while it swallows
its victim.
3. According to the poet the hyena laughs while it swallows its victim while a crocodile
cries while it swallows its victim.
4. The Asian Lion is large in stature. It is brownish-yellow in colour. It roars very loudly.
The Bengal Tiger has black stripes on a yellow hide. It roams freely in the forest. It is
noble and majestic in stature.
5. A chameleon is a garden lizard. It is an expert at camouflage. It is capable of changing
its colour as per its surroundings. This ability of camouflage makes it difficult to locate
a chameleon on a tree.
6. It is dangerous to go near wild animals as they can attack us and kill us. The poet tells
how to identify various wild animals and harms caused by them in a fun, humorous
manner.
7. The poet says that while walking round the courtyard of his house if a person meets
a creature who hugs him very hard, then he can be sure that it is a bear.

Long Answer Type Questions


1. ‘How to Tell Wild Animals’ is a beautiful poem by Carolyn Wells. She tells about
the strange habits and behaviour of some wild animals in this poem. First she writes
about Asiatic lion that it is a big sized tawny creature which has a terrifying roar. She
describes a tiger as a beast with black stripes on his yellow hide which is always ready
to eat its victim. The animal with black spots on its skin is leopard. It silently leaps
on its prey and eats it up. The bear has been described as an animal which hugs a
person tightly and puts him/her to death with its tight embrace. A crocodile always
sheds tears while eating its victim. A hyena always keeps smiling. A chameleon always
adjusts its colour according to its surrounding. Every creature has distinct quality
which distinguishes it from others. This uniqueness of every wild animal adds to the
beauty of the planet we live on. Thus, the poet brings about the uniqueness of each
animal.
2. The poet Carolyn Wells creates humour by describing different wild animals. The
poet mentions the uniqueness of different animals by highlighting their distinct
characteristics. Different animals have been described in a very funny and interesting
manner by the poet. Throughout the poem, the characteristic traits of animals have
been portrayed in a funny manner. The bear has been described as an animal which
hugs a person and puts him to death with its tight embrace. A crocodile always sheds
tears while eating its victim. A hyena always keeps smiling. Chameleon can be said
to be present on the tree if one can’t see them. All these descriptions are humorous
as they are wild beasts. They can neither laugh, smile nor be gentle. They will kill the
human beings at once as they get the chance.

94 ACME English Practice Book GR–10 (Teacher Manual)

HF_Acme_ENG_TM_G10.indd 94 5/22/2021 5:58:48 PM


Chapter 5 The Ball Poem

ANSWER KEY

Passage Based Questions


1. (a) The boy loses his ball and gets upset. He gets to learn his first lesson in realising
about loss in life. With the loss of the ball, he gains the experience of losing and
understands how to cope up with the loss.
(b) The boy was very upset after losing the ball. He was overcome with sadness,
which had a great impact on him. He stood stunned and stiff. He was filled with
grief. He stood trembling, and stared down where his ball went rolling.
2. (a) The poet refuses to intrude because he wants the boy to learn the meaning of loss
on his own. He does not try to calm him by offering him another ball or money
to buy another ball because he thinks he needed to learn to accept loss.
(b) The boy loses his ball and gets upset. This incident gave him his first lesson in
realising his first responsibility. With the loss of ball, he has experienced the pain
of losing something. So, he will be learning how to cope up with the loss. He
realises that from that day onwards he has to be more responsible towards his
possessions.
3. (a) (i) how to cope up with loss
(b) c) option (iii) (c) (iii) repetition
(d) (iii) Money can’t buy back or replace our sense of pain, sorrow or loss.
(e) (iv) All of these

Short Answer Type Questions


1. The writer uses the ball as a symbol where losing the ball symbolises losing his
childhood and the ball symbolises worldly possessions.
2. The boy loses his ball and gets upset. This incident gave him his first lesson in loss.
With the loss of ball, he experienced the pain of losing something. He was overcome
with sadness. He stood stunned and unmoved. He stood trembling and stared down
where his ball went rolling. Through his event, he learnt that he would have to learn
how to cope up with the loss. He understood what loss meant. He would be more
responsible and careful to avoid losing something and would accept loss in future.
3. Loss here has been envisioned as miseries arising out of the losses one suffers in life.
In present day’s competitive world, materialistic gains and losses go hand in hand.
So losses are an integral part of life. Everyone should know how to accept losses and
then move on.
4. The poet wants him to learn to cope with his loss. He does not offer him another ball
or money to buy another ball because he wants to make him understand that money
cannot buy everything. He wants him to be mature enough to let go of things and move
on in life. He wants him to learn that once something is lost, it may never be restored.
5. The boy was playing with his ball. The ball while bouncing rolled down the street.
From the street the ball fell into the water. In this way, the boy lost his ball.

Learnwell
ACME Math–III
ENGLISH Practice Book GR–10 (Teacher Manual) 95

HF_Acme_ENG_TM_G10.indd 95 5/22/2021 5:58:48 PM


6. The poet says that from this loss, the boy would learn what it means to lose something.
Thus he would understand the nature of loss and to cope up with losses one suffers
in life. This experience of losing something would teach him how to be responsible.
He would learn how to avoid losses and also to make up for loses that he might suffer
in life.
7. The poet believes that money cannot buy everything. He considers money to be
external which can bring only external happiness. It can buy us possessions and
luxuries but it cannot make someone feel contented. The boy cannot replace his loss
of a loved one or valued thing with materialistic things.

Long Answer Type Questions


1. The boy loses his ball and gets upset. This incident gives him his first lesson in loss.
With the loss of ball, he has experienced the pain of losing something. So he will be
learning how to cope up with the loss. He now understands what loss means. He
now will be more responsible and careful to avoid losing something. In present day’s
competitive world, materialistic gains and losses go hand in hand. So losses are an
integral part of one’s life. Everyone has to bear loss and need to learn to move on.
Everyone has to experience a loss of any kind like loss of a beloved, or a parent or a
close relative, or even a pet, separation from friends, loss or theft of favourite items,
etc. Humans get attached to things or people they have and when they have to face
separation from them, they feel grieved. The poet wants the boy to realise that he
cannot compensate for his loss of a loved one or a valued thing with materialistic
things. Loss is an inevitable and significant experience of one’s life, and one must
learn to deal with it and move on.
2. The boy loses his ball and gets upset. With the loss of ball, he experiences for the first
time, a sense of loss. With this loss, he has to learn to cope up with loss in life. He
has to learn to be more responsible for his possessions. He needs to learn to cope
up with loss and move on. I think the poet, in these lines, conveys a great message.
He conveys that in present day’s competitive world, materialistic gains and losses
go hand in hand. So losses are a part of life. Everyone should bear and make up
for their losses. According to the poet, the epistemology of loss is the greatest lesson
the boy is learning. It teaches him to value and preserve his cherished things. It also
teaches him how to cope up with the loss and overcome the grief caused by the loss of
valued things. If once we have understood how much painful it is to lose something,
we become more vigilant and learn to handle things in a better way. Thus it helps us
to be self-reliant and responsible.

Chapter 6 Amanda!

ANSWER KEY

Passage Based Questions


1. (a) In these lines, Amanda is being instructed not to bite her nails, not to hunch her
shoulders and not to sit in a slouchy manner.

96 ACME English Practice Book GR–10 (Teacher Manual)

HF_Acme_ENG_TM_G10.indd 96 5/22/2021 5:58:48 PM


(b) Amanda is getting these instructions to correct her behaviour and bad habits. Her
mother wants her to be more poised and graceful.
2. (a) Amanda imagines herself to be a Mermaid. She fancies moving in a place where
she is not observed and instructed by anyone in the same way as a mermaid
swims in a sea freely and blissfully.
(b) Amanda wants to swim peacefully and undisturbed in a languid, emerald coloured
sea. She wants to go to a place where she is not bothered by anyone.
3. (a) (iii) fairy tale (b) a) F—(i) and O—(ii), (iii), (iv)
(c) (iv) All of these (d) (iii) She wants to be free like her.
(e) (i) So that nobody could climb up to her
4. (a) (iii) Amanda (b) (ii) Amanda’s mother
(c) (i) Freedom-loving, moody
(d) (iv) aching (e) (iv) She nags Amanda.

Short Answer Type Questions


1. Amanda’s mother thinks that Amanda is a moody girl. She often escapes from reality,
which makes her moody and uninterested. She asks Amanda to stop sulking and pay
heed to her words.
2. Amanda’s mother wants that Amanda should follow her instructions. She is told to
not bite her nails, not hunch her shoulders, not slouch and sit straight, make her room
tidy, clean her shoes, do her homework, not eat chocolate and stop sulking.
3. Amanda imagines herself as an orphan because she wants to be alone without
anybody’s supervision. She wants to be free from her mother’s constant nagging.
4. Amanda’s mother is an overbearing mother. She keeps on giving instructions and
finding faults in Amanda. It is undoubtedly her duty to instill good values into her
daughter, but not in such a way that her daughter feels suppressed and frustrated.
She should be considerate of Amanda’s mental state.
5. Amanda does not take the instructions of her mother seriously as she is moody. Also,
she is sick of her mother’s regular instructions. She thinks that she is nagged most of
the time. Hence, she becomes uninterested in her mother’s instructions.
6. Amanda feels suppressed and irritated at constantly being nagged by her mother.
She is so frustrated that she escapes from reality by living in her imaginary world.
Her dreams are symbolic of her desire for freedom.
7. Amanda wants to be Rapunzel so that she could live carefree on a high tower. Rapunzel
lived in a high tower amidst a forest where she had no contact with the outside world.
She wishes to be like her so that she could escape her suffocating reality.

Long Answer Type Questions


1. Amanda is a small girl who is entering into her teenage. She has been shown as being
moody and careless. Amanda’s mother is a nagging mother. She keeps on giving
instructions and finding faults in Amanda. It is undoubtedly her duty to instill good
values in her daughter, but not in such a way that her daughter feels suppressed and
frustrated. She fails to understand Amanda’s mental state. She is too harsh on her
when she tells her that she has acne and should not eat chocolate. She doesn’t use
love to make Amanda understand the fault in her behaviour. Amanda feels frustrated

Learnwell
ACME Math–III
ENGLISH Practice Book GR–10 (Teacher Manual) 97

HF_Acme_ENG_TM_G10.indd 97 5/22/2021 5:58:48 PM


by the barrage of her criticism. She is so frustrated that she wishes to be an orphan.
Finding herself suppressed and helpless, Amanda seeks freedom in her imagination.
She even imagines herself to be an orphan. Understanding a child’s perspective and
attitude plays a very crucial role in their upbringing process. Upbringing requires
understanding from both the sides. One cannot just transform a child into a civilised
and good mannered adult through nagging. Love and proper care is indeed required
in nurturing of a child.
2. It goes without saying that guidance of parents is important for a child to become
civilized and educated member of the society. But some parents are so over protective
and over anxious about their children that they don’t let their children live in peace.
They don’t permit their children to take initiatives and gain new experiences. Child’s
freedom and creativity is hampered in such cases. Such parents instead of doing good,
suppress the child’s initiative in learning. These parents only create gap between them
and their children by always bossing around and irritating them. However, the parents
should guide their children to the correct path. Children too should understand parents’
concern towards them. Their intention of instructing children is for children’s better
future. So, we can say parent-child relationship is complicated and both the parents
and the child should work together to improve this relationship.

Chapter 7 Animals

ANSWER KEY

Passage Based Questions


1. (a) The poet wishes to live with animals. He wants to be like animals that do not
complain about anything and live free of sins and sorrows.
(b) According to the poet, animals are calm and self-sufficient. They do not complain
or get depressed about their conditions. They remain contented.
2. (a) Animals are calm and contented. They do not whine and weep about their
circumstances. Unlike humans, they do not cry over their misery and grief. They
also don’t lie awake at night due to the sins they have committed.
(b) When humans discuss about their duties towards God while committing all sorts
of sins, it makes the poet sick.
3. (a) (iii) Walt Whitman (b) (ii) Animals
(c) (i) Relationship of equality
(d) (ii) Virtues (e) (iv) Metaphor
4. (a) (i) Where have the humans left their good qualities?
(b) (iii) Animals, Walt Whitman
(c) b) F—(i),(ii), (iii) and O—(iv)
(d) (i) Path of service (e) (ii) Animals

98 ACME English Practice Book GR–10 (Teacher Manual)

HF_Acme_ENG_TM_G10.indd 98 5/22/2021 5:58:48 PM


Short Answer Type Questions
1. According to the poet, animals are calm, self-sufficient, and contended. They do not
whine and weep about their conditions. They are not dissatisfied and do not run after
materialistic things. They do not kneel before anyone.
2. Humans kneel before their ancestors who lived thousands of years ago to show them
respect. They worship them and wish to follow their path.
3. The poet says that animals instead of praying to God just perform their duty. The do
not kneel before anyone. No one is respectable or unhappy over the whole earth. The
virtues of animals and the lack of social conventions make them calm and contented.
4. No, animals do not show discrimination and inequality among themselves unlike
humans. They do not pray to God or to ancestors and consider everyone equal.
5. Humans are considered maniac in the poem. They suffer from hypocrisy, materialism,
dishonesty and greediness. They have artificial social conventions they follow. They
are hypocrites who don’t practise what they preach.
6. The vices that human beings posses are ill-will, hatred, greediness and cruelty. These
vices devoid them of love and understanding. They value one thing over other. This
makes them commit sin and then ask for forgiveness from God.
7. According to the comparison drawn in the poem, animals are better than human
beings. Human beings lack qualities of sincerity and innocence. On the other hand,
animals live life free of sins. They do not whine about anything and accept everything
as it is.
8. Humans commit sins yet discuss their duties towards God. This makes the poet sick.
This shows their hypocrisy. They commit wrongdoings but also pray to God.

Long Answer Type Questions


1. Animals live a placid, self-contained, and simple life. They are contended in their
natural surroundings and do not bow to anyone. They are not attracted towards
attaining materialistic possessions. They don’t worship god as they never suffer from
any sense of guilt. They do not complain. On the contrary, humans possess more vices
than virtues. They are greedy, arrogant and suffer from the mania for accumulating
possession. Animals live in the present whereas human beings struggle between
the past and the future. They worship their ancestors to make their lives smooth,
are overly ambitious and try to attain as much materialistic possessions as possible.
Humans are never satisfied.
2. The poet considers animals superior to men. Human beings are hypocrites. They
commit crimes on one hand and talk about their duties towards God on the other
hand. They run after materialistic things and still whine about their conditions. But
animals neither whine nor cry over their situations. They are not demanding. They
have a clear conscience. They live a simple life away from worldly pretensions. While
humans are taking control of the earth and proving their ownership over the lands,
animals do not care about possessions. The poet seems disappointed by the human
behaviour and feels drawn to the animals because of their virtues.

Learnwell
ACME Math–III
ENGLISH Practice Book GR–10 (Teacher Manual) 99

HF_Acme_ENG_TM_G10.indd 99 5/22/2021 5:58:48 PM


Chapter 8 The Trees

ANSWER KEY

Passage Based Questions


1. (a) The roots want to free themselves. They are struggling to come out of the cracks
in the veranda floor.
(b) The leaves are pushing themselves hard towards the glass to help the tree move
out.
2. (a) The poet is writing long letters. She had observed the trees struggling to make
their way to the forest but she did not describe any of it in her letters.
(b) The poet describes the night as fresh. The moon is shining in the sky. The smell
of leaves and lichen has spread in the rooms.
3. (a) ii) Readers (b) (iv) the continuous efforts by the tree
(c) (iii) I have a lot of fears and concerns.
(d) (i) attain freedom (e) (iv) Detectable
4. (a) (iv) simile (b) (i) trees
(c) (iv) Wind comes to meet the uprooted trees which have come to the forest.
(d) (i) To meet the trees (e) ii) Oak tree

Short Answer Type Questions


1. The poet’s head is full of whispers as the trees are struggling to free themselves from
their artificial habitat. They are marching toward the forest which is their original
habitat. The poetess is a witness to the trees’ struggles.
2. The roots of the struggle all night to free themselves from their artificial habitat. They
strive to come out from the cracks in the veranda floor. The leaves keep pushing
themselves towards the glass. Branches make great movements under the roof in
order to move outside. They become stiff with exertion.
3. The leaves move towards the glass with great effort. Small twigs have become stiff
due to exertion. The boughs are changing their positions by making sudden and
quick movements under the roof.
4. In the empty forests, the sun scorches as there were no trees to provide shade. Birds
have no place to perch on and insects to feed upon. The insects cannot find a place
to hide.
5. In the poem, trees symbolise women who have been held captive within the four
walls of the house and denied equal rights in the society. They are trying to break
the shackles of gendered roles and striving for equality.
6. In the poem, the nature has been personified. The trees are shown struggling to break
free from the artificial barriers. They are striving to move towards their natural habitat,
the forest. The poetess has used metaphor to compare trees to newly discharged
patients. She uses simile to compare reflection of the moon on oak trees to a broken
mirror.

100 ACME English Practice Book GR–10 (Teacher Manual)

HF_Acme_ENG_TM_G10.indd 100 5/22/2021 5:58:49 PM


7. The poetess is hopeful that the trees after effortlessly working all night to move
towards the forest will reach their natural habitat by morning. She is anxious about
their departure. She doesn’t mention their departure as she feels that might affect
their movement.

Long Answer Type Questions


1. The poem ‘The Trees’ represents the conflict between nature and man. In the poem
the nature has been personified. The trees which are kept at home away from their
natural habitat are struggling to move towards the forest. Without trees in the forest,
there will be no birds and insects. Hence, the ecological balance will be disturbed. But
man, for his own greed, has captured the nature in the boundaries of his home. He
uproots saplings from their natural place of existence and grows them in man-made
glasshouses. In the poem, the nature struggles to come out of these close boundaries
and return to their true natural habitat.
2. The march of the trees is symbolic of their struggle for freedom. The struggle of the
trees portrays the struggle of women to break the walls of domesticity and silence to
move out in the real world to make a better place for themselves. Just like the trees,
the women also labour long and hard to break the chains of oppression that keep
them behind. But their struggle will soon be fruitful just like the trees that will reach the
forest soon. As the poet herself is a renowned feminist, she is particularly concerned
about women who are suppressed by the patriarchal system and are struggling to
free themselves from its captivity.

Chapter 9 Fog

ANSWER KEY

Passage Based Questions


1. The fog enters silently and slowly like a cat. When it comes, it does not make any
noise. It silently comes and then moves away in the same manner.
2. It sits as silently as a cat does. 3. (ii) F—(a), (b) and O—(c), (d)
4. (a) personification 5. (a) silent movement
6. (a) personification 7. (d) then moves on

Short Answer Type Questions


1. The poet has used cat as a metaphor because it comes as silently as a cat does. Its
silent movement has been compared to a cat.
2. The poet personifies the fog as a cat. The fog like a cat comes on silently, sits on its
haunches and moves on in complete silence.
3. The poet says as the cat comes on its tiny silent feet when it is stalking, the fog also
comes in complete silence and engulfs the entire city and harbour. Then like a cat it
moves away silently.

Learnwell
ACME Math–III
ENGLISH Practice Book GR–10 (Teacher Manual) 101

HF_Acme_ENG_TM_G10.indd 101 5/22/2021 5:58:49 PM


4. The poet minutely describes the fog’s movement. The poet says a cat comes on its
tiny silent feet when it is stalking, in a similar manner the fog also moves in complete
silence and spreads over the city. This makes the reader visualise the entire scene of
the arrival and dispersal of the fog over the city and the harbour.
5. The poet makes the silence prominent in the poem by describing the way how fog
comes, spreads and goes away.

Long Answer Type Questions


1. There are three stages of the fog which have been discussed in the poem. First, it
comes silently just like a cat does on its tiny silent feet when it is stalking. Like a cat,
it moves in complete silence. Secondly, it spreads itself over the entire area. It covers
everything from the harbour to the city. It sits silently as a cat sits on its haunches. In
the third stage, it has been described that the fog doesn’t stay at a place for long. Just
like a cat never stays at one place for a long time, the fog also moves ahead without
anybody knowing its destination. Carl Sandburg describes the nature in its raw form.
He shows fog to be silent yet powerful. Although the fog spreads silently, everyone
feels its power.
2. The poet describes the fog as a cat. He does so through the use of metaphor. Like a
cat comes and sits silently, the fog also comes noiselessly. Then it moves on like a cat.
There is a strong similarity between the silent arrival of the fog and arrival of a little
cat. Just like the cat stays on its haunches, the fog stays over the city sitting calmly.
The fog has been described as performing human actions using personification like
coming on its tiny silent feet as a cat does when it is stalking and moving in complete
silence. The poet minutely describes fog’s movement.

Chapter 10 The Tale of Custard the Dragon

ANSWER KEY

Passage Based Questions


1. (a) ‘She’ refers to Belinda. She had four pets: a dog, a cat, a mouse and a dragon.
(b) Other animals generally thought Custard, the dragon to be a coward. They mock
poor Custard for his timidity.
2. (a) The dragon had big sharp teeth. He had spikes on top of him and scales
underneath. His mouth looked like a fireplace and there were dagger like nails
on his toes.
(b) The poet has used a simile to describe the dragon’s mouth. The dragon through
his mouth could spit fire. That’s why his mouth looked like a fireplace. He also
snorted smoke through his nose after spitting fire like a chimney.
3. (a) (iv) simile (b) d) F—(iv) and O—(i), (ii), (iii)
(c) (iv) cowardice (d) (iii) Bravery (e) (ii) chasing lions
4. (a) (ii) Ink, Blink and Mustard were friendly towards Custard.
(b) (iii) aabb (c) (i) mock at him (d) a) Option (i)
(e) (iii) they thought him to be coward

102 ACME English Practice Book GR–10 (Teacher Manual)

HF_Acme_ENG_TM_G10.indd 102 5/22/2021 5:58:49 PM


Short Answer Type Questions
1. The dog Mustard fled crying away from the dangerous pirate. Ink hid himself and
the little mouse Blink disappeared in the mousehole. Mustard said that he has been
twice as brave if he hadn’t been flustered. Ink and Blink also believed that they were
thrice as brave.
2. The pirate incident brought appreciation for Custard. Belinda embraced him. Mustard
licked him. Ink and Blink danced around him. Later, all of them started bragging
about their bravery and went back to calling the dragon a coward.
3. Everyone used to make fun of Custard’s cowardice. Even after killing the pirate,
Custard didn’t get long-term appreciation. After dancing around Custard to show
their appreciation, all of them started to brag about their bravery, forgetting about
Custard. Custard was also humble and didn’t like to brag about himself. Therefore,
Custard thought he was not as brave as others.
4. The poet repeats the first and the fourth stanzas to emphasise Belinda and other pets’
perspective towards Custard. It shows that despite behaving bravely in times of danger,
Custard could not win others’ admiration because of their superiority complex and
habit of bragging.
5. The poem gives the message that appearance can be deceptive. Dragon, who is
known to be fierce, is shown as a meek animal while pets like mouse and cat brag
about their bravery. Those who are boastful of their bravery are seldom brave. We
can see this when Mustard, Ink and Blink on encountering the dangerous pirate dash
into their hiding spot.
6. Ballad is a story sung in the form of a poem or song written in short stanzas. The
Tale of Custard the Dragon is a ballad. It is a humorous poem narrating the story of
Custard, the cowardly dragon who proved himself to be brave.
7. When the pirate tried to break into Belinda’s house, brave Mustard, Ink and Blink
got terrified and escaped from the pirate’s sight. Custard challenged the pirate and
attacked him. The pirate, in order to save himself, fired two bullets at Custard.
8. When the pirate was killed, Belinda hugged the dragon and Mustard licked him. Both
Ink and Blink danced around the dragon in happiness.

Long Answer Type Questions


1. The Tale of Custard the Dragon is a light hearted and humorous poem. All the
characters have names which rhyme with each other. Many devices have been used
by the poet to create humour in the poem. For example, the poet uses the word
‘winda’ instead of ‘window’ to rhyme with ‘Belinda’. To give a rhythmic effect, other
such rhyming words ‘household’ and ‘wagon’ are used to rhyme with ‘mouseholed’
and ‘dragon’ respectively. The poet uses poetic device of repetition. The word ‘little’
is used repeatedly to describe Belinda’s house and her pets. The repetition of the
line “And her realio, trulio little pet dragon” in many stanzas can be observed which
is the example of refrain.
2. The description of the dragon and his under-confident attitude provide a stark
contrast. Custard’s features are described as dangerous. He has big sharp teeth and
spikes on top. His skin is covered with scales. His mouth is called a fireplace because
of the dragon’s ability to spit fire from the mouth. His nose is compared to a chimney
through which the smoke comes out every time he spits fire. His feet are compared
to a sharp knife, i.e. a dagger. But in spite of his appearance, he was a mild creature.

Learnwell
ACME Math–III
ENGLISH Practice Book GR–10 (Teacher Manual) 103

HF_Acme_ENG_TM_G10.indd 103 5/22/2021 5:58:49 PM


He cried for a nice safe cage and others made fun of him. The other animals claimed
to be brave.
But the appearances can be deceptive. When the pirate attacked, all other animals
and Belinda ran away and hid themselves. Only the dragon challenged the pirate,
attacked him and ate him up. Soon everyone started praising him. But after some
time, they went back to their previous behaviour.

Chapter 11 For Anne Gregory

ANSWER KEY

Passage Based Questions


1. (a) The young men are in despair because the speaker rejects them. They are taken
in by her outer beauty and feel despondent due to her refusal.
(b) The speaker wants to find a young man who will love her for her inner beauty,
not her appearance. She feels that most men love her for her outer beauty and
she doesn’t want that.
2. (a) (iii) God only loves those that listen to his dictates given in religious texts
(b) (i) physical beauty (c) (iii) abcbdb (d) (iv) Anne Gregory
(e) (ii) God loves us for our inner worth so we should focus on improving our inner
qualities.

Short Answer Type Questions


1. Anne Gregory means that she wants to be loved for her inner beauty, not for her
outward appearance and personality.
2. Divine love is superior to human love as divine love is permanent. God loves us
for our inner beauty and worth. He does not care for our outer appearances and
personality.
3. According to the poem, external beauty is short lived but inner beauty remains forever.
Inner beauty gives us satisfaction and joy. We should thus, learn not to accept things
at their face value.
4. In the poem, the colour of hair is considered as the symbol of beauty. Anne Gregory
wishes that people love her for her inner qualities not for her hair. She can dye her
hair in brown, black or carrot colour. The appearance or personality can change but
her heart, i.e.; her inner beauty will remain the same forever.
5. Anne’s hair is described as honey-coloured. Her hair covers her ear like a wall covers
a fort. Her hair enhances her external beauty thoroughly and the young men are
attracted by it.
6. Love can be selfless. God’s love for his creation is unconditional. God can love a
person regardless of his appearance and other attributes.

Long Answer Type Questions


1. Physical appearance alone cannot give the true account of a person as it can be
changed. This is evident from Anne’s reply to the first speaker that her beautiful

104 ACME English Practice Book GR–10 (Teacher Manual)

HF_Acme_ENG_TM_G10.indd 104 5/22/2021 5:58:49 PM


hair colour which attracts men is transitory and prone to change. One should not
love someone on the basis of their physical appearance. So, Anne Gregory is right
in rejecting the impact of physical beauty on the concept of love. A person must be
judged on the basis of his virtues and behaviour that shows the true characteristics
of his/her personality. The same idea is explained by Anne in her reply to the first
speaker that her beautiful hair-colour which attracts men is changeable; hence, men
should not fall in love with her for the colour of her hair.
2. Yes, I agree with the old man’s assertion that only God can love humans for their
innate qualities not physical appearance. Humans are often misguided by external
beauty. Only God is capable of not caring for the outer beauty. He looks at the beauty
of one’s soul. He is capable of looking at a person’s real self, beyond the external
beauty. External beauty is short lived but inner beauty remains forever. Inner beauty
gives satisfaction and joy. Men like Anne Gregory for her outward appearance. Anne
wants men to like her for her inner beauty or personality. The poet tells her that it is
only God who loves us for what we are and not how we look. He loves us for our
inner qualities.

Section-C (C4: Supplementary Reader


(Footprints without Feet))

Chapter 1 A Triumph of Surgery

ANSWER KEY

Passage Based Questions


1. (a) Mrs Pumphrey used to overfeed her dog Tricki with luxury food, such as cakes
and chocolates. This led to Tricki becoming morbidly obese. When Dr Herriot
saw Tricki’s condition, he advised her to harden her heart, cut his food down and
give him a strict diet.
(b) The speaker here is Mr Herriot. He advised Mrs Pumphrey to give Tricki more
exercise and cut down on his food and give him only nutritious food.
2. (a) The beds, toys, cushions, etc. hadn’t been used because Mr Herriot wanted to
improve Tricki’s habits by giving him a lot of exercise. For this, he ensured that
he adapted to the environment as other pets.
(b) Dr Herriot was successful in treating Tricki. Seeing Tricki’s recovery, tears shone
in Mrs Pumphrey’s eyes. She was thankful to Dr Herriot for treating Tricki.
3. (a) (iii) Mr Herriot
(b) (i) He enjoyed the things sent for Tricki.
(c) (ii) To improve Tricki’s condition
(d) (iv) All of these (e) (iii) lavishly
4. (a) (i) Tricki (b) (iv) All of these
(c) (ii) Agile (d) (iv) All of these
(e) (iii) The doctor, Mr Herriot
Learnwell
ACME Math–III
ENGLISH Practice Book GR–10 (Teacher Manual) 105

HF_Acme_ENG_TM_G10.indd 105 5/22/2021 5:58:50 PM


Short Answer Type Questions
1. Mrs Pumphrey thought Tricki was suffering from malnutrition because he became
listless and had no energy. To provide him some relief, Mrs Pumphrey gave him some
little extras between meals to build him up, some malt and cod-liver oil and a bowl
of Horlicks at night to make him sleep.
2. Mrs Pumphrey was a rich lady who showered love on Tricki with all kinds of foods.
Tricki, on the other hand, never refused food. He would tackle a meal at any hour
of the day or night. For exercise, he just had little walks with Mrs Pumphrey. That is
why he became fat.
3. When Tricki was under treatment at the surgery, Mrs Pumphrey remained anxious
about his health. She would call the doctor dozen times a day for the latest update
on his health. When Dr Herriot was finally able to tell her about Tricki’s improved
condition, Mrs Pumphrey started sending two dozen fresh round eggs, wine and
brandy for him.
4. When Mrs Pumphrey got to know about Tricki’s improved health, she started sending
eggs and wine to the surgery for him to build up. It shows that she was a pampering
and an overcaring woman. She was ignorant towards the dog’s requirement for
maintaining good health.
5. The doctor informed about Tricki’s recovery when he was out of danger and recovering
rapidly. He had suggested a fortnightly hospitalisation and at the end of it he conveyed
the information about Tricki’s improved health to Mrs Pumphrey. He didn’t delay the
information as he knew that Mrs Pumphrey must have been very worried.
6. Tricki had been away from Mrs Pumphrey for a fortnight. Mrs Pumphrey was anxious
about Tricki’s health. She would phone the doctor dozen times a day and couldn’t
wait to see Tricki. Therefore, when Dr Herriot informed Mrs Pumphrey about Trick’s
complete recovery, she reached the surgery immediately.
7. According to Dr Herriot, Tricki was different from other dogs at the surgery. The other
dogs were active, whereas Tricki would lay motionless without any energy. He had
become obese due to being overfed. So he had no energy to do any activity.
8. During Trick’s treatment at the surgery, Mrs Pumphrey used to send eggs, brandy
and wine every day to help build him up. Dr Herriot would not give the food to the
dog for his health reason. He and his staff used to consume the items. This tempted
them to keep Tricki as a permanent guest.

Long Answer Type Questions


1. Mrs Pumphrey was an overcaring and ignorant owner. She used to overfeed Tricki with
sweets and other food. She fed Tricki malt, cod-liver oil and bowl of Horlicks when she
thought he was malnourished. Tricki had no exercise regime and had little walks with
his owner. This contributed to his obesity and illness. Mrs Pumphrey had to call Dr
Herriot when Tricki refused to eat and lied motionless. Mr Herriot immediately took
him to his surgery, gave him a strict diet and proper exercise. In a fortnight Tricki was
transformed into a lithe, hard-muscled animal. In this era of lifestyle diseases, we should
consciously choose our diet. A proper balanced diet with fruits, proper water intake
and strict exercise regime help us keep away from diseases and attain good health.
2. The parting scene between Tricki and Mrs Pumphrey is an emotional one. All the house
helps were aroused for putting Tricki’s things in the car. Everyone was teary-eyed at
his departure. It was evident from all the pampering that Mrs Pumphrey loved Tricki

106 ACME English Practice Book GR–10 (Teacher Manual)

HF_Acme_ENG_TM_G10.indd 106 5/22/2021 5:58:50 PM


immensely. The scene throws light on the bond between human beings and their pet
animals. Animals can communicate with their owners in a very special way. They
would use their actions to convey their feelings. They get attached to their owners,
feel loved when they are happy, and feel sad on experiencing separation. Human
beings also find companions in them who they can turn to and find solace in. The
relationship between human beings and pet animals is made on the foundation of
love and trust.

Chapter 2 The Thief’s Story

ANSWER KEY

Passage Based Questions


1. (a) Hari Singh crept up to the bed. He had stolen his owner, Anil’s money. But his
conscience pricked him and he came back and crept up to the bed to put the
money back.
(b) The narrator remained still for a minute after feeling the breath in fear of being
caught. He wanted to keep the money back stealthily, without waking Anil up.
2. (a) Anil had realised that Hari Singh had tried to rob him. However, he did not say
anything to him and behaved normally to show Hari Singh that he had forgiven
him.
(b) The smile was appealing. Hari Singh was both feeling guilty and grateful towards
Anil. His smile had no dishonesty. It came on its own without effort as it was
sincere.
3. (a) (iii) Hari Singh (b) (ii) He had stolen money.
(c) (i) Anil’s (d) (i) He can live like an Arab for some time.
(e) (i) String
4. (a) (iii) Hari Singh (b) (i) marketplace (c) (ii) 600
(d) (i) He wanted to learn to write.
(e) (iv) He wanted to get education.

Short Answer Type Questions


1. After stealing Anil’s money, Hari Singh counted the notes. They were 600 rupees.
He thought he could live like an Arab for a week. But soon his conscience pricked
him and he decided to go back to Anil.
2. When Hari Singh came back to Anil’s room after stealing his money, he felt nervous. It
is easier to steal something than to return it undetected. He was also guilty of robbing
Anil of loyalty and trust he had placed in him.
3. Hari Singh slept for long in the morning that followed the night when he had attempted
to steal. He was relieved to have restored Anil’s faith. He also wanted to learn to read
and write, and was contended to have returned to Anil.
4. Hari Singh felt nervous when he saw a fifty-rupee note in Anil’s hand. He thought
his theft had been discovered and Anil wanted to confront him.

Learnwell
ACME Math–III
ENGLISH Practice Book GR–10 (Teacher Manual) 107

HF_Acme_ENG_TM_G10.indd 107 5/22/2021 5:58:50 PM


5. The night when Hari Singh stole the money was chilly. It was early November and a
light drizzle added to his discomfort when he was sitting on a bench. Soon it started
raining heavily.
6. Anil knew that his money had been stolen. The notes were wet from the previous
night’s rain. He must have felt that notes were wet when he handed over `50 to
Hari Singh.
7. Hari Singh believed that it’s easier to rob a greedy man than a careless person like
Anil. He believed that a greedy man showed fear but would accumulate wealth again.
But people like Anil would show only a hint of sadness because of the loss of trust
than money.

Long Answer Type Questions


1. When Hari Singh first met Anil at a wrestling match, he gave him an artificial smile. He
desired to make him his acquaintance in order to rob him. He managed to convince
Anil to take him as a house help. He said he could cook but he didn’t know how to
cook. When he cooked his first meal for Anil, it was so terrible that Anil had to throw
it to the dogs. Hari smiled in the most appealing way feigning innocence to flatter
Anil. Anil laughed at him and decided not turn him out. When Anil handed over a
50-rupee note to Hari, he thought that his crime had been discovered. But Anil was
understanding and considerate. He told him that he would pay Hari regularly. He
also told him that they would start writing sentences from that day onwards. Hari
gave him an appealing smile. It was a smile of guilt and gratefulness. It was also his
most earnest smile.
2. A dose of kindness and education compels Hari Singh to become a good person from
a thief. Hari Singh made acquaintance with Anil to rob him at the right time. When he
got the opportunity, he stole 600 rupees from Anil. It was Anil’s hard-earned money.
Hari left the place to go to Lucknow. He was happy that he could live a luxurious
life for at least a week. When he reached the railway station, his conscience pricked
him. He felt guilty of robbing Anil, who had trusted Hari. He believed that Anil would
show sadness on knowing that he had been robbed of his trust more than his money.
All the good memories started haunting Hari as he wandered around aimlessly. He
thought that he had also missed the opportunity to educate himself. He felt lonely
and grieved at the thought of leaving Anil. He couldn’t stop himself to go back to
Anil and return the stolen money. Anil’s attitude of helpfulness and trust transformed
Hari from a petty thief to a good person.

Chapter 3 The Midnight Visitor

ANSWER KEY

Passage Based Questions


1. (a) The narrator had developed a different image of a detective’s appearance from
what he read. He mentioned that Ausable was fat because he did not fit in the
description of the detective according to him. He also mentioned it to give readers
a humorous account of Ausable’s mundane life.

108 ACME English Practice Book GR–10 (Teacher Manual)

HF_Acme_ENG_TM_G10.indd 108 5/28/2021 3:53:24 PM


(b) Ausable spoke French and German. He had never altogether lost the American
accent he had brought to Paris from Boston twenty years ago.
2. (a) Fowler’s first authentic thrill was encountering a man with a small automatic pistol
in his hand halfway across the room. It was the kind of adventure he would read
in thrillers.
(b) The man who stood halfway across the room was Max. He held a pistol in his
hand to intimidate Ausable. He had come to take away a very important paper
concerning some new missiles.
3. (a) (iv) A writer (b) (ii) Another secret agent
(c) (ii) To snatch an important report
(d) (ii) Stiffly (e) (iii) The report was arriving.
4. (a) (iv) Fowler and Ausable (b) (ii) Secret report on missiles
(c) (i) Waiter (d) (i) Protection from robbers
(e) (iii) Extra

Short Answer Type Questions


1. Max was Ausable’s midnight visitor. Max was a rival secret agent. Ausable knew him
personally. He blinked on seeing him because he didn’t expect him to be there. It
was an expression of surprise.
2. When Ausable saw Max halfway across the room pointing a small pistol at him, he was
shocked and surprised. He blinked a few times because he wasn’t expecting Max there.
3. Ausable said that it was the second time in a month that somebody had got into his
room through the balcony. He wanted to trick Max into believing that there was a
balcony below the window.
4. According to Max, the missile report should be reaching Ausable by the night. He
was right in his estimation.
5. The repeated knocking on the main door made Max nervous. When Ausable told
him that the police would not hesitate to shoot him, Max was black with anger. He
thought that he would wait in the balcony till the police went away. Without realising
that there was no balcony, he jumped out of the window.
6. Max wanted the report concerning some new missiles that was to be delivered to
Ausable that night. Max managed to get the pass key to open the door and entered
the room secretly.
7. Ausable put Max to death by tricking him into believing that there was a window
attached to the balcony and police were knocking at the door. This made Max jump
out of the window. He was right in doing so because as a secret agent it was his duty
to safeguard the report.
8. Fowler was a young romantic writer. He had really glamorous ideas about spying
and secret agents. He loved adventure and thrill. He was disappointed to have spent
a dull evening with Ausable. Therefore, Ausable shared the information given in
secret papers with Fowler to make him believe that spying indeed includes drama
and danger.

Long Answer Type Questions


1. A calm and composed mind has the ability to think quickly and act wisely in any
tricky situation of life. Ausable proved this right. He had an instinctive and active

Learnwell
ACME Math–III
ENGLISH Practice Book GR–10 (Teacher Manual) 109

HF_Acme_ENG_TM_G10.indd 109 5/22/2021 5:58:50 PM


mind. He didn’t lose his sense of composure on finding Max in his room pointing
a small gun at him. Max was another secret agent who had come to take the secret
missile report from Ausable. Ausable with his presence of mind invented a story of
a non-existent balcony. He was alert and quick. When there was a knocking on the
door, he told Max that he had called the police for extra protection. This flustered
Max and he went towards the window in order to hide himself on the balcony. He
jumped on the non-existent balcony and died. Thus, Ausable’s problem was solved.
Ausable used his presence of mind to turn an unfavourable situation in his favour.
2. Fowler was thoroughly disappointed by the first impression made by Ausable but
Ausable proved to Fowler that appearances can be deceptive. Fowler had a different
image in his mind about detectives. He thought they were physically fit and strong.
When he saw a fat Ausable and experienced a dull evening with him, he was
disappointed. His first authentic thrill came when he encountered Max in Ausable’s
room with a small pistol in his hand. He was intimidated but was amazed to see
Ausable’s instinctive and quick thinking. He saw Ausable use his presence of mind
to protect the secret papers and convince Max to jump on a non-existent balcony,
which unfortunately led to his death. Before coming to his room, Ausable told Fowler
about the sacrifices it took to protect a secret report.

Chapter 4 A Question of Trust

ANSWER KEY

Passage Based Questions


1. (a) ‘She’ is the lady thief. ‘She’ smiled at Horace to trick him into believing that she
was the owner of the house and she had caught him red-handed.
(b) She meant that she had come just in time otherwise her family would have been
robbed by Horace. She, thus, pretended to be one of the members of the family
living at the house in Shotover Grange.
2. (a) The lady’s voice was sharp. She pretended to have caught Horace red-handed
who was about to rob her house. She wanted to protect society from such people.
(b) The lady meant by ‘men like you’ that men like Horace are a threat to the society
who rob people of their hard-earned money.
3. (a) (ii) Horace Danby (b) (iii) He was a locksmith. (c) (i) Sneeze
(d) (iv) In the drawing room (e) (iv) He had hay fever.
4. (a) (iv) The lady in red (b) (ii) thief (c) (i) Jewels
(d) (ii) She wanted to wear them. (e) (iv) Casket

Short Answer Type Questions


1. ‘Honour among thieves’ means that there is a standard behaviour that thieves follow
that doesn’t allow them to cheat each other. It is an unsaid promise. The lady in red
who was also a thief. She tricked Horace to open the safe and got him arrested.
Nobody believed Horace.

110 ACME English Practice Book GR–10 (Teacher Manual)

HF_Acme_ENG_TM_G10.indd 110 5/22/2021 5:58:51 PM


2. Horace robbed rich people once a year to buy rare books. When he met the lady
in red in Shotover Grange, he assured her that he would not hurt her and she must
forget about the incident. It made her angry. She said that society must be protected
from men like Horace who were a threat to it.
3. Horace Danby failed to get enough information about the real occupants of the
house. The lady pretended to be the owner of the house. She tricked Danby into
believing that he was helping her by opening the safe. He took out jewels for her
without wearing the gloves to ensure that he trusted her and handed them over to her.
4. Horace Danby got himself trapped in a tricky situation. When the lady pretended
to be the owner of the house, he willingly opened the safe without wearing gloves
and left his fingerprints. The police arrested him and didn’t believe his story that he
opened the safe for the lady of the house. The lady of the house turned out to be
some other woman.
5. Horace Danby was a fifty-year-old unmarried man who lived with his housekeeper.
He was a successful locksmith. He was respectable but not honest. He was fond of
collecting rare and expensive books. So, he committed only one robbery every year.
6. Horace Danby was a respectable person of the society. He was a successful locksmith
and people recognised him. He was not honest because he used to commit robbery
to satiate his desire of owning rare books.
7. Horace had been studying the room, paths and gardens of the house at Shotover
Grange for two weeks. When he entered the house, he went into the hall to cut the
wires of the burglar alarm. The burglar alarm was poorly built. He knew where the
safe was and with the experience of being a locksmith, opening the safe was not hard
for him.
8. Horace Danby was capable of opening the safe without any difficulty because he
had lived with locks and safes all his life. He was a locksmith. The burglar alarm was
poorly built. He cut the wire without facing any difficulty.

Long Answer Type Questions


1. Horace Danby studied the room, paths and gardens of the house at Shotover Grange
for two weeks and gathered information about the family. The family was in London
and the two servants of the house had gone to watch a movie that afternoon. He
was very meticulous in collecting all the little details including the house map, wiring
and the location of valuable things. Unfortunately, he focused on the details of the
things. He didn’t pay much attention on researching about the occupants of the
house. When the lady in red pretended to be the owner of the house, he fell prey to
her pretention. Horace, in his nervousness, opened the safe without wearing gloves
which led to his downfall. He later got arrested for robbing the house.
2. Aristotle had once said that our own weakness or fault in character is responsible for
tragedy in life. This statement stands true with reference to Horace Danby. Horace
was a respectable man in the society. He was a successful locksmith who lived with
his housekeeper. Though he was respectable, he was not honest. He had a hobby
of collecting rare books for which he used to rob a safe every year. That year he
planned to rob the house at Shotover Grange. Even after meticulous study of the
house, Horace failed in recognising the lady of the house. He failed in researching
about the members of the family and, thus fell prey to the tricks of the lady in red
and was arrested for the robbery. His taste for collecting things beyond his capacity
led to his downfall.
Learnwell
ACME Math–III
ENGLISH Practice Book GR–10 (Teacher Manual) 111

HF_Acme_ENG_TM_G10.indd 111 5/22/2021 5:58:51 PM


Chapter 5 Footprints without Feet

ANSWER KEY

Passage Based Questions


1. (a) ‘He’ is Griffin in the above lines. He was the scientist who managed to make
himself invisible using some drugs. ‘He’ moved to the village of Iping where he
booked two rooms at the local inn.
(b) He was eager to get away from London because he was a lawless person. He
had set fire to his landlord’s house who had wanted to eject him. He wanted to
escape from being caught.
2. (a) The horrified people were Mrs Hall and other people in the bar. They were horrified
as Griffin’s head was invisible. Griffin has taken off his bandages, whiskers,
spectacles, and even nose and had become completely invisible from neck above.
(b) Griffin was the ‘headless man’. He threw off his bandages, whiskers, spectacles
and even nose to appear headless as he had become invisible using some rare
drugs.
3. (a) (ii) Mrs Hall (b) (i) Griffin
(c) (iii) It had been swung in air.
(d) (i) She was the innkeeper. (e) (iv) Obnoxiously
4. (a) (iii) Griffin (b) (i) He had put his landlord’s house on fire.
(c) (i) The man was invisible.
(d) (iv) Both (i) and (ii) (e) (i) Adventure

Short Answer Type Questions


1. Griffin sneaked into the store around closing time. In order to warm himself, he
opened boxes and wrappers of warm clothes to fit himself in them. He ate and drank
to his full. Later, he slept on the pile of quilts. He was seen by two attendants of the
store the next morning and chased by them. He had to take off his clothes to become
invisible.
2. Griffin went to Drury Lane in the hope of finding not only clothes but also something
that would hide the empty space above his shoulders. He was successful in putting
on bandages around his forehead, dark glasses, a false nose, bushy sideburns and a
large hat from the shop.
3. After becoming invisible and committing many crimes, it was risky for Griffin to stay in
London as he feared recognition. He decided to go to a place called Iping so that he
could conduct his experiments secretly and without any disturbance in the secluded
place.
4. The unusual behaviour and appearance of Griffin aroused the suspicion of the people
of Iping. His face was covered with bandages and he would wear goggles all the time.
5. On hearing unusual noises in the study, the clergyman and his wife came downstairs.
They heard the chink of money being taken from the desk. When they investigated

112 ACME English Practice Book GR–10 (Teacher Manual)

HF_Acme_ENG_TM_G10.indd 112 5/22/2021 5:58:51 PM


further, they were shocked to see the room empty but the desk was opened and the
money was missing. Griffin carried out the burglary by being invisible.
6. When Mr and Mrs Hall entered Griffin’s room, they peeped inside. The bedclothes
were cold and the clothes and bandages that Griffin always wore were lying about
in the room. There was no one in the room. Mrs Hall was startled to hear a sniffing
sound near her ear.
7. Mr and Mrs Hall found the scientist’s door open. Griffin was not in his room. Suddenly,
Mrs Hall heard a sniff close to her ear and saw his hat leapt up in air. The chair in
the room started moving automatically and charged at her. It pushed both of them
out of the room and slammed the door.
8. Once when Mr and Mrs Hall found the door of Griffin’s room open, they tried to
investigate. The bed clothes were cold, clothes and bandages were lying about in the
room. Suddenly Mrs Hall heard a sniff near her ear then a chair leapt up in air and
charged at them. Both of them ran out of the room terrified. They believed that the
room is haunted by a ghost. But it was Griffin who was taking advantage of being
invisible and frightening them.

Long Answer Type Questions


1. Griffin was an eccentric scientist with rare talent. He had discovered some unique
drugs which could make him invisible. Despite being so talented, Griffin didn’t do any
good to society. He used his invisibility to commit crimes and harm people. Through
these misdeeds, he satisfied his own ego and fulfilled his greed. Though he was a
brilliant scientist, he was also a lawless man. His discovery made him arrogant and
turned him into a criminal. His indulged in wrongful deeds – stealing money from the
clergyman’s house, setting the landlord’s house on fire and trying to hurt the people
of Iping for his selfish motives. He was greedy, sadistic and violent. He brought utter
chaos to the society and proved a failure. His discovery could have been used for a
better purpose. But it only turned him into a monster.
2. The village constable Jaffers came to arrest Griffin who was involved in stealing money
from the clergyman’s study. After a lot of hullabaloo, when Jaffers tried to get hold of
Griffin, he threw off his bandage. Griffin became headless but Jaffers was not easily
prevented from his duty. He was adamant to make an arrest with or without head.
Then there followed a remarkable scene as the policeman was trying to get hold of
Griffin, he started taking off his clothes. With every layer being taken off, Griffin was
becoming more invisible. Then the unseen scientist knocked Jaffers unconscious and
managed to escape. Therefore, despite being serious about fulfilling his duty, Jaffers
was not able to do so and hence, this statement was ironical.

Chapter 6 The Making of a Scientist

ANSWER KEY

Passage Based Questions


1. (a) ‘He’ refers to Richard H. Ebright in this extract. He possessed great curiosity to
know about things and a bright mind.

Learnwell
ACME Math–III
ENGLISH Practice Book GR–10 (Teacher Manual) 113

HF_Acme_ENG_TM_G10.indd 113 5/22/2021 5:58:51 PM


(b) His mother encouraged his interest in learning by taking him on trips, bringing
him telescopes, microscopes, cameras, mounting materials, and other equipment.
This helped him in many other ways.
2. (a) ‘That’ refers to Ebright’s collection of all twenty five species of butterflies found
around his hometown.
(b) The book told the speaker (Richard Ebright) how the monarch butterflies migrated
to Central America. It opened the world of science to the eager young Ebright.
3. (a) (ii) Ebright (b) (iii) butterfly
(c) (iii) For experimenting (d) (iv) Eggs, caterpillar, pupa, adult
(e) (iv) Development
4. (a) (i) DNA is the blueprint for life.
(b) (iv) National Academy of Science
(c) (iii) By looking at the X-ray photos of the chemical structure of hormones
(d) (iii) simplest unit of a chemical substance
(e) (iii) Richard’s room-mate

Short Answer Type Questions


1. Ebright’s advanced experiment on the monarch pupa helped him attain the first place
in zoology at the International Science Fair. It also gave him the chance to work at
the army lab as well as the U.S. Department of Agriculture’s laboratory.
2. Ebright made great contribution to the world of science. He discovered that the
hormone produced from the gold spots of a pupa helps in the formation of butterfly
wings. He also discovered how the cell can ‘read’ the blueprint of its DNA. DNA is
the blueprint for life which control heredity.
3. Ebright wanted to study migration of monarch butterflies. Therefore, he would catch
a female monarch, take her eggs and raise them in his basement. He would then tag
them and let them go.
4. At the county science fair, he showed slides of frog tissues under a microscope and
lost the competition. He realised that winners showed real experiments not just a
simple neat display.
5. Ebright’s project for the county science fair that earned the third position was to test
the theory that viceroy butterflies copy monarch butterflies. He observed that viceroys
look like monarchs because monarchs don’t taste good to birds. This way viceroys
save their lives.
6. Ebright was a straight-A student. He had keen interest in science. He turned a lot
of his energy towards debating and Model United Nations clubs. He was inquisitive
and had a will to win for the right reasons. He was a person who put in three or four
hours at night doing debate research besides doing all his research with butterflies
and his other interests.
7. Ebright tagged butterflies and let them go to study migration of monarch butterflies.
But tagging butterflies was a tedious work. Moreover, he could recapture only two
butterflies out of the many that he let go off. So, he stopped tagging butterflies.
8. Besides Science, Ebright had pursued many other interests too. He was a champion
debater, a good public speaker and a good canoeist. He was an expert photographer
too particularly of nature and scientific exhibits.

114 ACME English Practice Book GR–10 (Teacher Manual)

HF_Acme_ENG_TM_G10.indd 114 5/22/2021 5:58:51 PM


Long Answer Type Questions
1. Ebright success belonged to his inquisitiveness and learning ability which helped him
earn many prizes during high school. When he first participated in the county science
fair, he couldn’t win anything. He realised that winners did real experiments than make
a neat display. The zeal of learning more and winning helped him improve himself.
The following years, he conducted many experiments. His experiment with monarch
and viceroy butterflies helped him earn the third position in the county science fair.
The discovery of an unknown insect hormone earned him a chance to work at the
entomology laboratory of the Walter Reed Army Institute of Research. Then there
was no stepping back. He went on to do experiments which showed how the cell can
‘read’ the blueprint of its DNA. Thus he became a renowned scientist. Real talent is
worked upon since childhood. His scientific curiosity and zeal to learn more helped
him become a great scientist.
2. Richard Ebright’s had a curious mind since childhood. In kindergarten, he started
collecting butterflies, rocks, fossils and coins. By the time he was in the second grade,
he had collected all twenty-five species of butterflies found around his hometown.
When he was in the seventh grade, he participated in the county science fair and
learnt the first lesson of his life. The result of the fair taught him that winners did real
experiments rather than make a neat display. He failed once but it never stopped him
from making greater efforts to achieve success. The next year his science fair project
was testing the theory that viceroy butterflies copy monarch butterflies. This project
was placed first in the zoology division and third overall in the county science fair.
Continuous research and experimentations won him great honours and prizes locally
as well as international. His continuous hard work and scientific bent of mind helped
him become a famous scientist. Therefore, it is said that ‘Failure is a step towards
success.’

Chapter 7 The Necklace

ANSWER KEY

Passage Based Questions


1. (a) Mr Loisel is the speaker of these lines. He dictated his wife to write a letter to Mme
Forestier that the clasp of the necklace had been broken and they would have
it repaired. This would give them some time to find the necklace or replace the
original one.
(b) The necklace which Matilda borrowed from Madam Forestier to wear to the ball
is being referred to here. The necklace had been lost.
2. (a) Madam Forestier is the ‘friend’. She was astonished to be addressed by a common
personage. She couldn’t recognise Matilda as she looked like a poor old woman.
(b) The ‘friend’ failed to recognise Matilda, the other person, because she had changed
a lot. Mme Forestier met Matilda after ten years and was shocked to see her in
such a poor state.

Learnwell
ACME Math–III
ENGLISH Practice Book GR–10 (Teacher Manual) 115

HF_Acme_ENG_TM_G10.indd 115 5/22/2021 5:58:52 PM


3. (a) (i) Mr and Mrs Loisel (b) (iii) They couldn’t find any cab.
(c) (i) Party (d) (ii) Enthusiasm of the party had ended.
(e) (iv) Exhaustedly
4. (a) (i) Matilda and her husband (b) (iii) Lost necklace
(c) (i) They borrowed money on high interest rates.
(d) (iii) For working hard and paying off the loan (e) (ii) satisfactorily

Short Answer Type Questions


1. During dinner Matilda would think of elegant dinners, of shining silver; of the exquisite
food served in the marvellous dishes. She always felt that she was born for all delicacies
and luxuries.
2. On the day of the ball, Mme Loisel was a great success. She was prettiest of all –
elegant, gracious and joyful. Everyone paid attention to her, asked her name and
wanted to dance with her.
3. When they noticed that the necklace had been lost, they looked at each other in
horror. Matilda uttered a cry of shock and Loisel was perturbed. Loisel rushed out
immediately to look for the necklace in the streets. When he found nothing, he returned
home with a pale face. At the end of the week, they had lost all hopes of finding the
necklace.
4. Loisel was perturbed when Matilda demanded a new dress of the ball. He had saved
four hundred francs to buy a new gun for some hunting parties. Initially, he turned
pale after knowing the cost of the dress but then sacrificed his savings for his wife’s
happiness.
5. Loisel suggested Matilda to go to her friend, Mme Forestier and borrow some jewels.
Matilda went to her friend’s house, related her story of distress and chose a diamond
necklace from her collection of jewels.
6. After realising that the necklace had been lost, Mr. Loisel tried to search it at every place
where they had walked on foot. When he failed to find it, he reported the matter to
the police and also went to the cab office. He also advertised about the lost necklace
in the paper.
7. Matilda was born in a family of clerks and was married to a clerk. She believed herself
to be born for all delicacies and luxuries. She would always dream of elegant dinners,
of shining silver, of the exquisite food served in marvelous dishes. Her dreams caused
her unhappiness.
8. Matilda did not like to visit her rich friend. Her unhappiness regarding her family
background and marriage made her suffer from despair and disappointment. She
would come home and weep for whole days.

Long Answer Type Questions


1. We have to live life as per the dictates of fate. This is true in context of Matilda’s life
story. Matilda belonged to a family of clerks and was also married to a clerk. She was
pretty but her lack of privileges made her unhappy. She always dreamt of leading
a lavish and luxurious life. When Mr Loisel shared the ball invitation with her, she
found the opportunity to live a lavish life. She got herself a new dress and borrowed
a necklace from her friend. At the ball, she was intoxicated with pleasure. But her
happiness was short-lived. She lost the necklace that she had borrowed from her

116 ACME English Practice Book GR–10 (Teacher Manual)

HF_Acme_ENG_TM_G10.indd 116 5/22/2021 5:58:52 PM


friend. Replacing the necklace cost her ten years of hard work. She had to do all
housework herself and had to leave all comforts of life to make ends meet.
2. Mr Loisel worked as a clerk in the office of the Board of Education. He was
unpretentious and humble. He believed in simple living. He lived in an ordinary house
with dilapidated walls and furniture, and ate simple food but never complained about
anything. He was sacrificing. When Matilda demanded a new dress for the ball, he
gave him the money he was saving for himself to buy a hunting gun. Even when
she lost the necklace, he supported her. He went to look for the necklace all over the
place they had travelled and reported it to the police. He also arranged money for the
necklace replacement and worked evenings, putting the books of some merchants in
order, and nights he often did copying at five sous a page. And this life lasted for ten
years. He wasted ten golden years of his life living in dire poverty due to his wife’s
foolish aspirations.

Chapter 8 The Hack Driver

ANSWER KEY

Passage Based Questions


1. (a) The narrator found his work unpleasant because he had to go to dirty and shadowy
corners of the city to serve summons. He would often get beaten up by some of
the larger men.
(b) Though the narrator worked in a good law firm, his work was not to prepare legal
briefs but to serve summons, like a cheap detective. He considered fleeing to his
hometown as there he could have been a real lawyer right away, without going
through the unpleasant training period.
2. (a) The narrator thought that the hack driver was a kind person as he helped him
find Oliver Lutkins. He was friendly and warm. The narrator didn’t know anyone
in the town. So, he felt comforted by the hack driver.
(b) The ‘business’ being referred to here is earning money through hack driving. He
knew that he was being nice to earn money by transporting him through the town.
3. (a) (i) Bill (b) (i) Lutkins
(c) (iii) To serve summons (d) (iv) Walking down the main street
(e) (ii) to remain in a place longer than expected
4. (a) (iii) The narrator (b) (i) Search for Oliver Lutkins
(c) (iii) Represent court (d) (iv) Impressed
(e) (iv) She was his mother.

Short Answer Type Questions


1. The narrator told Bill that the purpose of his visit was to serve summons on Oliver
Lutkins. Bill was calm and friendly. He told the narrator that he saw Lutkins an
hour ago. He also told him that it was not easy to find him as he was always up to
something.

Learnwell
ACME Math–III
ENGLISH Practice Book GR–10 (Teacher Manual) 117

HF_Acme_ENG_TM_G10.indd 117 5/22/2021 5:58:52 PM


2. Bill told him that he saw Lutkins an hour ago. He also told him that Lutkins could be
found at Fritz’s shop. He offered the narrator his help in finding Lutkins. The narrator
was happy at his response and his offer.
3. Bill was helping the narrator in finding Lutkins. He knew the places Lutkins would go
to and the activities he did. He was appreciative of Lutkins’s smartness and talent of
deceiving people. The narrator believed had Bill been a policeman, he would have
arrested Lutkins with utmost respect and regret. Bill told the narrator that Lukins played
a lot of poker. He added that Lukins was good at deceiving people. The manner in
which he talked about Lutkins made the narrator think that the former appreciated
Lutkins talent for dishonesty.
4. Bill took the narrator to Fritz’s shop to look for Lutkins. He believed that Lutkins
would probably be playing his poker game at Fritz’s. Bill led him there and advised
him to hide behind him for Lutkins might become suspicious and make an escape.
5. At Gustaff’s shop, one of the customers told the narrator that he had seen Lutkins in
the main street near the hotel. At Gray’s barber shop, they were late by five minutes
and were told that Lutkins had probably gone to the poolroom. At the poolroom,
they were told that Lutkins left after buying cigarettes.
6. When Bill took the narrator to Lutkins’ mother’s farm, his mother shouted at them that
she knew nothing about Lutkins. When Bill made the narrator sound very important,
she went to the kitchen, took out a hot iron and threatened to burn them. However,
they did search the house by peeping stealthily through all the windows and found
no one.
7. Lutkins’ mother took out a hot iron from the stove and threatened to burn Bill and
the narrator. This disrespectful treatment frightened them. Therefore, Bill asked the
narrator to get out immediately from Lutkins’ mother’s farm.
8. The narrator hated city life and his job of serving summons. He was not unhappy
for not being able to find Lutkins. He was happy to have found Bill, who gave him
a ride through the village.

Long Answer Type Questions


1. The narrator was happy being sent to New Mullion. He was expecting sweet and
simple country village but was severely disappointed on reaching there. Its streets
were rivers of mud, with rows of wooden shops either painted a sour brown or not
painted at all. He met Bill whom he found friendly and kind. Bill offered to help
him in finding Lutkins. During the search, Bill showed the narrator the beauty of the
village and the village people. He painted the picture of its people with his words.
The narrator was touched by Bill’s kindness and helpful nature which made him
grow fond of the village people and the village. However, all this was a sham. The
people of the village helped Lutkins to fool the lawyer from the city. They were not
as innocent as the narrator thought them to be.
2. Lutkins was not right in befooling the lawyer and earning money by using unfair
means. His actions and behaviour show that he did not bother about the law at all.
He shrewdly manipulated the narrator for his favour. The lawyer was easily taken by
his lies and pretentions. One should not take everything on their face value. We should
minutely observe and judge every action before believing someone completely. We
should be capable of conducting our enquiries ourselves, instead of following others’
hints and directions. The lawyer was befooled because he believed Lutkins without

118 ACME English Practice Book GR–10 (Teacher Manual)

HF_Acme_ENG_TM_G10.indd 118 5/22/2021 5:58:52 PM


knowing his credibility and let him do the enquiry. He didn’t doubt Lutkins actions
and kept following him. As a result, he could not serve the summons on Lutkins and
became a laughing stock for the entire village.

Chapter 9 Bholi

ANSWER KEY

Passage Based Questions


1. (a) The speaker wished Bholi to be dressed up in decent clothes for he was taking
Bholi to the new village school.
(b) This dialogue reflects that Ramlal is very conscious about his goodwill. But he
was a bad parent as he never paid attention to his child who was mentally weak
and hence needed more support and love.
2. (a) ‘She’ is Bholi in this extract. When the teacher asked Bholi about her name, she
could not speak properly. She began stammering and couldn’t say her complete
name. So, she began to weep.
(b) When she stammered, the girls in the class started laughing at her. Bholi felt
humiliated by the treatment and did not dare look up.
3. (a) (iii) To perform the opening ceremony of a girls’ school
(b) (iii) Send his daughters to school
(c) (ii) Girl education was not popular in the village.
(d) (i) To set an example for villagers
(e) (ii) spokesperson
4. (a) (i) Sulekha (b) (iv) All of these (c) (iii) Bholi
(d) (iii) The teacher showed confidence in her. (e) (iv) Throbbing

Short Answer Type Questions


1. No, wealth is not the only criterion for being a perfect bridegroom. Bishamber was
not a suitable bridegroom for Bholi. He was greedy and mean. He demanded dowry
from Bholi to compensate for the pockmarks on her face. He disregarded the fact
that he was old and lame. He was an awful person.
2. Bishamber Nath was a man of wealth. He was a prosperous grocer. On the wedding
day, he came with a big party of friends and relatives. A brass band was playing a
popular tune from an Indian film. Bishamber was riding a decorated horse. Bholi’s
family was happy to see such pomp and grandeur.
3. Bishamber was shocked when he saw pockmarks on Bholi’s face. He even demanded
five thousand rupees as dowry from Ramlal to marry Bholi and compensate for her
disfigured face.
4. On Bishambar’s demand of five thousand rupees for marrying Bholi, Ramlal went
and placed his turban at Bishambar’s feet. He asked him not to humiliate them and
offered him two thousand rupees.

Learnwell
ACME Math–III
ENGLISH Practice Book GR–10 (Teacher Manual) 119

HF_Acme_ENG_TM_G10.indd 119 5/22/2021 5:58:52 PM


5. The ill-treatment of her parents at the hands of Bishamber made Bholi change her
attitude towards him. She refused to marry him to save her and her father Ramlal’s
self respect.
6. Bholi looked at her with extreme anger. She told her that everybody took her as a
dumb-driven cow and hence, wanted to marry her off. But now the dumb Bholi had
a voice and she would use it to defend her and her family’s self-respect.
7. Ramlal stood rooted to the ground, his head bowed low with the weight of grief
and shame. He then turned to Bholi and said that no one would marry her. He was
worried what would happen to Bholi.
8. Bholi consoled her father saying that she would take care of both her parents in their
old age. She would teach in the same school that had made her learn so much.

Long Answer Type Questions


1. It is the responsibility of the teachers to make their students active, bold, confident and
fearless. Bholi’s teacher played an important role in shaping Bholi’s life. The teacher
consoled Bholi when she was being laughed at for her stammer. She encouraged her
to complete her sentences. She affectionately supported Bholi. The relationship Bholi
shared with her teacher and the confidence the teacher had in her, helped Bholi to
be filled with hope for a new life. The teacher treated Bholi with utmost respect and
love. Even during her marriage fiasco, she supported her. When Bholi asked her if
she could teach in the school, she replied in affirmation. The teacher’s eyes had the
light of a deep satisfaction that an artist feels when contemplating the completion of
her masterpiece.
2. In the beginning of the story, Bholi was an under-confident and a shy girl. When she
was ten-month old, a sudden fall damaged her brain. This made her a little slow. She
also had pockmarks on her face and a habit of stammering. People always ignored her
and made fun of her. When she joined the school, her teacher helped her to control
her stammer and gain confidence. She motivated her when everybody laughed at her.
She filled her heart with new hope and a new life. Her efforts helped Bholi transform
into a confident woman. On her wedding day, when Bishamber asked for dowry as
compensation for her pockmarks and disrespected her family, she raised her voice.
She didn’t shy away from showing her anger against the ill-treatment meted out to
her and her family. She promised to support her parents in their old age by teaching
in her school. Thus, the character took a 360 degree turn by the end of the story.

Chapter 10 The Book That Saved the Earth

ANSWER KEY

Passage Based Questions


1. (a) Historian is the speaker of these lines. The speaker is referring to Martian invasion
of Earth that happened centuries ago.
(b) Think-Tank led the invasion. A book of nursery rhymes stopped the invasion.

120 ACME English Practice Book GR–10 (Teacher Manual)

HF_Acme_ENG_TM_G10.indd 120 5/22/2021 5:58:53 PM


2. (a) Think Tank is the speaker here. He wants communication with space exploration
team that had gone to a funny planet so that they could have their liberal rulership
on it.
(b) Earth is the ‘ridiculous little planet’ referred to by the speaker. The speaker plans
to invade Earth and put it under his generous rulership.
3. (a) (iii) Omega (b) (ii) To invade the Earth
(c) (i) A storage barn (d) (ii) Shelter (e) (iv) arrived safely
4. (a) (iii) Books (b) (ii) Omega
(c) (ii) For communication (d) (i) A large, colourful book (e) (i) Clarify

Short Answer Type Questions


1. No, the space people weren’t able to decipher the code of the book. Think-Tank wanted
to give the probe crew vitamins to increase their intelligence so that they could find
out what was in the books.
2. Think-Tank reached to the conclusion that the earthlings have discovered how to
combine agriculture and mining. They can grow crops as well as explosives. He said
that there’s no time for levity as the discovery was serious.
3. Think-Tank thought that it was him in the picture and got so frightened that he screamed
out of fear. He anticipated that the Earthlings would soon invade Mars and take over
his reign from him.
4. Think-Tank ordered the invasion fleet to evacuate the entire planet of Mars because
he thought that the Earthlings would invade Mars and kill him. They wanted to go
hundred million miles away from Mars. They headed to Alpha Centauri.
5. The misinterpretation of the nursery rhymes helped in saving Earth from Martian
invasion. Think-Tank, the Martian commander-in-chief misinterpreted the rhyme
‘Humpty Dumpty’. He thought that the Earthlings wanted to kill him and invade Mars.
Therefore, he decided to escape from Mars to Alpha Centauri which was millions
miles away from Mars.
6. The Earthlings and the Martians resumed their friendly relations when Noodle, the
clever Martian took over the rulership of Mars from the arrogant and dimwitted Think-
Tank. The Earthlings taught them how to read, too, and established a model library
in their capital city of Marsopolis.
7. The ‘strangest thing’ that a book from the twentieth century did was that it saved Earth
from the Martian invasion. It was strange because Earth saved by the misinterpretation
of nursery rhymes by the Martians.
8. A historiscope is a device to look into history. It can be used to see past incidents.

Long Answer Type Questions


1. All meanings depend on interpretation holds true about Think-Tank and his interpretation
of things found on Earth. Think-Tank took decisions in an amateurish way. He was
self-obsessed and immature. He excelled in misinterpreting and misrepresenting things.
When his team discovered books on Earth, they could not comprehend what they
were. He repeatedly misinterpreted the books and their contents. He thought them
to be sandwiches. When Noodle told him that the books were for communication, he
asked the team to read out the content. His misinterpretation of ‘Humpty Dumpty’
helped save Earth from invasion. He misinterpreted ‘had a great fall’ and gave a cry

Learnwell
ACME Math–III
ENGLISH Practice Book GR–10 (Teacher Manual) 121

HF_Acme_ENG_TM_G10.indd 121 5/22/2021 5:58:53 PM


of fear. He ordered to evacuate the entire planet instantly. He assumed that he would
be killed and Mars would be taken over by the Earthlings soon. He decided to run
away and save his life. He boarded his space shuttle and left Mars without delay.
2. Theoretical knowledge without practical experience leads a person towards failure.
Think-Tank had little regard for the inhabitants of Earth and planned to invade the
planet. But his lack of knowledge about Earth led him to make wrong interpretations.
He came to know through his delegates who had landed on earth that they were
not able to understand what the strange things were in the Centeville Public Library.
Think-Tank ordered the crew to consume the vitamins to enable them to decode the
information. Much to the amusement of the readers, he himself misinterpreted the
nursery rhymes, and thought that the Earthlings were really powerful creatures and
were planning an invasion on Mars. He thought Humpty-Dumpty in the rhyme was
an allegory for him and left Mars and escaped to Alpha Centauri to save his life.

Assessment of Speaking and Listening

ANSWER KEY

SPEAKING TEST-1
Stage 1: Introduction
To be attempted by students on their own.

Stage 2: Presentation
(Candidate A : I want to express my views on the topic “Importance of Trees”.)
Without trees human existence on earth is almost impossible. Trees provide us with two basic
elements of survival – oxygen and food. With human evolution, trees started to be used for
medicines, shelter and other commercial purposes. Trees protect the earth from soil erosion,
fires, floods, and wind. They are used as fences, windbreaks, and barriers. They help slow
down water evaporation from soil and act as nature’s air conditioner. They provide habitat to
a diverse array of wildlife from its lower levels up to the very high canopies. Trees and shrubs
also improve soil and water conservation, store carbon, moderate local climate by providing
shade, regulate temperature, provide habitat to wildlife and improve the capacity of the land
to adapt to climate change.
(Candidate B: What is more important – environment or industry?)
Candidate A : The very existence of industry is dependent on the environment. Moreover,
industry cannot provide us primary goods like air, water, food, etc. It can be provided only by the
environment. Industry can provide us secondary goods by making use of these primary resources.
(Examiner: In what manner do trees impact human life?)
Candidate A : Without trees, human life on earth is not possible. During the process of
photosynthesis, trees take in carbon dioxide and produce the oxygen we breathe. They stabilise
the soil, regulate the temperature, provide protection against flood and droughts and provide
habitat to a large number of animals and birds. They also provide the materials for infrastructure
and shelter.

122 ACME English Practice Book GR–10 (Teacher Manual)

HF_Acme_ENG_TM_G10.indd 122 5/22/2021 5:58:53 PM


(Examiner: Can Internet help in raising awareness about pollution and environment degradation?
How?)
Candidate A : Yes, the Internet can help in raising awareness about pollution and the environment
degradation. Social media apps like Twitter, Facebook and Instagram can be used to share images
and videos, activities undertaken by various groups, events done to promote tree planting,
creating awareness about need for saving certain forest areas, news about environmental issues
to sensitise people about environmental damages and the need to saving it. Mass appeal can
be made through the internet.
(Candidate B : My topic is ‘Brain Drain’.)
Brain drain is the migration of talented people like doctors, engineers, lawyers, teachers, IT experts
and technicians from their home country to a foreign country for higher remuneration, better
prospects and improved working conditions. It is troublesome for developing countries like India.
Brain drain can have many reasons, for example–political instability, lack of economic
opportunities, lack of health facilities, lack of social mobility, etc. These factors prompt skilled
and talented workers to leave source countries for places that offer better opportunities. Brain
drain is unfortunate for the source country because it loses skilled workers. A country spends
time and efforts to train these workers only to lose them to other countries. Brain drain can
change the skill structure of the labour force, cause labour shortages, and affect tax revenues.
Those who emigrate tend to be most skillful. If they had stayed in their country, they might
have set up a business which would contribute to economic growth and create employment
for others.
(Candidate A: How is brain drain becoming a cause for concern in India?)
Candidate B : The rising brain drain of Indian academicians, medical experts and IT professionals
is a growing cause for concern for the economic setup of India. When Indian talent migrates
abroad in search of greener pastures, the country ends up losing its major skilled workforce.
Brain drain hampers the growth of a nation as talented people move to other countries. The
resources spent on them by the country go to waste as skilled professional help the emigrated
country to progress.
(Examiner: Is there any positive effect of brain drain?)
Candidate B : A moderate amount of brain drain can be beneficial for the country of origin
because it results in more educated workers. Those who are left behind get the opportunities.
Those who establish themselves in other countries may engage with their home countries through
business visits. A lot of skilled professionals send home a large sum of money that benefits those
who stay back.
(Examiner: How do you think brain drain be stopped/lessened?)
Candidate B : Social, political and economic conditions should be improved to lessen brain
drain. Skilled people should be provided with satisfactory job opportunities. Education facilities
need to be improved. The standard of living should be improved.

Stage 3: Problem Solving


(Examiner: Your time is over now. Could you agree on a solution? What is it?)
Candidate : Overuse of mobile phones may result in deterioration of health as well as affect
relationships. Children need to go out and socialise with their friends. They should spend more
time with their family. Families should have food together or they can watch TV together. They
should make an effort to interact with each other every day in a healthy manner.

Learnwell
ACME Math–III
ENGLISH Practice Book GR–10 (Teacher Manual) 123

HF_Acme_ENG_TM_G10.indd 123 5/22/2021 5:58:53 PM


(Examiner: How can you motivate people to adopt a healthy approach towards the use of
technology?)
Candidate : Technology should be used in a balanced way. To motivate people to adopt a healthy
approach towards the use of technology, they should be guided how to make productive use of
technology for good purposes like education, marketing, spreading awareness, etc. They should
be made aware about the negative impact of technology on health and also about cyber crimes.
(Examiner: How can we maintain a balance between technology’s use and abuse?)
Candidate : Technology must be used for beneficial purposes such as education, entertainment,
banking, making purchases, etc. But we must keep in mind that moderation is the key. Anything
that is overused affects our lifestyle and intellect. Technology should make our life easier but it
has to be used with caution.
(Examiner: What is the impact of frequent use of mobile on health?)
Candidate : Apart from risk of cancer, excessive use of mobile phones affects our nervous system.
They may be a reason for headaches, decreased attention, shortness of temper, sleep disorders
and depression. These are more prominent among teenagers.

SPEAKING TEST-2
Stage 1: Introduction
To be attempted by students on their own.

Stage 2: Presentation
(Candidate A : I want to express my views on the topic “Over Population and Unhealthy Living
Conditions”.)
Overpopulation is one of the major threats being faced by the nation today. The excessive rise
in population has made a mockery of all our plans for development. Poverty and unhealthy
living conditions escalate with the rise in the number of people. The growing population is
hampering the living standards of the people. It is very difficult to provide good health services
to so many people. A large number of people suffer from various diseases because of lack of
medical facilities. Inaccessible, inadequate, or non-existent health care makes the poor exposed
to infectious diseases. High densities of population increase the chance of the emergence of new
pandemics and epidemics. Freshwater sources are limited. Overpopulation creates immense
pressure on the world’s freshwater availability. The unavailability of freshwater to a large number
of people leads to death and diseases.
(Candidate B: Should India adopt a strict one-child policy to control population?)
Candidate A: India should implement a one-child policy to control the population. Not only
will the policy help control head counts, but it will also provide many advantages socially and
economically. One-child policy can help reduce population growth in India which in turn will
lead to more and better opportunities and access for people in terms of education, employment,
health facilities and living standards.
(Examiner: How is overpopulation harmful for the environment?)
Candidate A : Overpopulation is harmful for any nation as it is associated with negative impact on
the environment due to over-farming, excessive exploitation of natural resources, deforestation,
and air, water and soil pollution and global warming.
(Examiner: How does overpopulation hinder the development of a country?)
Candidate A : The larger the size of population, the larger number of mouths are to be fed. It
puts burden on existing resources. If other resources are not available in sufficient quantities
then it leads to starvation, disease, death and conflict.

124 ACME English Practice Book GR–10 (Teacher Manual)

HF_Acme_ENG_TM_G10.indd 124 5/22/2021 5:58:53 PM


(Candidate B : My topic is “Importance of Books”.)
Books play an important role in our life. It has been truly said that books are our best friends.
They demand nothing from us and give us plenty of joy. We also learn a lot about other people,
other cultures, human psychology, etc. after reading books. Reading a book takes us into a
different world of imagination. Good books have a positive impact on our thought process and
actions. Books give hope and make us happy when we feel sad or depressed. They remove our
ignorance, make us aware and add to our knowledge. They give exposure to various experiences
and sharpen our intellect.
A good book is our ‘friend, philosopher and guide’. When we read a good book, we forget our
worries and tensions. We get transported into the land of beauty, imagination and happiness.
A well-read man is respected by all. He has the capability to entertain people and talk about
many topics. A room is alighted with the wisdom of such persons and one does not feel boredom
in their company. This is another advantage of reading books.
(Candidate A: Which is your favourite book and why?)
Candidate B : My favourite book is the Harry Potter series. It is a series of seven fantastical novels
written by J.K Rowling. It contains adventures and struggles of the lead character, Harry Potter
which is very thrilling and inspiring. It encourages the readers to be brave to face the challenges
in life. It also depicts importance of friendships in life.
(Examiner: What are the benefits of reading?)
Candidate B : Reading is important because it helps us to exercise our mental faculties. It
gives us endless knowledge. We learn many important life lessons from reading about different
characters and personalities. Books can provide different kinds of information, stories, thoughts
and analyses which add to our knowledge and experience.
(Examiner: How can we encourage students to take interest in reading?)
Candidate B : To develop reading habit in students, it is essential to create a reading space. A
library with a selection of books on varied topics is a must. Students should be encouraged to
read selected material and then the teacher can discuss with them the books in relation to other
books, movies, news items, or TV shows. Initially the students should be motivated to read what
they are interested in.

Stage 3: Problem Solving


(Examiner: Your time is over now. Could you agree on a solution? What is it?)
Candidate : Lack of physical activity can lead to development of many cardio-vascular diseases
and certain cancers later in life. It can lead to obesity. It can also add to feelings of anxiety and
depression. To solve this problem, the students should be provided adequate physical exercise.
Physical education should be made a necessary part of education. Yoga should be made a
compulsory part of morning assembly. Students can be given examples of good sports players
and their achievements.
(Examiner: Why do you think children do not indulge in regular physical activities?)
Candidate : Lack of time to exercise due to hectic school schedule, lack of space for outdoor
activities, lack of self-motivation, lethargy induced by eating junk food and availability of electronic
means of entertainment are the reasons children do not indulge in regular physical activities.
(Examiner : What are some mental benefits of playing sports?)
Candidate: Playing sports can boost mood, reduce depression and stress, improve sleep, keep
your mind sharp, increase self-confidence, etc.

Learnwell
ACME Math–III
ENGLISH Practice Book GR–10 (Teacher Manual) 125

HF_Acme_ENG_TM_G10.indd 125 5/22/2021 5:58:53 PM


(Examiner : Should sports be considered as important as studies?)
Candidate : Yes sports should be considered equally important as studies. Academic excellence
is central to education, but participation in sports is equally important. By playing sports a
person can focus more on studies. Studies can enhance our intellectual skills but sports teaches
us time management, self-confidence, teamwork, leadership skills, etc. apart from providing us
a healthy body to gain academic excellence.

SPEAKING TEST-3
Stage 1: Introduction
To be attempted by students on their own.

Stage 2: Presentation
(Candidate A : I want to express my views on the topic “The Menace of Drug Addiction”.)
Drug addiction has become a problem of gigantic proportions for our society. Drug abuse
has become quite common in almost every country nowadays. This evil spreads easily and
increasingly in the places like hostels and campuses where a huge number of students become
easy victims. Many of the youngsters and students have easy access to drugs of various kinds.
Drugs have been used since long for medical purposes. Nowadays, they are being abused by
our young generation for recreational purposes. Sometimes young boys and girls take drugs and
narcotics due to peer pressure. Sometimes, they start taking drugs to alleviate pain but gradually
become addicted to this bad habit. In India the number of drug abusers has been increasing at
an alarming rate. Several reasons are responsible for this menace. The most observed reason
is the lack of enough parental attention, love and care. In unloving families, young people go
astray and find relief in drugs. Drug addicts or sellers in the name of friendship mislead them
and induce them to find solace by taking drugs and narcotics. Once tasted, they get addicted
to it very soon.
Once addicted, they become so immoral and helpless that they don’t hesitate even to commit
crimes of various types in order to get drugs. More de-addiction camps and rehabilitation centres
should be established by the government to provide medical treatment for the addicts. Also,
strict action should be taken against those who sell drugs.
(Candidate B: How can you spread awareness against drug addiction?)
Candidate A : The young boys and girls must be made aware of the fact that they will be crippled
for life because of drug addiction. They should be guided about the value of health, family
through the means of camps, group discussion, counselling sessions, plays, etc. Their parents
and friends should participate actively in order to help them to give up on this addiction.
(Examiner : Do you think meditation or alternative therapy can be helpful in controlling drug
addiction?)
Candidate A : Yes, meditation or alternative therapy can be helpful in controlling drug addiction.
Meditation and other alternative therapy helps in improving self-control and suppressing the
cravings for drugs.
(Candidate B : My topic is ‘Changing Face of Mass Media’.)
Human communication has been changing and developing since its origin. Till the middle of
the last century, newspaper and magazine were the principal source of mass communication.
Later, radio became a popular source of mass communication and entertainment. Individuals
frequently tuned in to radio to know what was happening in the world. Newspapers still remained
popular. As soon as televisions were invented, TV turned into the fundamental source of the mass
communications. Since TV had the qualities of both radio and newspaper, it had a much wider
appeal. In 1962 with the launch of the first satellite, people got access to live news. Nowadays
126 ACME English Practice Book GR–10 (Teacher Manual)

HF_Acme_ENG_TM_G10.indd 126 5/22/2021 5:58:53 PM


more than 2,500 satellites orbit the earth to track weather, military, give users accurate directions
through GPS, etc. These days, the media has become so advanced that you can chat, video
call, send or receive documents at a click of a button. Various social media apps offer a huge
range of means of entertainment, education and business opportunities catering to the needs
of all age groups.
(Candidate A : How does mass media affect an individual’s thinking?)
Candidate B : Mass media has always had a great influence on many aspects of human life.
It affects voting patterns, affirms or changes views and beliefs, helps people engage in certain
activities, influences their buying habits, etc. Mass media can bring changes in people’s thinking
and mindset and also reinforce an existing belief.
(Examiner : Why do you think mass media exaggerates some news?)
Candidate B : Mass media works like business. A lot of media channels see profit earning as
their basic purpose. For this, they need viewership which they try to get by exaggerating news
to keep people hooked on to their channel for entertainment.
(Examiner : Do you think mass media is capable of bringing about change in the society?)
Candidate B : Yes, mass media not only affects intellectually but also influences emotional, social
and cultural aspects of the audience. It has the power to change opinions, build thinking in a
particular direction. So, the mass media is capable of bringing about change in society.

Stage 3: Problem Solving


(Examiner : Your time is over now. Could you agree on a solution? What is it?)
Candidate : We can motivate our friend by making him realise that it is unethical to cheat in
examination and nothing can substitute hard work. We can also tell him the consequences of
adopting unfair means to clear an exam. He/She must realise that such shortcuts to pass exams
will not help in attaining a good career.
(Examiner : How would you spread awareness in your school against cheating in exams?)
Candidate : We can try to appeal to students’ conscience. With the help of seminars, workshops,
nukkad natak, posters, they can be made to realise that cheating may serve them in short term
but it will not lead to success in future. In fact it’s going to hamper their progress and stop them
from achieving their goals.
(Examiner: Suggest some measures to control cheating during examinations.)
Candidate : Some measures to control cheating during examinations are proper supervision by
invigilator, proper checking of students before exam, making proper seating plan for exams, etc.
(Examiner: Do you think it is necessary to pass examination on the basis of merit to be successful
in life?)
Candidate : Yes, it is necessary to pass examination on the basis of merit to be successful in
life. If we don’t have enough knowledge, we won’t be able to pass the entrance examination
for higher studies. Cheating will affect one’s career as lack of knowledge will hamper one from
cracking the job interviews. Even later in career, his/her work efficiency will be poor.

LISTENING TEST–1
Task–1
1. increasing visitor traffic and improving online presence of the business website in
every search engine
2. lesser rates as compared to other marketing agencies

Learnwell
ACME Math–III
ENGLISH Practice Book GR–10 (Teacher Manual) 127

HF_Acme_ENG_TM_G10.indd 127 5/22/2021 5:58:53 PM


3. three
4. the ROI (Return on Investment) by presenting all your products and services to end
users with keywords.

Task–2
1. True 2. False 3. False 4. True 5. True
6. True 7. False 8. True

Task–3
1. Extra 2. Speaker 5 3. Speaker 4 4. Speaker 3 5. Speaker 2
6. Speaker 1 7. Extra

Task–4
1. (a) Hygiene 2. (c) both (a) and (b)
3. (c) infectious microbes 4. (a) Sanitation
5. (b) hygienic condition 6. (c) India

LISTENING TEST–2
Task–1
1. 1st July to 15th July from 9 am to 5 pm
2. Summer Reading Book Fair
3. the Bookworm Reading Book Fairs App
4. a selection of affordable books, including the newest titles for every reading level.

Task–2
1. False 2. False 3. True 4. True 5. False
6. True 7. True 8. True

Task–3
1. Speaker 1 2. Speaker 2 3. Extra 4. Speaker 4 5. Extra
6. Speaker 5 7. Speaker 3

Task – 4
1. (b) it gets evaporated 2. (c) underground water sources
3. (c) plants use it up 4. (b) seep down to rock
5. (a) human consumption 6. (c) located in a few areas of Rajasthan

LISTENING TEST–3
Task–1
1. Ooty

128 ACME English Practice Book GR–10 (Teacher Manual)

HF_Acme_ENG_TM_G10.indd 128 5/22/2021 5:58:53 PM


2. Scholastica Travel Inc., an educational travel company
3. positive memories 4. in seminar hall

Task–2
1. True 2. True 3. False 4. False 5. True
6. True 7. False 8. True

Task–3
1. Speaker 1 2. Speaker 3 3. Speaker 5 4. Speaker 4 5. Extra
6. Extra 7. Speaker 2

Task–4
1. (b) Marie Curie 2. (a) disappointed 3. (c) husband
4. (a) 1895 5. (b) professorship 6. (c) Chemistry

Test Papers

ANSWER KEY

TEST PAPER 1

Reading
(1)
1. (iii) Option (c) 2. (i) Options (a) and (b)
3. (c) The Importance of Friendship in Our Life
4. (d) Friends are more important in the modern era where most of us live away from
original families.
5. (a) Because, in absence of friendship, one lacks intimacy and tends to fall into
depression
6. (d) making investments 7. (c) Both (a) and (b) 8. (d) All of these
9. (a) two people do not judge each other and are happy about it
10. (d) Different 11. (d) Am I free to choose a dress of my choice?
12. (b) To be a good friend, be a good communicator.
(2)
1. (b) Delhi, a city that is the centre of government of a country
2. (c) The stubble burning in Punjab
3. (d) All of the above 4. (b) genocide
5. (a) Life expectancy of the inhabitants would be 3.2 years less than what it ought to
be.
6. (ii) (a) is acting irresponsibly and (b) is acting responsibly

Learnwell
ACME Math–III
ENGLISH Practice Book GR–10 (Teacher Manual) 129

HF_Acme_ENG_TM_G10.indd 129 5/22/2021 5:58:53 PM


7. (b) Because Punjab government has not taken action despite the warnings of the HC
8. (ii) Options (c) and (d) 9. (b) 1460 kg
10. (d) To segregate garbage and use it for power generation
11. (c) Delhi Government; Yes 12. (iii) Option (c)

Literature
1. (A)
1. (b) God 2. (iii) (a), (b), (d) 3. (d) All of the above
4. (ii) F – (a), (c) and O – (b), (d) 5. (b) Affixed
(B)
1. (c) Young seagull 2. (d) 3. (iii) (a), (b), (d)
4. (a) For diving and soaring 5. (iv) Option (d)
2. (A)
1. (b) Hatred 2. (a) ababa
3. (d) Ice represents greed. 4. (a) Ice is capable of destroying the world.
5. (i) Option (a)
(B)
1. (a) How to cope up with loss 2. (ii) Option (b) 3. (c) Repetition
4. (c) Money can’t buy back or replace our sense of pain, sorrow or loss
5. (d) All of these

Grammar
1. (a) (ii) advised me to take that medicine twice a day for five days
(b) (iii) asked me if I was sure that it had been the same medicine
(c) (i) requested him to prescribe me some other medicine
2. (a) (i) that live (b) (ii) are related (c) (iv) taking over
3. (a) (iii) used to (b) (i) are (c) (iii) Few
(d) (iii) A; The (e) (ii) had been watching (f) (i) May

Writing
1. (a) 23-J, Ashok Vihar
New Delhi
6th October 20xx

The Medical Superintendent


Sunrise Hospital
Market Road
New Delhi

Sub: Chaotic conditions in the casualty department

130 ACME English Practice Book GR–10 (Teacher Manual)

HF_Acme_ENG_TM_G10.indd 130 5/22/2021 5:58:53 PM


Sir
This is to bring to your attention that I had an extremely disappointing experience
in the reputed Sunrise Hospital yesterday. A person was badly injured in a hit-and-
run case. I had brought the victim to your hospital to be treated as an emergency
case. The victim needed urgent medical attention, but to my utter disappointment
the victim was attended after a prolonged period of painful and tedious wait. The
emergency department appeared to be in a chaos. There was no doctor. A long
queue of restless patients had already been formed before the doctor arrived. I
remained profoundly worried for the well-being of the patient who was bleeding
severely. Only after much pleading with the doctor I could compel him to see
the patient. Such a response is not acceptable on ethical grounds. I would like to
file a complaint and expect you to carry out a full investigation according to the
standards of your hospital. I will be available if you need further information.

Thanking you
Yours sincerely
Neha

(b) 29, Rajpur Road


Ambala
4th March 20xx

The Editor
The Tribune
Chandigarh

Subject: Spread of waterborne diseases

Through the column of your esteemed daily, I would like to draw the kind attention
of the concerned authorities and common masses towards the spread of diseases
during summers. As a responsible citizen, I’d like to create awareness about the
sudden spurt in the water-borne diseases among the public through this letter.
With the onset of summer, there is a sudden spurt of water-borne diseases like
cholera, dysentery, etc. It has become highly necessary to keep a check on such
diseases and their sources. Awareness campaigns need to be organised by the
government. A drive for pasting informative posters containing messages on how
to control spread of water borne diseases should be launched by local authorities.
We as citizens must also do our bit to control the spread at our end. I hope attention
would be given to my suggestions and some immediate steps must be taken in
this direction.

Thanking you
Yours sincerely
Vineet

Learnwell
ACME Math–III
ENGLISH Practice Book GR–10 (Teacher Manual) 131

HF_Acme_ENG_TM_G10.indd 131 5/22/2021 5:58:53 PM


2. (a) The given graph shows the sales and earnings of a big departmental store from
the year 2000 to 2007. It is evident from the study of the graph that there are lots
of ups and downs in the quantity of both sales and earnings over the period of
seven years. Earnings have been shown in million pounds whereas sales have been
shown in lakhs of pounds. The graph depicts that both sales and earnings have
gone up in the year 2001 in comparison to the year 2000. Earnings decreased
in the next two consecutive years by 4 and 3 million pounds respectively. The
earnings keep on increasing in the next two years. In 2006 the earnings decrease
by 1 lakh pounds and 2 lakh pounds in 2007. Sales reflect a rise of 4 lakhs of
pounds in the year 2001 and fall of 1 lakh of pounds for the next year. Earnings
show a sharp rise of 5 million pounds in the year 2005 and a slight decrease in
the next two years. A rise is observed in sales in the year 2007.The graph displays
profitable conditions of the business.
(b) The emergence of novel pathogens, bacterial or viral, has always posed serious
challenges to public health around the globe. Regular and proper handwashing can
check the spread of these viruses. As the fear of this deadly virus is increasing, there
has been an increase in the use and popularity of alcohol-based hand sanitizers in
healthcare sector as well as and social communities. It is progressively replacing
the traditional handwashing with soap and water. The use of alcohol-based hand
sanitizers has become a common practice in every household because it is being
considered as an effective alternative to handwashing to prevent the spread of
bacterial and viral infections. A range of hand sanitizers are available with various
combinations of ingredients and modes of delivery. Currently, strategies to deal
with COVID-19 are purely supportive and preventative, and are aimed at reducing
the transmission of this deadly virus. Hand hygiene is an effective and basic
method of controlling transmission of infections. The increasing popularity of hand
sanitizer shows that most alcohol-based hand sanitizers are effective at inactivating
viruses, including coronaviruses. So, as far as hand washing is concerned, instead
of soap and water, sanitizer should be used to ensure complete and appropriate
hand hygiene.

Literature
1. (A) (a) Maddie lacked the courage to look Peggy in the face and tell her what she’s doing
was not right. After writing the note Maddie tore it rather than giving it to Peggy
because she thought that Peggy would never value her thoughts and instead
would get annoyed by that note. She might also direct her bullying at her.
(b) When the pirate came, Belinda cried for help and became pale with fear. Other
pets ran and hid themselves. Later Mustard said that he would have been brave
if he hadn’t been flustered. Ink and Blink said they would have been three times
as brave as Custard.
(c) Nelson Mandela associated himself with those African patriots who had gone
before him by saying that he was simply the sum of all those African patriots who
had sacrificed their lives for the sake of freedom. He said that he was grateful to
those who had gone before him because those heroes of the past had paved the
path of courage and sacrifice for him.
(B) (a) Bill told the narrator that probably Oliver Lutkins was trying to start a poker game
in the back of Fritz’s shop. Bill led him there and he asked the narrator to hide
behind him.

132 ACME English Practice Book GR–10 (Teacher Manual)

HF_Acme_ENG_TM_G10.indd 132 5/22/2021 5:58:53 PM


(b) Mr Herriot is a veterinary doctor. He is very kind and gentle with animals. He
warns the dog’s mistress of the consequences of over-feeding the dog but she
loves the dog so much that she doesn’t change her ways. Mr Herriot’s kindness
and compassion compels him to bring the dog to the surgery. He treats the dog
and returns to its mistress healthy and fit.
(c) Bholi’s father is worried about her as she has neither good looks nor intelligence.
When she was ten months old, she fell off the cot. This led to damage to some
part of her brain due to which she could not speak till five years of her age. She
also has pock-marks on her face due to small-pox. Her father worries that no one
will marry her.
2. (A) (a) Anne’s diary is different in a way that she considers it a real person and in fact a
friend in whom she can confide everything. It was not just a journal for her to jot
down facts like other people do. Kitty was the name she gave to her diary.
(b) According to the poet, the world will end due to the ‘fire’, which symbolises
boundless human desires. Frost considers human desires to be powerful enough to
lead the world to a quick end. Desire leads to greed and greed leads to dishonesty,
violence and destruction to fulfil the desire.
(c) Amanda remains moody most of the time because she lives in an imaginative
world and tries to make an escape from her reality where she is constantly nagged
by her mother for everything she does. She does not like to be instructed and
controlled by her mother. So, she chooses to ignore her.
(B) (a) Griffin didn’t care if he harmed anybody while carrying out his malicious wishes.
He set his landlord’s house on fire because the landlord tried to make him leave.
So, we can say he was a lawless person. Committing crimes did not bother him
at all.
(b) Reading books is just an aspect of learning and gaining knowledge about a
particular subject. It helps us to improve subject knowledge but mere reading of
books will not make a person veteran in any subject. A person should possess other
qualities of observation, thoughtfulness and carrying out experimental activities
to reach his/her goal.
(c) Matilda’s husband is a genuine, simple and supportive man. He immediately gives
his savings to his wife when she asks for a dress for the ball. This shows that he is
very caring and concerned about his wife. He is a man of dignity too. He works
very hard to pay for the lost necklace.
3. (a) I agree with the statement that depths of oppression create heights of character.
Nelson Mandela had given the examples of great heroes of South Africa in his
speech to justify this. He reminded people of the sacrifices of his people who
sacrificed their lives during the long freedom struggle. The independence achieved
through their sacrifices signified the triumph of tolerance over discrimination,
justice over oppression and humanity over barbarity. Mandela called himself
as the sum of all those African patriots who had gone through several years
of oppression and torture in the bygone years. He paid his tributes to all those
who had sacrificed their lives for the sake of gaining freedom for all because
those heroes of past paved the path of co-operation and unity for other South
Africans. He also believed that during freedom struggle there was a galaxy of
leaders with great character. Probably the oppression of British rule created so
many men with such great character. Nelson Mandela was profoundly grateful to
these people.

Learnwell
ACME Math–III
ENGLISH Practice Book GR–10 (Teacher Manual) 133

HF_Acme_ENG_TM_G10.indd 133 5/22/2021 5:58:53 PM


(b) Kisa Gotami’s only son had died. She was so swayed in her grief that she carried
the dead child to all her neighbours, asking them for medicine to cure her son.
Finding her in so much misery, someone suggested her to go to Sakyamuni, the
Buddha. Kisa Gotami made the same request to Gautama Buddha, i.e. to give
her the medicine that would cure her son.
The Buddha replied that he could cure her son if she brought him a handful of
mustard-seed from a house where no one had lost a child, relative, husband,
parent or any dear one. Kisa Gotami went from house to house. People pitied her
but she could not find any house where a near and dear one had not died. She
realised that she had been selfish in her grief. She realised that death is common
to all human beings. It is the inevitable truth of life.
4. (a) Hari Singh was a thief and he stole Anil’s money. After the theft, he realised that
he had robbed not only Anil but also himself of the chance of being literate and
having a bright future. His conscience pricked him. He was haunted with the
thought that he could have gained a lot more than just money had he not robbed
Anil. He realised that he could no longer learn how to read and write. His inner
self did not let him overcome this thought and compelled him to return.
Hari’s act of returning to Anil shows that in spite of indulging in criminal acts, he
still had a practical and optimistic attitude towards life and people. The moment
he decided to return to Anil and put back his money, he turned from a lawless
person to a good human. Anil’s love and care was responsible for bringing this
reformation in Hari’s mindset and character. It gives us the message that love and
care can change a bad person into a good one. Anil’s understanding and helpful
nature made Hari mend his ways.
(b) Mme Loisel belonged to a family of clerks. They lived on meagre income which
was just enough for fulfulling basic needs but it was not good enough to lead a
luxurious life. After getting married to a clerk, Matilda remained in an imaginative
world of her dreams of wealth and pleasure. She was so caught up in her fantasies
that she was not able to realise the truths of her real life. She spent huge amount
of her husband’s income to buy a dress just to flatter her pride. She didn’t even
hesitate to borrow a necklace from her friend to flaunt her fake status at the party.
She did all this just to impress the wealthy and rich with her beauty and glamour.
She temporarily got what she wanted. She was praised by all. Unfortunately, she
had to pay a huge price for fulfilling her fancies. The borrowed necklace was
lost and the couple had to sacrifice their entire savings and had to take a loan to
replace it unnoticed. The next ten years of their youth and happiness were ruined
in the repayment of the debt. They had to live in extreme poverty. Hence, her
ambition invited her doom.

TEST PAPER 2

Reading
(1) 1. (c) eat foods in sodium 2. (iv) (c), (e) 3. (i) Option (a)
4. (iii) (a), (d) 5. (b) Food Problems in Food Deserts
6. (a) The members of the club had easy access to its premium benefits.
7. (c) food retailer 8. (b) Poorest areas
9. (ii) F – (a), (b) and O – (c), (d)

134 ACME English Practice Book GR–10 (Teacher Manual)

HF_Acme_ENG_TM_G10.indd 134 5/22/2021 5:58:53 PM


10. (a) Because they can’t earn enough from lower income regions
11. (ii) (a), (b) 12. (ii) Option (b)
(2) 1. (b) As a very special human being
2. (b) She couldn’t retain what her teachers taught.
3. (iii) (c), (d) 4. (c) medicine 5. (iv) (c), (e)
6. (c) Dr Bacelli 7. (d) Ignorance 8. (b) first Italian
9. (iii) (a), (c), (e) 10. (c) Housing in a mental asylum
11. (c) technical school 12. (a) better

Literature
1. (A) 1. (c) Lomov 2. (ii) (a), (c), (d)
3. (ii) F – (b), (c) and O – (a), (d)
4. (d) Boldness 5. (i) Option (a)
(B) 1. (a) Matilda 2. (c) She was a pretty young lady.
3. (d) All of these 4. (iii) Option (c) 5. (c) Unrealistic
2. (A) 1. (d) Dust of snow 2. (iii) (b), (d)
3. (b) He could not enjoy the day to the fullest.
4. (c) Alliteration 5. (b) abab
(B) 1. (b) World 2. (iv) F—(b), (c) and O—(a),(d)
3. (d) All of these 4. (d) Enough 5. (a) Enjambment

Grammar
1. (a) (i) was covered (b) (iii) to breath (c) (iv) for
2. (a) (iii) what he wanted to do that day
(b) (i) the weather was so pleasant outside
(c) (ii) to watch his steps so he didn’t trip
3. (a) (i) does (b) (i) visit (c) (i) will
(d) (ii) are (e) (iv) has (f) (iii) The

Writing
1. (a) House No. 123
XYZ City
8th January 20XX

The Editor
The Times of India
Delhi

Sub: Expressing concern for environmental pollution

Sir
Through the columns of your esteemed newspaper, I would like to express my
concern and paint the present situation to caution people against environmental
pollution.

Learnwell
ACME Math–III
ENGLISH Practice Book GR–10 (Teacher Manual) 135

HF_Acme_ENG_TM_G10.indd 135 5/22/2021 5:58:53 PM


Pollution has been a serious threat since a very long period of time. Emissions
from factories, smoke emitted by chimneys and vehicles are causing severe air
pollution. Industrial waste, plastics, sewage, etc., are polluting our water bodies.
Excessive use of pesticides and deforestation are leading to soil pollution. The
needless car honking, the use of loudspeakers add to noise pollution.
Every year Delhi-ites find themselves struggling to breathe with the onset of winters
due to excessive pollution. The frequency of floods, cloud bursts and landslides
have increased in Uttarakhand. A large part of the country is experiencing either
drought or flood at regular intervals.
But, we just complain and do nothing. It is well said by Walt Disney that the way
to get started is to quit talking and begin doing. The government as well as the
public needs to take drastic and immediate steps to fight this menace. Otherwise
we won’t be able to leave a livable earth for our future generations.

Thanking you
Yours truly
Arun

(b) Sector 16
Nirman Vihar
New Delhi
7th January 20xx

The Manager
Vikas Sales
Preet Vihar
New Delhi

Sub: Raising complaint against faulty appliance

Dear sir
This is with reference to the order no 1234 placed on 1st January 20XX. I bought
a refrigerator from your store on New Year.
It worked fine for a few days after the purchase but I feel disappointed to say that
it has developed certain problems in its functioning. Quite frequently, it shuts
down on its own. Cooling has stopped entirely. In addition to that, it also makes
a lot of noise while working. So, I request you for an immediate replacement for
the same since the warranty hasn’t expired. I am attaching a copy of the bill with
the letter. I hope you take immediate action to remedy this grievance.

Thanking you
Yours truly
Shivam

2. (a) The bar graph compares the population growth in five large countries in the years
1980, 1990 and 2000. The line graph analyses the population of five countries,

136 ACME English Practice Book GR–10 (Teacher Manual)

HF_Acme_ENG_TM_G10.indd 136 5/22/2021 5:58:53 PM


namely France, United Kingdom, Mexico, Nigeria and Pakistan in millions. In
1980, Mexico had the highest population among all the countries however by
1990, Nigeria emerged as the country with the highest population. The population
growth in France and United Kingdom had been lesser than other countries. As
the data suggests, Pakistan has seen a consistent rise in population in the next
two decades with approximately 75 million residents in 2000. There is a sharp
increase in Mexico’s population in the decade between 1980 and 1990 but very
slight growth in the decade between 1990 and 2000. Finally, it can be concluded
from the graph that the population was on rise in all the five countries by the end
of the year 2000.
(b) Today, we are faced with the serious problems of environmental pollution,
the depletion of ozone layer, problem of waste disposal, the disappearance
of wilderness areas, a steady loss of biodiversity and the extinction of species.
These problems call for definite and legally binding solutions. The environmental
problems are caused due to the massive increase in human population and the
associated urbanisation and industrialisation. People need to be made responsible
towards the environment. They should be issued guidelines about proper waste
disposal, recycling and reuse of goods, using public transport, planting trees and
saplings, using water judiciously, using renewable sources of energy, rainwater
harvesting, etc. Those who are seen flouting the laws should be punished.
Government needs to take strict measures to check pollution and reduce carbon
footprint. Only stringent laws and strict implementation can save the earth now.

Literature
1. (A) (a) Anne’s statement, that no one could understand her intensity of love for her
grandma tells that she loved her grandmother very dearly. Moreover, the gesture
of lighting up one candle for grandmother during Anne’s birthday is also a symbol
of her love for her grandma.
(b) Lomov and Natalya were fighting over the issue of the ownership of the Oxen
Meadows. Both of them claimed to have the ownership of the meadows. When
Chubukov intervenes, Lomov and Chubukov start quarrelling over the Oxen
Meadows.
(c) If Amanda were a mermaid, then she would drift slowly in a languid emerald sea.
She would live alone in the green sea. She would find peace and move without
restrictions.
(B) (a) Hari lied about knowing to cook. Before Anil appointed Hari Singh as a servant,
he told him that he would not give Hari a regular salary. When Hari cooked on
the first night, Anil realised that he didn’t know how to cook. So, Anil offered to
teach him how to cook.
(b) Griffin can be called a brilliant scientist as he made a unique discovery of how to
become invisible. It was a great scientific invention. But he didn’t use his scientific
discovery for social cause. He misused his talent. He set fire to a house, beat
people up and stole money and things. He was a lawless person.
(c) The narrator, Mr Herriot is a good veterinary surgeon. He is an efficient and wise
doctor. His handling of Tricki properly shows that he is sincere and dedicated to
his profession. He treats Tricki by adopting simple methods. He doesn’t give him
any medicines but cures him by altering his diet and activities.

Learnwell
ACME Math–III
ENGLISH Practice Book GR–10 (Teacher Manual) 137

HF_Acme_ENG_TM_G10.indd 137 5/22/2021 5:58:53 PM


2. (A) (a) By using the words ‘an extraordinary human disaster’ Mandela is referring to the
systematic discrimination against the Blacks and the practice of apartheid in South
Africa. There was racial segregation based on colour and the blacks were oppressed
and had to suffer a lot. They had very limited right and mobility. Mandela himself
spent thirty years in prison. A black person becoming the president of South Africa
has been called “glorious human achievement” by Mandela. This was a glorious
achievement as it ended the policy of apartheid and established a society where
everyone was free and equal.
(b) The young seagull was very hungry. It was his hunger that ultimately compelled
and encouraged him to fly. His mother took a piece of fish and flew near the
edge of the cliff. The young seagull, who hadn’t eaten for a day, ran towards his
mother. His mother moved away from the edge and he fell downwards. Soon he
learnt to flutter his wings and started flying.
(c) The tiger can only walk a few paces in the tiny cell he is locked in. He feels like
a prisoner. He longs for freedom. He wants to be in the forest where he lives his
own life by lurking in the shadows and stalking his prey near the water hole, where
plump deer come to drink water.
(B) (a) After James Herriot’s encounter with Mrs Pumphrey and Tricki, he was expecting
a call from her soon for help. He anticipated that the overfeeding and lack of
physical activity would soon put Tricki’s health in danger. And just as anticipated,
Mrs. Pumphrey called the vet a few days afterwards because Tricki had become
very ill.
(b) Bholi’s parents were fearful that if they rejected Bishamber’s proposal, Bholi
might not be married all her life. Her mother thought they were fortunate that
Bishamber was from another village and did not know about Bholi’s pock-marks
and her stammering. Moreover, they didn’t have to pay him dowry. Hence, they
accepted the marriage proposal.
(c) The lawyer was sent to New Mullion to serve summons on Oliver Lutkins. Lutkins
was required as a witness in a law case. Bill wanted to avoid the summons and
go as a witness. So, when he met the lawyer and got to know his purpose of
visit he pretended to be another person, a hack driver. He offered his help to the
lawyer in searching for Lutkins so that the lawyer would not know about him from
someone else.
3. (a) In the beginning of his life, Mandela was not aware about the true meaning of
freedom. As Mandela grew, he started realising that his freedom had been taken
away from him. When he was a student, being immature, he desired to win
freedom only for himself but gradually he started considering the freedom of his
people as important as freedom for himself. This changed him completely. Freedom
had varied meaning for Mandela at various stages of his life and experience. As
a boy, freedom meant for him to be able to run freely in the fields and swim in
the stream. When he was a student, he wanted freedom to stay out at nights, to
read what he liked, later he realised that this was real freedom. He found that all
the black natives of South Africa were discriminated against and had no freedom.
They had no freedom to live a respectful life. They were discriminated against
socially and politically. He wanted the basic and respectable freedom to live a
normal social life. He believed that everyone has to be free.
(b) The conflict between humans and nature is illustrated by the destruction of Lencho’s
crops by the hailstorm. Lencho was very hopeful for a good harvest as he had

138 ACME English Practice Book GR–10 (Teacher Manual)

HF_Acme_ENG_TM_G10.indd 138 5/22/2021 5:58:53 PM


worked really hard on his fields. As his entire harvest got destroyed by heavy
storm, he had nothing left to feed his family. Lencho was devastated and felt
desperate. This appropriately projects the conflict between nature and man. One
more conflict is reflected by the story, the conflict between humans themselves.
The postmaster, with the help of his colleagues, tried to provide financial assistance
to Lencho and he managed to collect some money and sent it to Lencho. Their
help was not driven by any selfish motive. In spite of having no relationship with
Lencho, they helped Lencho out of kindness and selflessness. Even though they
did a good deed, Lencho blamed them for taking away some amount of money.
This proves that man does not trust fellow humans, thereby giving rise to this
conflict.
4. (a) The lady in the red dress manipulated the whole situation and very skillfully got
the jewels from Horace without even touching anything in the house. She boasted
of being morally superior. She said that the society should be protected from men
like Horace. Horace promised her that he would follow the path of honesty. He
was taken in by her appearance and helped her by getting the jewels out of safe
by breaking it. He did it without wearing gloves. After two subsequent days, he
was caught by the police for stealing jewels. He could not convince them that he
had not taken the jewels and had helped the owner’s wife as the actual owner’s
wife turned out to be someone else. So, we can say the real culprit was the lady.
She tricked Horace Danby and promised him that she will not tell the police.
Horace’s honesty in the wrong act proved to be a great trouble for him.
(b) Anil would have come to know about the theft because the notes had got damp
due to rain. He was a kind and wise man. It wouldn’t have been difficult for Anil
to discern the series of events that would have taken place the previous night.
But he does not hand over the thief to the police because he realised that Hari
had learnt his lesson and he had changed. Otherwise, he would never have come
back and kept the money at the same place from where he had stolen it. Anil
wanted to bring about a change in Hari by giving him a chance to get educated
and become worthy. So, he didn’t hand him over to the police. I think most people
would have handed him to the police thinking of him as a common thief who
can’t be trusted.

TEST PAPER 3

Reading
(1) 1. (a) intestinal tract 2. (c) Digestive problem
3. (d) Constricted breathing 4. (iii) Option (c)
5. (b) unexpected increase 6. (i) F – (a), (b) and O – (c), (d)
7. (a) His anger was triggered by his brother’s actions.
8. (c) In the skin and pulp of fruits
9. (iv) Options (b), (d) and (f) 10. (a) Figs
11. (b) digestive system 12. (d) 25 g – 35 g
(2) 1. (d) all age groups 2. (iii) Options (b) and (c) 3. (a) Thailand
4. (c) Millennium Development Goals
5. (ii) F – (a), (c) and O – (b), (d) 6. (b) 2015

Learnwell
ACME Math–III
ENGLISH Practice Book GR–10 (Teacher Manual) 139

HF_Acme_ENG_TM_G10.indd 139 5/22/2021 5:58:53 PM


7. (b) 2005, 2015 8. (c) Promote tourism in low income areas
9. (i) Option (a) 10. (d) Trivial
11. (a) implement the EFA movement
12. (b) Millennium Development Goals

Literature
1. (A) 1. (iii) F – (d) and O – (a), (b), (c)
2. (b) Nelson Mandela: Long Walk to Freedom
3. (a) Black people 4. (i) Options (a) and (b)
5. (a) The black population was discriminated against.
(B) 1. (b) Life was temporary. 2. (b) A young cow
3. (iii) Options (a) and (d) 4. (b) It was transformed into a lifeless one.
5. (a) Comforting
2. (A) 1. (a) To cope up with loss 2. (d) Repetition
3. (c) Money can’t buy back or replace our sense of pain, sorrow or loss.
4. (b) People can snatch you materialistic possessions.
5. (c) Possession of materialistic things
(B) 1. (d) repetition 2. (b) Tiger 3. (d) patrolling cars
4. (b) stars 5. (a) Keeping a watch over something

Grammar
1. (a) (ii) that promise (b) (iii) of doing (c) (iv) have
2. (a) (iv) had taken him to (b) (iii) remarked
(c) (i) were just them
3. (a) (iii) had left (b) (iii) may (c) (iv) Could
(d) (ii) should (e) (iv) has (f) (iii) the

Writing
1. (a) Abc School
Vasant Vihar
New Delhi

8th January 20xx

The Manager
Ecotourism Society of India
E-75, Basement
East of Kailash
New Delhi

Sub:- To place an order for the goods

140 ACME English Practice Book GR–10 (Teacher Manual)

HF_Acme_ENG_TM_G10.indd 140 5/22/2021 5:58:53 PM


Dear sir
I am writing to you in reference to the advertisement published in the Hindustan
Times on 1st January 20xx. Our institution has decided to place an order of the
items mentioned below:
Sr. No. Item Name Quantity
1 Black T- shirts 500
2 Badges 500
3 Caps 500
4 Wrist bands 500
Please find with this letter the attached logo to be printed on all the above listed
items. You have a good reputation of selling quality products, so we expect high
quality products to be delivered to us. We want to establish a long term relationship
with your firm. We expect a good discount to be given on the listed items. We
expect the items to be delivered by 25th January 20xx. Please attach a copy of
bill and ensure proper packaging. The transportation cost will be borne by you.
Payment will be done through cheque on receiving the order.

Yours truly
Ratan

(b) House No.215, Sector-5


XYZ Colony
Bengaluru

6th February 20xx

The Editor
Hindustan Times
Bengaluru

Sub: Nuisance caused by traffic jams

Sir
Through the columns of your esteemed newspaper, I would like to draw the
attention of concerned authorities towards the nuisance caused by traffic jams.
Traffic jams are the bane of city life. Caused by accidents, political rallies, strikes,
construction work, VIP movement and encroachment, these jams are a menace.
Time is the most essential thing. These jams lead to the wastage of time to a great
extent. Traffic jams not only affect the environment but also the health of human
beings. It has been proved that increased air pollution generated by vehicle jams
causes blood pressure to rise and arteries to inflame, increasing chances of heart
attack and stroke for people who reside near traffic-prone areas.
To get rid of the problem, I would suggest that more and more traffic instructors
should be appointed at traffic prone areas. Traffic rules should be made stringent

Learnwell
ACME Math–III
ENGLISH Practice Book GR–10 (Teacher Manual) 141

HF_Acme_ENG_TM_G10.indd 141 5/22/2021 5:58:53 PM


and followed strictly. Public should be encouraged to travel by public transport.
I hope my views find space in your esteemed newspaper.

Thanking you
Yours truly
Divya

2. (a) The pie chart shows the percentage of money spent on various items and savings
by a family during the year 2001. The family spent 23% of its income on food,
15% on household activities, 10% on clothing, 15% on children’s education, 5%
on transport, and 20% on miscellaneous things. After these expenditures, the
family managed to save 15 per cent of their total income.
The family spent their money wisely. The largest percentage of their income was
spent on food, i.e. 23%. Education also got sufficient attention of the family
with 15% of spending. The same amount was also contributed towards housing.
Spending 5% on transport and 10% on clothing are also reasonable. With all
these expenditures the family is able to save about 15 percent of the total income,
which is pretty good.
(b) Women are the creators of life. Their empowerment needs to be ensured so that
they can contribute efficiently in the economic, social, political, and environmental
growth of the society. Women and girls everywhere must have equal rights and
opportunity, and should be able to live freely. Violence against women and gender-
based discrimination must be dealt with strictly. Urgent steps must be taken to
eliminate the many root causes of such discrimination that still curtails women’s
freedom in private and public spheres. Gender equality is crucial for the future of
the society. It is proven that empowering women helps achieve higher economic
growth and development. Women are better able to understand emotions and
body language making them rational thinkers and team coordinators. Women are
better at negotiating as they combine their wider knowledge of societal experiences
with their negotiation skills and hence have a better chance of cracking a deal.
A society cannot progress efficiently if women are not provided a safe space and
opportunities to grow as well as men.

Literature
1. (A) (a) Wanda Petronski was a poor Polish migrant living in America. She lived in
poverty ridden area called Boggins Heights where many poor people lived. She
was different from the other children. She neither had many friends, nor many
dresses. She was very introvert and usually remained quiet.
(b) The poet sees the ball escaping from the boy’s hand, bouncing down the street
and falling in to the water. The poet doesn’t want the boy to get temporary refuge
from his grief by offering him a substitute for his lost ball. He wishes him to be
emotionally strong by facing and accepting such situations.
(c) The box in which Mijbil was to be transported was lined with a metal sheet. Mij
was not at all comfortable in the box and tried his best to escape. While trying to
escape, Mij tore into the metal lining of the box and badly hurt himself.
(B) (a) Hari Singh planned to run away to some distant place with the money stolen
from Anil by catching the Lucknow Express. He thought he would live like Arab
for some days.

142 ACME English Practice Book GR–10 (Teacher Manual)

HF_Acme_ENG_TM_G10.indd 142 5/22/2021 5:58:54 PM


(b) Matilda did not want to go to the ball because she had no good clothes, and no
jewels to wear at the party. She thought that she would not look good in her old
fashioned dull dresses.
(c) Ramlal was a prosperous farmer. He was worried about Bholi because she
had pock marks on face. She was also mentally unfit due to an accident in her
childhood. She stammered while she spoke. Ramlal worried that nobody would
agree to marry such a girl.
2. (A) (a) The people of Coorg are of Greek or Arabic origin. According to the Greek theory,
a part of Alexander’s army married and settled with the locals of Coorg after their
return seemed impossible. The traditions here are noticeable amalgamation of
Greek and Hindu traditions. The Arab origin can be observed in the long, black
coat worn by the Kodavus, which resembles the kuffia worn by the Arabs and the
Kurds.
(b) The tiger expresses his anger quietly because he has been put behind the bars of
the cage. He moves very softly and quietly with his soft paws in the limited range
of the cage.
In a concrete cell of a zoo, his powerful and strong body is useless as he is behind
the bars. His movements have been limited up to the length and breadth of his
cage. He simply ignores the stares of visitors.
(c) The leaves strain to move towards the glass while the small twigs become stiff
with exertion. They struggle hard for freedom. They struggle with all their might
attempts to make themselves free and move towards the forest.
(B) (a) Pumphrey thought that Tricki was suffering from malnutrition as he was weak
and listless and he was refusing food. So, she started to give him extra things like
cod-liver oil and malt between the main meals and Horlicks after dinner to make
him stronger.
(b) Griffin entered the big London store and broke the boxes and opened the wrappers.
He took out warm clothes and wore them on his body. He drank coffee, wine
and ate cold meat from the shop. After eating to his content, he lay on the pile
of quilts and fell asleep. In the morning when workers came to the store, he had
to take off his clothes and become invisible to get away from being caught.
(c) Ebright conducted his research and experiment to discover a hormone that is
essential for the growth of a butterfly. His experiments showed the cells would
divide and develop into normal butterfly wing scales only if they were fed the
hormone from the gold spot. Finally, he was able to identify the hormone’s
chemical structure. A year-and-a half later, Ebright got the idea for his new theory
about cell life while observing the X-ray photos of this chemical structure. The
photos gave him the answer to one of biology’s puzzles: how the cell can ‘read’
the blueprint of its DNA, the blueprint of life.
3. (a) Mr Keesing punished Anne for her excessive talking in class and asked her to write
an essay, she thought that he was playing a joke on her. She wrote three pages
about talking describing it as a trait of a student.
Mr Keesing was amused but he gave her another topic ‘An Incorrigible Chatterbox’
to write an essay on. She completed the essay to the satisfaction of her teacher.
Mr Keesing didn’t complain for two classes but by the third class he had had
enough. As a punishment he asked to write an essay on ‘Quack, Quack, Quack,
Said Mistress Chatterbox’.

Learnwell
ACME Math–III
ENGLISH Practice Book GR–10 (Teacher Manual) 143

HF_Acme_ENG_TM_G10.indd 143 5/22/2021 5:58:54 PM


She wrote a satirical poem on the topic. It was about a mother duck and a father
swan with three baby ducklings. The three ducklings were bitten to death by the
father because they quacked too much. Mr Keesing was amused by the poem. He
read the poem to the class adding his own comments and also to other classes.
Hence he was very predictable. But in her later assignment by Mr Keesing, she
composed a poem on ‘The incorrigible Chatterbox’ and gave a message through
it to the teacher. The same teacher who wanted to punish Anne, got so impressed
by her little poem that he decided not to punish her. He took the poem in the
right way and understood the joke. After that he decided never to assign any extra
homework to Anne for talking in the class. On the contrary, he started making
jokes in the class. This instance shows teacher’s unpredictable behaviour.
(b) Valli collected all the necessary details of the bus journey by listening carefully to
the conversations between her neighbours and regular bus users. It came to her
knowledge that the town was just six miles from her village and the one way fare
was thirty paise. The trip took forty-five minutes. Valli saved money and carefully
planned her visit to the town. While on the bus ride, Valli saw a dead cow on the
road. So, she learnt that life was short and unpredictable. It was a great learning
experience for her as she got exposure to the world outside her home and got a
chance to view panoramic sight of the landscape. She learned to see the world
beyond her house and this proved to be a great learning experience for Valli.
4. (a) Horace Danby was passionate about collecting rare and expensive books. To
satisfy this expensive passion, he arranged money by robbing one safe every
year and then secretly buying the books through an agent. He was sure that his
plan of robbery at Shotover Grange would be a successfully executed because
he had studied the house, the drawing room where the safe was kept, the wiring
and its garden very minutely. He had also studied the movement of the servants.
He had made sure that nothing could go wrong with extensive planning. He was
so taken in by the pretention of the young lady who claimed to be the mistress
of the house that he opened the safe for her. He committed a blunder by not
wearing gloves while he was opening the safe. He got arrested soon afterwards.
His mania for owning expensive books thus led to his downfall.
(b) The misinterpretation of the nursery rhyme by Think Tank frightened him and
led to the cancellation of Earth’s invasion by the Martians. He took the rhyme’s
figurative and imaginative words literally. Think Tank declared that Earthlings had
discovered how to combine agriculture with mining. He also told his team that
they were actually able to grow crops of rare metals such as silver. He interpreted
that ‘cockleshells’ were explosives and Earthlings could also grow explosives. He
thought that Humpty-Dumpty was referred to him. He thought that Earthlings had
seen him, and they were coming to attack him. He misinterpreted “Had a great
fall!” and thought it meant that Earthlings were planning to capture Mars Central
Control and him. He got frightened and ordered the invasion fleet to evacuate the
entire planet of Mars and go to Alpha Centauri, a hundred millions miles away
from Earth.

TEST PAPER 4

Reading
(1) 1. (iii) Option (c) 2. (iii) Options (c) and (e)
3. (b) Evolution of Computers in the Human World

144 ACME English Practice Book GR–10 (Teacher Manual)

HF_Acme_ENG_TM_G10.indd 144 5/22/2021 5:58:54 PM


4. (d) All of the above 5. (d) All of the above
6. (d) It will enable human beings to enjoy more luxurious and comfortable life than
they do today.
7. (i) Options (a) and (b) 8. (c) Sir Leon Bagrit
9. (a) machines will not be able to rule over human beings
10. (a) the whole process by which machines can be used to work for us
11. (c) You need to have practical as well as theoretical knowledge if you want to pursue
your career in any of the branches of Science.
12. (d) Computers will continue to make human life easy, but they will never be able to
function without human support.
(2) 1. (b) A member of the University of Hertfordshire’s School of Computer Science
2. (b) socially and emotionally connected
3. (b) European
4. (b) Robots that develop and display emotions as they interact with human
5. (d) All of the above
6. (a) To truly integrate robots in humans’ everyday lives as companions or carriers
7. (a) if they live and grow interacting with humans
8. (iii) Options (d) and (e) 9. (c) humans’ faces and voices
10. (b) different light and sound patterns
11. (a) baby 12. (i) Option (a)

Literature
1. (A) 1. (a) Anne Frank 2. (c) No one would be interested in them.
3. (c) Writing a diary 4. (iii) F – (a) and O – (b), (c), (d)
5. (iv) Option (d)
(B) 1. (d) Troubled, brief and combined with pain
2. (i) Option (a) 3. (a) dying
4. (i) F – (a), (b) and O – (c), (d)
5. (i) Option (a)
2. (A) 1. (c) Amanda; Robin Klein
2. (d) Because Rapunzel had very long hair that she would let down her tower for the
prince charming to climb up
3. (iii) F – (a) and O – (b), (c), (d)
4. (c) She wants to be free like her.
5. (b) aaa
(B) 1. (d) Dust of snow 2. (b) Gloomy to happy
3. (b) He overcame his sadness and became happy.
4. (c) Alliteration 5. (iv) Option (d)

Grammar
1. (a) (ii) should (b) (ii) will (c) (iv) ought to
2. (a)(i) if she was not going to the Maths class

Learnwell
ACME Math–III
ENGLISH Practice Book GR–10 (Teacher Manual) 145

HF_Acme_ENG_TM_G10.indd 145 5/22/2021 5:58:54 PM


(b) (iii) if she had informed her teacher
(c) (i) asked her to call on that
3. (a) (i) will attend (b) (i) had parked (c) (iii) little
(d) (ii) The; the (e) (ii) was (f) (ii) must

Writing
1. (a) House No. 243
Preet Vihar
Delhi

8th March 20xx

Chief Administrator
Pollution Control Board
Delhi

Sub: Severe problems caused by an industry

Sir
I am a resident of Preet Vihar, East Delhi. Through this letter, I would like to draw
your attention towards the hazardous problems caused by an industry near our
colony.
The waste generated by the industry is dumped directly into the lake beside it
without being treated. The dumped waste not only pollutes the lake but also gives
off a foul smell that permeates through the entire area. Apart from the foul smell,
the industrial plant also creates a lot of noise. This disturbs the peace of the people
living in the area, especially sick, senior citizens, babies and students.
The unrestricted dumping of the waste material is a health hazard for the people
living in this area.
Kindly take immediate action to rectify the situation to restore peace and health
of the people residing in the area.

Thanking you
Yours truly
Xyz

(b) 386-E, L Pocket


Gopal Park
Bengaluru

11th February 20xx

The Editor
The Deccan Herald
Bengaluru

146 ACME English Practice Book GR–10 (Teacher Manual)

HF_Acme_ENG_TM_G10.indd 146 5/22/2021 5:58:54 PM


Sub: Voicing concern for increasing commercialisation of education

Sir
Through the columns of your esteemed newspaper, I would like to draw the
attention of common folk and the concerned authorities towards the issue of rapid
commercialisation of education.
Education these days has become a business venture rather than a means to
contribute to the society. Quality education can now be afforded by only those
with money and privilege. Schools regularly and randomly increase their fees
leading to added pressure on common people. Added to this is the cost of
additional educational expenditures like books, stationery articles, uniform and
transportation. The increasing prices of all these items are breaking the backbone
of the parents of school-going children. Schools also charge for lab equipment,
educational tours, technology charges, etc. It is high time to realise the gravity of
this problem.
Government should take swift and drastic measures to stop the blatant
commercialization. It should form a body to regulate school fee and check
indiscriminate rise in the cost of education. The students from lower income
groups should be given additional financing support so that they can get good
quality education. Government should also improve the condition of government
schools so that people are not forced to send their children to expensive public
schools to get good education.
I hope my views find space in your esteemed daily.

Thanking you
Yours truly
Abc

2. (a) The table tells about the population characteristics of six states in a particular year.
Population, literacy and level of education shown in the table are basic indicators
of the level of development achieved by these states in a particular year. According
to the table, state D with literacy rate of 82% has the highest literacy among all
six states. As evident from the table, all the states except D and F have sex ratio
skewed against women. The gap in sex ratio is more in the states A and C. State A
has lowest literacy rate while state D has lowest population. The state with highest
population is E with 54 million people. The table clearly shows a relationship
between literacy rate and sex ratio. States with high literacy rate have better sex
ratio. In conclusion, state D can be called the best state with least population,
highest literacy rate and good sex ratio.
(b) Air pollution has many forms and is caused by several factors. The government
has banned the use of firecrackers to control pollution. But what we need is
awareness and regulation here. Diwali is celebrated once in a year but pollution
is an all year phenomenon. The problem is not that if people burst crackers,
the air will become polluted but it is that the air is already so polluted that we
cannot add more pollutants by bursting crackers. Pollution caused by the burning
of firecrackers during Diwali is just the tip of the iceberg. A number of factors
contribute to hazardous air quality especially during winter months in Delhi.
Adopting measures like regulating the firecrackers manufacturing industry, raising

Learnwell
ACME Math–III
ENGLISH Practice Book GR–10 (Teacher Manual) 147

HF_Acme_ENG_TM_G10.indd 147 5/22/2021 5:58:54 PM


awareness among retailers to not buy or sell crackers that cause more pollution
than allowed, raising awareness among consumers to buy no or limited number
of crackers, etc. will be far better than a complete ban for a few days.

Literature
1. (A) (a) Lencho wrote in his letter to God that he urgently needed one hundred pesos to
sow his fields again and to feed his family until the next crop. He went to city, put
a stamp on the envelope and dropped it into the mailbox at the post-office.
(b) The young woman’s hair is of yellow colour. She says that it’s not difficult for
her to get her hair dyed to brown, black, green or any other colour. She wanted
to change the colour of her hair so that the young men desirous of her attention
would love her for her inner qualities and not her yellow hair.
(c) The pilot of Dakota DS 088 saw the storms when he was 150 km away from Paris.
All the instruments of his aeroplane suddenly stopped working. There were dark
clouds all around him so he could not see anything. Clouds appeared to him like
huge mountains.
(B) (a) An old woman called her “a shameless girl.” She also called her “a harmless dumb
cow.” Bholi turned defensibly on the old woman. She told her that everybody
had thought of her to be a dumb cow and so they wanted to hand her over to
such a heartless creature. But the dumb cow had learnt to speak and would be
silenced no more.
(b) Griffin wanted to go to a place where he could relax and get protection from the
chilly winter days. He also needed warm clothes and something to eat. So, he
slipped into a big London store and stole some money and food from there. He
also opened boxes and took out warm clothes and clothed himself.
(c) Richard Ebright’s mother helped him in all possible ways. She adopted every
means to encourage his interest in learning. She took him on field trips, brought
telescopes for him, she also brought other things like microscopes, cameras,
mounting materials and other equipment. If he did not have anything to do, she
found things for him to learn.
2. (A) (a) The young seagull was afraid to fly because he was fearful that he would fall. It
was his first flight so he didn’t have courage and confidence. He thought that his
wings would not support him if he tried to fly. It is absolutely natural and common
that trying something for the first time can be a bit challenging and frightening. It is
possible that every bird is afraid of making its first flight. Similarly, a human baby
is also fearful when it takes the first step to try to walk. It also finds it challenging
when it learns to sit, crawl or stand up without assistance.
(b) The tiger feels as if he is enslaved in the concrete cell. It gives him a feeling of
being a prisoner. He longs to be free in the forest where he should be lurking in
the shadows following his prey in the long grass near the water hole, waiting to
attack the plump deer who come there to drink water. He ignores his visitors. He
is helpless and walks about the limited space of his cell.
(c) The poem ‘How to Tell Wild Animals’ by Carolyn Wells revolves around the
various ways of how to identify the wild animals. The poet has made an attempt
to distinguish one animal from the other in a humorous way. The poet suggests
that although adventurous but it can be very risky to be in such a close proximity
to these wild beasts. Some other titles that can fit the theme of the poem are
“Unique Wild Animals”, “How to Identify Wild Animals”, etc.

148 ACME English Practice Book GR–10 (Teacher Manual)

HF_Acme_ENG_TM_G10.indd 148 5/22/2021 5:58:54 PM


(B) (a) Hari Singh took a new name every month. That kept him ahead of the police and
his former employers. He never used to stay at the same place for a long time.
He was constantly on the move to escape being caught by police.
(b) Matilda was a pretty woman who always dreamt of living a lavish and luxurious
life. But unfortunately, she was born in a family of clerks that had limited means
of sustenance. She was also married to a clerk. Matilda would always dream of
beautiful things like attractive frocks and expensive jewels which she didn’t possess.
So she remained perpetually unhappy and sad.
(c) When the lawyer reached the railway station of the small town, he went to the
hack driver and told him that he had to find a man named Lutkins. He also told
him that he was in a hurry as he was to return in the afternoon train. Since it was
apparent to Bill that the lawyer had no idea how Lutkins looked liked, he decided
to fool him. When he went to bring his hack for the lawyer, he made his plan of
taking the lawyer around the town and making a fool of him.
3. (a) Buddha’s first Sermon at Benares was regarding the acceptance of death as the
ultimate reality that cannot be changed. Grieving and lamenting over the past
incidents and dead people cannot reduce the pain or revive the dead. But it can
only lead to misery and suffering.
Man is mortal, so his end or death is inevitable just like various other natural
phenomena. The cycle of birth and death is continuous and goes on ceaselessly.
It is very necessary to accept this truth of life to make life bearable. Human life is
transient, so one must help others and live life to the fullest. I agree with the lesson
of life given by Gautama Buddha. In modem times, people have very hectic lives
and they are running in the mad race to achieve more than the next person. Due
to this they have no peace of mind. People are only concerned about amassing
more material wealth. They have lost touch with their inner self. They need to
understand that life is unpredictable and death is the ultimate truth of life. So
they need to sit back and introspect about their life. No matter how much money
they earn or accumulate, they can’t defeat death. It is useless to run after material
wealth and worldly, short-lived success.
(b) Natalya and Lomov fought over the ownership of Oxen Meadows. Later, they
fought over the superiority of their dogs, Squeezer and Guess. It pained Lomov a
lot when he heard from Natalya that his dog, Guess had two major defects. One
is that the dog was old and second that it was short in the muzzle. Lomov replied
that he was proud that his dog was a pure breed and it had well sprung ribs. It
was also a good hunter. He called Natalya’s Squeezer a bad hunter. She argued
that her dog, Squeezer was far better than Guess. She tried to degrade Guess by
calling it old, ugly and as worn out as a cab horse. She also called Guess a lame
dog. Lomov clarified that it had become lame after its leg had been bitten by
some other dog. Lomov’s real purpose to come there was to propose Natalya for
marriage but, instead they indulged themselves in silly arguments over trivial issues.
Their main purpose was sidelined in the dispute. After Chubukov intervened, both
set their issues aside and agreed to get married.
4. (a) The hack driver told the narrator that it was a real terror to meet Lutkins’ mother.
He told him that she was a large and hefty lady with a furious temper. He also
said that she was quick as a cat. He told him that she was about nine feet tall and
four feet thick. He shared his own bitter experience with her. He told that once she
almost had taken his skin off when he had taken a trunk for her at her farmhouse.

Learnwell
ACME Math–III
ENGLISH Practice Book GR–10 (Teacher Manual) 149

HF_Acme_ENG_TM_G10.indd 149 5/22/2021 5:58:54 PM


This was because he had not treated the trunk like a box of eggs. When Bill told
Lutkin’s mother that the narrator was a lawyer and he had come there to search
her property, Lutkin’s mother invited them to the kitchen. Then she took out an
iron rod from the stove and threatened to burn them with that. Then she chased
them out and laughed at them. She did so to scare the lawyer away so that he
could not serve summons to Lutkins.
(b) Mrs Pumphrey was a prosperous woman who loved and pampered her dog very
much. She provided every item necessary for living a lavish life to Tricki from
fancy fur coats, dresses to expensive beds. Not only this, she was so concerned
for Tricki that she used to overfeed him. In her effort to make Tricki stronger, she
used to serve him cod-liver oil and malt between the main meals and Horlicks
after dinner. She never realised that Tricki was turning obese and her overfeeding
was spoiling his health. Soon her overfeeding made Tricki extremely sick. The
dog became lazy, inactive and obese. He kept lying on his rug and panting all
day long. Tricki fell ill to such an extent that he had to be taken to a surgery. Even
in the surgery, she continued to send eggs, wine and brandy for him. It was only
after the doctor gave the dog good exercise and nutritious food that his condition
improved. Hence we can say that excess of everything, from food to love, is bad.

TEST PAPER 5

Reading
(1) 1. (c) A retired civil engineer 2. (b) A cold desert 3. (i) (a), (b)
4. (a) This has resulted in water shortage and droughts.
5. (b) agriculture totally depends on glacier melt
6. (a) 15 7. (b) Padma Shree 8. (iv) (b), (f)
9. (a) 3 to 4 10. (a) Chewang Norphel, the messiah of Ladakh
11. (b) Conferred 12. (iv) Option (d)
(2) 1. (a) To honour mothers all over the world 2. (d) Greece
3. (iv) F – (a), (d) and O – (b), (c)
4. (c) She wanted the day to be only about sentiment and love.
5. (c) On fourth Sunday of Lent
6. (d) Her mother had done that as a way to heal the scars of racism and slavery.
7. (d) Woodrow Wilson
8. (c) She was disturbing the peace at a War Mother’s Convention.
9. (d) Some Eastern European and Central Asian countries
10. (c) Arab Nations 11. (i) Option (a) 12. (iii) Option (c)

Literature
1. (A) 1. (c) To reaffirm his faith in God 2. (d) All of these
3. (d) both (a) and (b) 4. (i) F – (a), (b) and O – (c), (d)
5. (a) Contentment
(B) 1. (ii) Option (b)
2. (d) they vow never to let discrimination practiced in their country
3. (c) Both (a) and (b) 4. (d) both (a) and (b) 5. (c) Emancipation

150 ACME English Practice Book GR–10 (Teacher Manual)

HF_Acme_ENG_TM_G10.indd 150 5/22/2021 5:58:54 PM


2. (A) 1. (a) He looked terrifying. 2. (iv) Options (e), (f)
3. (iii) F – (a) and O – (b), (c), (d)
4. (iv) Option (d) 5. (a) aabb
(B) 1. (a) The tiger in the zoo 2. (b) He is in a state of longing for freedom.
3. (i) F – (a), (b) and O – (c), (d) 4. (d) abcb 5. (iii) Option (c)

Grammar
1. (a) (ii) a (b) (i) and (c) (iv) are
2. (a) (iv) why she hadn’t come
(b) (i) when she was going (c) (iii) with whom she was travelling
3. (a) (i) does (b) (i) travel (c) (i) will
(d) (i) is (e) (i) is (f) (iii) The

Writing
1. (a) House no. 1
Abc Colony
XYZ city

9th January 20xx

The Manager
Customer Care
Herbal Tea

Sub: Enquiring about the herbal tea

Dear sir
This is with reference to the advertisement of the new herbal tea launched by
your company published in The Tribune on 8th January 20XX. I would like to
enquire certain details about this newly launched herbal tea.
My queries are-
• How is the tea prepared? What are the ingredients used in it?
• Does this have any side effects on the body?
• How often can one have it in a day?
• How can I order the tea? Is there any subscription offer?
It would be very humble of you to answer the above cited questions. It would
help me in making an early decision. Please answer me in the next letter.

Thanking you
Yours truly
Megha

Learnwell
ACME Math–III
ENGLISH Practice Book GR–10 (Teacher Manual) 151

HF_Acme_ENG_TM_G10.indd 151 5/22/2021 5:58:54 PM


(b) House no. 1
Abc Colony
XYZ City

9th January 20xx

The Editor
The Tribune
New Delhi

Sub: Growing menace of domestic violence against women

Sir
Through the columns of your esteemed newspaper, I would like to draw the
attention of everyone towards the growing menace of domestic violence against
women.
Women are often the victims of violence in their own households. Most of the
married women face domestic violence from their husbands; whereas unmarried
women are treated violently by their parents, siblings or teachers. They continue to
be the silent victims instead of seeking any help. This is because since childhood,
girls observe their household ladies being oppressed and they grow up to believe
that they are weak and are destined for this ill treatment against them.
It is necessary that women must be provided with good education that makes
them self-dependent, and also aware about how to fight the injustices done against
them. The males should also be raised in such a manner that they learn to treat
women equally and respectfully. The government and the people need to come
together to fight violence against women. Strict laws must also be made by the
Government for the safety of women. I believe my views will find a place in the
columns of your newspaper and help to bring about a change in the society in
favour of women.

Thanking you
Yours truly
Viraaj

2. (a) The given pie chart shows the sale of Coca-Cola Company in various regions of
the world in the year 2000. It is clearly evident from the pie chart that the sale
of the soft drink is 25.7% in Latin America, 30.4% in North America and 7.0%
in Africa and the Middle-East. The share of sales is 15.4% in Asia and 20.5 % in
Europe. The highest sale is in North America.
The line graph shows information about the share prices for Coca-Cola over
a period of five years, i.e. from 1996 to 2001. The prices of shares rose up to
approximately 68% between the year 1996 and 1998. Later, the prices decreased
to around 60% in 1997 itself. In 1998, the share prices were the highest, i.e. about
75%. The share prices continued to go up and down between 1998 and 2001.
In the year 1996, the prices were the lowest, i.e. about 35%.

152 ACME English Practice Book GR–10 (Teacher Manual)

HF_Acme_ENG_TM_G10.indd 152 5/22/2021 5:58:54 PM


(b) The bar graph shows the changes in population demography of Sikkim over a
period of ten years, i.e. from 2001 to 2011. It is evident from the graph that the
total population in the 2011 census is 6 lakh, i.e. an increase from the figure of
5.5 lakh in the 2001 census. Total population of Sikkim as per the 2011 census
is approximately 600000 of which around 3,10,000 people are males, while the
remaining 2,90,000 are females. In 2001, the total population of Sikkim was
around 550,000 of which around 2,90,000 people were males while the remaining
2,60,000 were females. The total population growth in the decade was 12 percent.

Literature
1. (A) (a) Rajvir displayed his knowledge about tea plantation by telling Pranjol’s father that
it was the second sprouting period of the year that lasts from May to July and
gives excellent yield. So, Pranjol clearly knew a lot more than Rajvir’s father had
expected. This made Rajvir’s father very impressed with Pranjol.
(b) Valli found her woman co-passenger to be ‘absolutely repulsive’ as she noticed
that there were big holes in her earlobes which carried ugly earrings in them. She
was chewing betel nut and the betel juice was about to spill over her lips. Thus,
Valli did not feel like befriending her.
(c) The phrase ‘O there are other balls’ suggests that the loss is not important enough
to worry about. Here, the ball that the child lost was his prized possession. So, it
was pointless to console him by saying that his loss can be easily compensated
by any other materialistic thing (another ball).
(B) (a) Tricki’s health made a rapid progress in the surgeon’s care. He was not given
any medicines. Instead, he was provided with opportunities for enough physical
exercise and his diet was strictly controlled. He was given food only when he was
hungry. Thus the dog got healthy soon.
(b) The narrator did not like his job as a junior assistant clerk where he only had to
serve summons. He wanted to be a real lawyer in a good city. After Bill’s friendly
description of the people of New Mullion, he had started to like the city, and thus
considered returning there to practise law.
(c) A sense of real Science entered Ebright and he realised that winners try to do real
experiments and not just put up a simple neat display. So, when he didn’t win
any prize at the Science Fair, he didn’t lose heart. The incident motivated him to
become a bright scientist.
2. (A) (a) Lomov told Natalya and Chubukov that his aunt’s grandmother gave the Meadows
to Chubukov’s grandfather’s peasants for the temporary and free use. So he was
the rightful owner of the Oxen Meadows. But when they kept on claiming the
ownership over the Meadows, Lomov offered to show them the documents and
even threatened them to the court.
(b) Wanda’s family left the city and moved to another city because her father didn’t
want her daughter Wanda to feet hurt when the children teased her. He was of
an opinion that in bigger cities, people are accustomed to seeing people from
different cultures and races and have a better understanding of cultural disparities
and sensibilities. So, it may be different for the family there as people won’t bother
to tease them by calling them ‘pollack’ or by asking them why their name was
funny.

Learnwell
ACME Math–III
ENGLISH Practice Book GR–10 (Teacher Manual) 153

HF_Acme_ENG_TM_G10.indd 153 5/22/2021 5:58:54 PM


(c) In the poem ‘Animals’ by Walt Whitman, the poet feels more comfortable with
animals than human beings because he believes that animals have much better
qualities than humans, and they do not possess any human vices. He considers
animals to be uncomplicated and satisfied with their lives. He finds that animals,
unlike human beings, are not hypocrites. They have no pretence. They accept
the poet in his actual state and present themselves to the poet as they are.
(B) (a) Indeed, Matilda’s unrealistic desires brought about her and her husband’s ruin.
She longed and cared too much about materialistic things. She could have easily
avoided the crisis she had to face if she had not desired more than her means.
Her mind was always occupied with the thought that she deserved all the luxuries
of life, and thus she always remained unhappy and dissatisfied with her current
life as a normal clerk’s wife. She lost the borrowed necklace and had to purchase
a new one to replace it unnoticed by taking a loan whose repayment ruined the
life of Matilda and her husband completely.
(b) Hari Singh felt guilty and decided to return back to Anil because he realised the
consequences of Anil’s knowledge of theft. He felt that Anil would definitely feel
sad, not for the loss of money but for the loss of trust. He also realised that if he
escaped, he would never be able to learn to read and write. His inner conscience
pricked him. He was a transformed person now as he wanted to be an educated
man. So, he returned back and placed the money back where he had taken it
from.
(c) Griffin was a scientist who had made a discovery by the means of which he
became invisible. He could have used this discovery for performing some good
deeds. But he was rather a lawless scientist, who misused a scientific discovery
for his own petty, selfish gains. His discovery proved temporarily useful for him.
As the use of his invisibility increased, so did his problems. He had to live a dual
life which was not respectful. His discovery turned himself into a criminal.
3. (a) When the postmaster read Lencho’s letter to God, he was amazed by his faith in
God. He didn’t want that such unmatched and exceptional faith of someone in
God be shaken. So he thought of a brilliant idea, that was, to answer the letter.
He collected money from other post office employees, friends and relatives and
sent money to Lencho in order to keep Lencho’s faith in God alive. He signed
it ‘God’ so that Lencho’s faith would not get shaken. After receiving the letter
Lencho became angry. Because when he counted the money he found only 70
pesos, but he had asked God to send 100 pesos. He was angry as he thought
that the post office employees had stolen the remaining 30 pesos. He called the
post office employees a brunch of crooks. He wrote the letter to God again and
requested Him not to send money through post office employees.
(b) Walt Whitman’s poem shows that he is very fond of living in the company of
animals. He wishes to live with the animals because he thinks that they are
calm and composed. They are well contented. They do not complain about
their condition like human beings do. Animals do not even need to bow to their
ancestors. They are always satisfied and do their duty towards God. They are
more loving and caring than humans. He believes that animals have much better
qualities than humans as they do not possess any human vices. He strongly feels
that the animals are uncomplicated and wiser than the human beings. Animals
unlike human beings are not hypocrites; they have no pretence, and they accept
the poet as he is and present themselves to the poet as they are.

154 ACME English Practice Book GR–10 (Teacher Manual)

HF_Acme_ENG_TM_G10.indd 154 5/22/2021 5:58:54 PM


4. (a) One day, the Halls got up early and were surprised to see that the door of the
scientist’s room was open. Normally, it remained shut and locked. The Halls
peeped into the room but there was no one inside. They went inside to investigate.
They saw that the bed clothes were cold and Griffin’s clothes and bandages were
lying loose in the room. Suddenly, Mrs Hall heard a sniffing sound close to her
ear. This completely shocked and scared her because there was no one in the
room. Then, suddenly, the hat that was kept on the bedpost moved up itself and
dashed into her face. To their greater shock, suddenly the chair rose up in the
air and pushed the Halls out of the room. They both got terrified and Mrs Hall
was almost about to fall down the stairs in hysterics. She was convinced that the
room was haunted by spirits, and that the stranger Griffin had somehow caused
the spirits to enter into her furniture.
(b) Ausable was an intelligent spy. When he found Max waiting for him in his room,
he immediately hatched a plan and invented a story to protect himself. He told
Max there was a balcony under his window which belonged to the next apartment
and somebody had entered his room through it the previous month. By using
his wits, Ausable made Max believe in the existence of the balcony, which was
actually just a work of Ausable’s imagination and wit. When someone knocked at
his door, Ausable said that it was the police who had come to give him protection
as he had some important papers with him. Due to the repeated knocking, Max
became nervous. He wanted to escape from the police. He told Ausable that he
would wait in the balcony and asked Ausable to send the police away or he would
shoot Ausable. So, he jumped out of the window and fell down as there was no
balcony there in reality. Ausable’s ability to think quickly and calmly turned the
tables on Max.

Printed at : MBD PRINTOGRAPHICS (P) LTD.


Industrial Area Ram Nagar, Gagret, Tehsil Amb, Distt. Una (H.P.)
Learnwell
ACME Math–III
ENGLISH Practice Book GR–10 (Teacher Manual) 155

HF_Acme_ENG_TM_G10.indd 155 5/29/2021 1:35:07 PM


HF_Acme_ENG_TM_G10.indd 154 5/22/2021 5:58:54 PM

You might also like